Download as pdf or txt
Download as pdf or txt
You are on page 1of 128

LAND TITLES AND DEEDS

DIGESTS
CASES 39-85

Block 2E 2022
Atty. Robert Nomar Leyretana
39. HEIRS OF THE LATE SPOUSES PEDRO S. PALANCA, ET. AL.
Resources Office stating that "the islands of Talampulan and Capar
v. REPUBLIC OF THE PHILIPPINES
Island located in the municipality of Busuanga, Palawan are within the
August 30, 2006 | J. Azcuna | Land of Public Domain: When Alienable
unclassified public forest."
and Disposable
BARLONGAY & CHECKER
ISSUE:
● W/N the lands in question are alienable lands. ​No.
PETITIONER: ​Heirs Of The Late Spouses Pedro S. Palanca And
Soterranea Rafols Vda. De Palanca Namely: Imelda R. Palanca, Mamerta RULING: ​CFI as land registration court has no jurisdiction to decree the
R. Palanca, Ofelia P. Miguel, Estefania P. Pe, Candelaria P. Punzalan, registration of the questioned lands since they are not alienable lands,
Nicolas R. Palanca, Constantino R. Palanca, Edmundo Palanca, Leocadia otherwise, the tile is null and void.
R. Palanca And Oliverio R. Palanca, Represented By Their
Attorney-In-Fact, Ofelia P. Miguel The possession of public (forest) land on the part of the claimant,
RESPONDENT: Republic Of The Philippines, (Represented By The however long, cannot convert the same into private property. Possession
Lands Management Bureau), Regional Trial Court Of Palawan (Office in such an event, even if spanning decades or centuries, could never ripen
Of The Executive Judge) And The Register Of Deeds Of Palawan into ownership. It bears stressing that unless and until the land classified
as forest is released in an official proclamation to that effect so that it
RECIT-READY: T​he heirs of Pedro S. Palanca, (petitioners herein), may form part of the disposable lands of the public domain, the rules on
filed an application to bring the pieces of land they allegedly owned confirmation of imperfect title does not apply.
under the operation of the Land Registration Act. They acquired said
realties by inheritance from the late Pedro S. Palanca, who had occupied While it is true that the land classification map does not categorically
and possessed said land openly and continuously in the concept of an state that the islands are public forests, the fact that they were
owner 39 years before the filing of said application, and planted on said unclassified lands leads to the same result. In the absence of the
lands coconut trees, declared the same for taxation purposes and paid the classification as mineral or timber land, the land remains unclassified
taxes.Petitioners likewise presented six witnesses in support of their land until released and rendered open to disposition. When the property is
application. Alfonso Lucero as one of the witnesses testified that he is a still unclassified, whatever possession applicants may have had, and
Forester in the Bureau of Forestry before and he is convinced that the however long, still cannot ripen into private ownership. This is because,
lands in question have already been released before the war for pursuant to Constitutional precepts, all lands of the public domain belong
agricultural purposes in favor of Pedro S. Palanca through which a to the State, and the State is the source of any asserted right to ownership
certification was issued by his office. in such lands and is charged with the conservation of such patrimony.
Thus, the Court has emphasized the need to show in registration
After trial, the CFI of Palawan issued a decision declaring petitioners as proceedings that the government, through a positive act, has declassified
the owners in fee simple of the two parcels of land in question. After inalienable public land into disposable land for agricultural or other
almost 23 years, respondent Republic of the Philippines filed with the CA purposes.
a petition for annulment of judgment, cancellation of the decree of
registration and title, and reversion. DOCTRINE:​The possession of public (forest) land on the part of the
Respondent presented a Land Classification Map showing that the claimant, however long, cannot convert the same into private property.
subject properties were unclassified lands as of that date as well as a Possession in such an event, even if spanning decades or centuries, could
certification issued by the Community Environment and Natural never ripen into ownership. It bears stressing that unless and until the
● Both Constantino Palanca and Ofelia Palanca-Miguel testified that:
land classified as forest is released in an official proclamation to that
(1) they were heirs of one Pedro S. Palanca; (2) they, together with
effect so that it may form part of the disposable lands of the public
their other siblings, were applicants for the registration of two
domain, the rules on confirmation of imperfect title does not apply.
parcels of land located in Barrio Panlaitan, Busuanga, Palawan; (3)
their father, Pedro S. Palanca, acquired ownership over the subject
FACTS: properties by continuous, public and notorious possession; (4) their
● On July 19, 1973, the heirs of Pedro S. Palanca, (petitioners father built a house on each parcel of land and planted coconut
herein), filed an application to bring 2 pieces of land (in Palawan) trees; (5) since their father’s death, they have continued their
they allegedly owned under the operation of the Land Registration possession over the lands in the concept of owners and adverse to
Act. all claimants; and (6) the properties have been declared for taxation
● They acquired said realties by inheritance from the late Pedro S. purposes and the corresponding taxes religiously paid for over
Palanca, who had occupied and possessed said land openly and forty (40) years.
continuously in the concept of an owner since 1934, or 39 years ● Alfonso Lucero testified that he is a Forester in the Bureau of
before the filing of said application, and planted on said lands Forest Development, formerly the Bureau of Forestry. His duty
about 1,200 coconut trees on each land, declared the same for was to supervise the team that conducted the limitation,
taxation purposes and paid the taxes. segregation and deviation of agricultural lands within the area
● During the initial hearing of the case, verbal oppositions to the where the lands in question are situated. As such, he issued
application were made by the Provincial Fiscal of Palawan certifications after due classification by his office, of alienable and
● The Provincial Fiscal stated that the lands subject of the disposable land. He identified Exhibits "JJ" and "KK" to be
application had no clearance from the Bureau of Forestry and that certifications to the effect that Talampulan in Panlaitan Island and
portions thereof may still be part of the timberland block and/or Talampetan, a portion of Capari Island, both in Busuanga
public forest under the administration of the Bureau of Forestry (formerly Coron), Palawan, are fully cultivated and mainly planted
and had not been certified as being alienable and disposable by the to coconuts before World War II by herein applicants, the heirs of
Bureau of Lands. Pedro S. Palanca. He is fully convinced that the lands in question
● After the lapse of three years from the date of the initial hearing, have already been released before the war for agricultural purposes
however, no valid and formal opposition was filed by any of the in favor of Pedro S. Palanca. On the basis of the Bureau of
oppositors in the form and manner required by law. Neither did Forestry investigation, a certification was then issued as to its
the Provincial Fiscal present witnesses from the relevant availability for the purpose for which the application was made.
government bureaus and agencies to support his contention that the ● Augustin O. Timbancaya testified that he is a licensed geodetic
subject lands had not yet been cleared for public disposition. engineer, formerly called a land surveyor. His services were
● On the other hand, petitioners submitted the plan and technical engaged by applicant Ofelia P. Miguel, the representative of the
description of the land, a survey certificate approved by the Bureau other applicants, to conduct and prepare a land plan for two parcels
of Lands and also tax declarations showing that they have of land subject of the application. He believes that both parcels of
consistently paid the realty taxes accruing on the property. land have been released for agricultural purposes because if it were
Petitioners likewise presented six witnesses in support of their otherwise, the survey plans he executed would not have been
application, namely Constantino Palanca, Ofelia Palanca-Miguel, approved by the Director of Lands.
Lopez Libarra, Alejandro Cabajar, Alfonso Lucero and Augustin ● After trial, the CFI of Palawan issued a decision on December 15,
Timbancaya. 1977 declaring petitioners as the owners in fee simple of the two
parcels of land in question.
● On December 6, 2000, or after almost 23 years, respondent Government of the Philippine Islands. They likewise argue that the
Republic of the Philippines filed with the CA a petition for CA erred in relying upon Executive Proclamation No. 219 and
annulment of judgment, cancellation of the decree of registration upon Land Classification Map No. 839, Project 2-A to nullify
and title, and reversion. Respondent sought to annul the December petitioners’ mother title. According to petitioners, the reversal of
15, 1977 decision of the CFI, arguing that the decision was null the CFI’s decision violated the principle of res judicata as well as
and void because the two lands in question were unclassified the rule on incontrovertibility of land titles under Act No. 496.
public forest land and, as such, were not capable of private ● Respondent: CFI has no jurisdiction to declare petitioners as the
appropriation. owners of the questioned lands since the said lands are not
● In support of this proposition, respondent presented a Land alienable lands.
Classification Map showing that the subject properties were
unclassified lands as of that date as well as a certification dated RATIO:
November 24, 2000 issued by the Community Environment and ● Section 48(b) of the Public Land Act upon which petitioners
Natural Resources Office stating that "the islands of Talampulan anchor their claim states:
and Capar Island located in the municipality of Busuanga, Palawan Sec. 48. The following-described citizens of the Philippines,
are within the unclassified public forest." occupying lands of the public domain or claiming to own any such
● Respondent likewise drew attention to Executive Proclamation No. lands or an interest therein, but whose titles have not been
219 issued on July 2, 1967 which classified the Province of perfected or completed, may apply to the Court of First Instance of
Palawan as a National Game Refuge and Bird Sanctuary and the the province where the land is located for confirmation of their
small islands off Palawan as national reserves closed to claims and the issuance of a certificate of title therefor, under the
exploitation and settlement under the administration of the Parks Land Registration Act, to wit:
and Wildlife Office, subject only to existing private rights. xxx
● Respondent also maintained that a culture of collusion existed (b) Those who, by themselves or through their
between and among the petitioners, the Provincial Fiscal and the predecessors-in-interest, have been in continuous, exclusive, and
ranking officer of the District Forestry Office, Alfonso Lucero, notorious possession and occupation of agricultural lands of the
such that the State was deprived of the opportunity to fairly present public domain, under a bona fide claim of acquisition or
its case to the court. ownership, for at least thirty years immediately preceding the filing
● CA favoured the respondent. of the application for confirmation of title, except when prevented
by war or force majeure. Those shall be conclusively presumed to
ISSUES: have performed all the conditions essential to a government grant
● W/N the lands in question are alienable lands. No. and shall be entitled to a certificate of title under the provisions of
this chapter.
RELEVANT ARGUMENTS (if any): ● The above provision clearly requires the concurrence of two
● Petitioner: Opine that it is not necessary for them to prove that the things: (1) that the land sought to be registered is public
government had expressly given a grant of the subject properties to agricultural land, and (2) that the applicant seeking registration
Pedro S. Palanca, separate of the legislative grant given to them must have possessed and occupied the same for at least thirty years
purportedly under Public Land Act. Petitioners also insist that a prior to the filing of the application.
particular land need not be formally released by an act of the ● The possession of public forests on the part of the claimant,
Executive before it can be deemed open to private ownership, however long, cannot convert the same into private property.
citing the cases of Ramos v. Director of Lands and Ankron v. Possession in such an event, even if spanning decades or centuries,
could never ripen into ownership. It bears stressing that unless and ● The court also pointed out that petitioners’ contention that the State
until the land classified as forest is released in an official has the burden to prove that the land which it avers to be of public
proclamation to that effect so that it may form part of the domain is really of such nature applies only in instances where the
disposable lands of the public domain, the rules on confirmation of applicant has been in possession of the property since time
imperfect title donot apply. immemorial. When referring to this type of possession, it means
● The Land Classification Map indicated that the Talampulan and possession of which no person living has seen the beginning and
Capari Islands on which the properties were located were the existence of which such person has learned from the latter’s
unclassified public lands as of December 9, 1929. It was by virtue elders. Immemorial possession justifies the presumption that the
of Executive Proclamation No. 219 issued on July 2, 1967 that land had never been part of the public domain or that it had been
these islands were subsequently classified as national reserves. private property even before the Spanish conquest. The possession
Based on these, it becomes evident that the subject properties have of petitioners in this case does not fall under the above-named
never been released for public disposition. Obviously, from the exception as their possession, by their own admission, only
time that petitioners and their predecessor-in-interest were commenced sometime in 1934.
occupying the properties in 1934 until the time that an application ● Where there is a showing that lots sought to be registered are part
for registration was filed in 1973, these properties remained as of the public domain, the applicant for land registration under
inalienable public lands. Section 48 of Commonwealth Act No. 141 must secure a
● While it is true that the land classification map does not certification from the government that the lands claimed to have
categorically state that the islands are public forests, the fact that been possessed by the applicant as owner for more than 30 years
they were unclassified lands leads to the same result. In the are alienable and disposable. Petitioners’ failure to do so in this
absence of the classification as mineral or timber land, the land case, when taken with the evidence adduced by respondent
remains unclassified land until released and rendered open to showing that the lands in question indeed remain part of the public
disposition. domain and form part of the national reserves, confirms that the
● Petitioners’ reliance upon Ramos v. Director of Lands and Ankron CFI never acquired jurisdiction to order the registration of such
v. Government is also misplaced. These cases were decided under lands in favor of petitioners, and certainly justifies their reversion
the Philippine Bill of 1902 and the first Public Land Act No. 926 to the State.
enacted by the Philippine Commission on October 7, 1926, under
which there was no legal provision vesting in the Chief Executive WHEREFORE,the petition is DENIED for lack of merit. No costs.
or President of the Philippines the power to classify lands of the
public domain into mineral, timber and agricultural so that the
courts then were free to make corresponding classifications in
justiciable cases.
● But as petitioners themselves admit, registration of the properties is
sought under Commonwealth Act No. 141. The classification or
reclassification of public lands into alienable or disposable,
mineral or forest lands is now the exclusive prerogative of the
Executive Department of the government. Clearly, the courts no
longer have the authority to determine the classification of lands of
the public domain.
40. BUENAVENTURA v. REPUBLIC
property by reason of open, continuous and exclusive possession of at
March 2, 2007 | Chico-Nazario, J. | Land of Public Domain: When
least 30 years Petitioners’ right to have their title to the subject property
Alienable and Disposable
registered cannot be defeated simply because the possession of
R. BARRALES & CHECKER
petitioners commenced on a date later than 12 June 1945

PETITIONER: ​Angelita and Preciosa Buenaventura DOCTRINE: ​SEC. 14. Who may apply. – The following persons may
RESPONDENT: ​Republic of the Philippines file in the proper Court of First Instance [now RTC] an application for
registration of title to land, whether personally or through their duly
RECIT-READY: ​Angelita and Preciosa are the applicants for authorized representatives:
registration of title over the subject property. They are the heirs of (1) Those who by themselves or through their predecessors-in-interest
spouses Amado Buenaventura and Irene Flores (spouses Buenaventura) have been in open, continuous, exclusive and notorious possession and
from whom they acquired the subject property. The facts reveal that the occupation of alienable and disposable lands of the public domain under
subject property was acquired by the spouses Buenaventura even ​before a bona fide claim of ownership since June 12, 1945, or earlier.
World War II​. However, it was only on ​3 January 1948 that the (2) Those who have ​acquired ownership of private lands by
corresponding Deed of Sale was executed in favor of the spouses prescription under the provisions of existing laws.
Buenaventura.
FACTS:
ISSUE: ​W/N the Petitioners have the right to have their title to the
subject property registered - ​YES
● Petitioners Angelita and Preciosa are the applicants for registration
of title over the subject property.
RULING: ​Section 14 of the Property Registration Decree speaks of who
● They are the heirs of spouses Amado Buenaventura and Irene
may apply for registration of land. There are three requisites for the filing
Flores (spouses Buenaventura) from whom they acquired the
of an application for registration of title under the first category, to wit:
subject property.
1. that the property in question is alienable and disposable land of the
● The facts reveal that the subject property was acquired by the
public domain;
spouses Buenaventura from the Heirs of Lazaro de Leon, namely:
2. that the applicants by themselves or through their
Aurelio de Leon and his sister Rodencia Sta. Agueda even ​before
predecessors-in-interest have been in open, continuous, exclusive
World War II​. (important)
and notorious possession and occupation; and
● However, it was only on ​3 January 1948 that the corresponding
3. that such possession is under a bona fide claim of ownership since
Deed of Sale was executed in favor of the spouses Buenaventura.
12 June 1945 or earlier.
● In 1978, the spouses Buenaventura transferred, by way of Deed of
The second classification relates to the ​acquisition of private lands by
Sale, the subject property, together with the adjacent property, to
prescription. ​This Court found out that petitioners offered in evidence a
their children, among whom are herein petitioners.
certification from the DENR, dated 29 October 2001, to prove that the
● Petitioners then filed an Application for Registration of Title
subject property was alienable and disposable land of the public domain.
before the RTC of Parañaque City of the subject property, located
It is well-settled that properties classified as alienable and disposable land
in San Dionisio, Parañaque City.
may be converted into private property by reason of open, continuous and
● Petitioners alleged that "they and their predecessors-in-interest
exclusive possession of at least 30 years. It is well-settled that properties
acquired title to the said parcel of land thru inheritance, transfer,
classified as alienable and disposable land may be converted into private
and possession as owners of the same since time immemorial fide claim of ownership since ​June 12, 1945, or earlier. ​(2) Those
and/or within the period provided for by law.” who have acquired ownership of private lands by ​prescription
● In order to establish the fact that petitioners and their predecessors under the provisions of existing laws.
have acquired vested right over the subject property by their open, ● From the provisions of the Property Registration Decree, there are
continuous, and exclusive possession under a bona fide claim of three requisites for the filing of an application for registration of
ownership for over 50 years completely unmolested by any title under the first category, to wit:
adverse claim. 1. that the property in question is alienable and disposable land
● Meaning, their possession of the subject property was in the of the public domain;
manner and for the period required by law; likewise, to prove the 2. that the applicants by themselves or through their
alienable and disposable character of the subject property. predecessors-in-interest have been in open, continuous,
● They also presented and identified several documents offered in exclusive and notorious possession and occupation; and
evidence, which tend to establish further the following: (1) 3. that such possession is under a bona fide claim of ownership
petitioners’ fee simple title over the subject property;(2) the nature since 12 June 1945 or earlier.
of the possession and occupation of the property;(3) its ● The second classification relates to the ​acquisition of private
classification as part of the alienable and disposable zone of the lands by prescription.
government​; and(4) the improvements introduced thereon and the ● This Court found out that petitioners offered in evidence a
taxes paid on the subject property. certification from the DENR, dated 29 October 2001, to prove that
the subject property was alienable and disposable land of the
ISSUES: ​W/N the Petitioners have the right to have their title to the subject public domain. It is well-settled that properties classified as
property registered - ​YES alienable and disposable land
● The only reason posited by the appellate court (and SC) in denying
RELEVANT ARGUMENTS (if any): ​According to the Republic, to grant the application for registration of title of the petitioners
petitioners failed to prove continuous, open, exclusive and notorious was the fact that petitioners’ ​evidence was not sufficient to prove
possession by their predecessors-in-interest and by themselves; hence, the that their possession of the subject property was since ​12 June
trial court erred in granting petitioners’ application for registration of the 1945 or earlier.
subject property. ● It is well-settled that properties classified as alienable and
disposable land may be converted into private property by reason
RATIO: of open, continuous and exclusive possession of at least 30 years.
● Such property now falls within the contemplation of "private
● Section 14 of the Property Registration Decree speaks of who may lands" under Section 14(2), over which title by prescription can be
apply for registration of land. The said provision of law refers to an acquired.
original registration through ordinary registration proceedings. ● Hence, because of Section 14(2) of Presidential Decree No. 1529,
● SEC. 14. Who may apply. – The following persons may file in the those who are in possession of alienable and disposable land, and
proper Court of First Instance [now RTC] an application for whose possession has been characterized as open, continuous and
registration of title to land, whether personally or through their exclusive for 30 years or more, may have the right to register their
duly authorized representatives: (1) Those who by themselves or title to such land despite the fact that their possession of the land
through their predecessors-in-interest have been in open, commenced only after 12 June 1945.
continuous, exclusive and notorious possession and occupation of ● IN ALL, petitioners were able to prove sufficiently that they have
alienable and disposable lands of the public domain under a bona been in possession of the subject property for more than 30 years,
which possession is characterized as open, continuous, exclusive,
and notorious, in the concept of an owner.
● By this, the subject alienable and disposable public land had been
effectively converted into private property over which petitioners
have acquired ownership through prescription to which they are
entitled to have title through registration proceedings.
● Petitioners’ right to have their title to the subject property
registered cannot be defeated simply because the possession of
petitioners commenced on a date later than 12 June 1945, for the
law and supplementing jurisprudence amply, justly and rightfully
provides the necessary remedy to what would otherwise result in
an unjust and unwarranted situation.

WHEREFORE, premises considered, the instant Petition is hereby


GRANTED.
41. LIMCOMA VS. REPUBLIC
In fact, the petitioner introduced in evidence a ​Certification 31
10 July 2007 | J. Nachura |
from the DENR-CENRO​, dated September 30, 2002 which
‘When is a land of public domain alienable and disposable?’ and the
reads:
Judicial Confirmation of Imperfect or Incomplete Titles.
Calilung & Checker
“This is to certify that the parcel of land identified as Lot 972-A,
Csd-04-015172-D, situated at Barangay Namuco, Rosario, Batangas
PETITIONER:​LIMCOMA MULTI-PURPOSE COOPERATIVE containing an area of SIX HUNDRED FORTY-SIX METERS and
RESPONDENT: ​REPUBLIC OF THE PHILIPPINES shown at the reverse side thereof has been verified to be within the
ALIENABLE AND DISPOSABLE ZONE under Project No. 27-A, land
RECIT-READY: Classification Map No. 718 certified on 26 March 1928.”
Limcoma Multi-Purpose Cooperative filed with the RTC an application
for registration and confirmation of title over a 646 sq meter parcel of The DENR-CENRO Report, likewise, contains the foregoing notation.
land in Barangay Namuco, Rosario, Batangas. Limcoma alleged that it is Further, the subject lot has been classified as commercial for tax
the owner in fee simple of the subject lot and the improvements thereon, purposes. These documents all point to the undeniable fact that the
and that it has been in the open, exclusive, peaceful, and continuous subject lot is public alienable land and, thereby, overcome the
possession thereof for more than 30 years, reckoned from the time of presumption that such forms part of the public dominion.
possession of its predecessors-in-interest. There were no private
oppositors, hence Limcoma presented the testimonies of Olivia P. In the recent case of Buenaventura v. Republic we ruled that said
Gomez, Arsenia P. Alcantara, and Lorenzo P. Limbo. (If you want more Certification is sufficient to establish the true nature or character of the
details on the testimonies check the facts.) RTC granted. CA reversed subject property as public and alienable land. We similarly ruled in
and set aside on the basis that: (Limcoma) ‘failed to (1) demonstrate the Republic v. Court of Appeal and intoned therein that the certification
open, continuous, exclusive, and notorious possession since June 12, enjoys a presumption of regularity in the absence of contradictory
1945 or earlier, required by the Property Registration Decree and the evidence.
Public Land Act; and (2) overcome the presumption that the subject lot is Both the DENR-CENRO Certification and Report constitute a positive
public and alienable land.’ Hence the present petition. government act, an administrative action, validly classifying the land in
question. As adverted to by the petitioner, the classification or
ISSUE: reclassification of public lands into alienable or disposable, mineral, or
1. W/N the subject lot is public and alienable land, and Limcoma has forest lands is now a prerogative of the Executive Department of the
been in open, continuous, exclusive, and notorious possession thereof government. Clearly, the petitioner has overcome the burden of proving
since June 12, 1945, or earlier, under a bona fide claim of ownership; the alienability of the subject lot.

2. Corollarily, whether or not the subject lot acquired a private character 2. The testimonies of petitioner's witnesses consistently declared that the
in 1968, thus within the operation of the laws on prescription. Spouses Andres and Trinidad occupied and possessed the subject lot in
the concept of owner since 1938. Worth noting is the testimony of
RULING: Lorenzo Limbo who had resided in and frequented the area since he was
1. The records reveal that the petitioner presented several a child and is thus familiar with the Spouses Andres' and Trinidad's
documents to prove that the subject lot is alienable public land. ownership of Lot 972. He gave direct and categorical testimony
consisting of specific acts of ownership to substantiate the petitioner's
Venustiano and Arsenia Alcantara on September 4, 1991. As a cooperative,
claim that the Spouses Andres and Trinidad possessed and occupied the
subject lot. Lorenzo Limbo certainly knew from whereof he spoke as his the petitioner is empowered by its Articles of Cooperation to hold real
father was the compadre of the Spouses Andres and Trinidad, he property. However, it is exempted from the payment of real estate taxes.
eventually married Trinidad's sister, and he had been a long time The subject lot is not within a military reservation or forest zone but falls
neighbor of the Alcantaras. under a commercial classification. There are no tenants on the subject lot
Moreover, petitioner proffered in evidence the TDs showing payment of and it is located along the provincial road. Upon continuation of Olivia's
realty taxes by the Spouses Andres and Trinidad from 1938 which was testimony, she identified the evidence to establish the historical ownership
subsequently continued by Venustiano and Arsenia. Although as a rule, of the subject lot traced back from the petitioner's predecessors-in interest.
tax declarations are not conclusive evidence of ownership, they are proof
Unfortunately, Tax Declaration (TD) No. 0884 12 could not be found in the
that the holder has a claim of title over the property and serve as
sufficient basis for inferring possession. These tax declarations bolster files of the Office of the Municipal Treasurer of Rosario, Batangas despite
the petitioner's claim that its predecessors-in-interest possessed and diligent efforts to locate said document. However, TD Nos. 00584 13 and
occupied the lot in question since 1938. 0452 both specifically refer to TD No. 0884 which provides the link to
reflect the transfer of ownership from the Spouses Andres and Trinidad
DOCTRINE: Alcantara to Venustiano. The character of petitioner's and its
In the recent case of Buenaventura v. Republic we ruled that predecessors-in-interest's possession of the subject lot was peaceful, open,
(DENR-CENRO Certification) is sufficient to establish the true nature or continuous, exclusive, and in the concept of owners. Olivia further testified
character of the subject property as public and alienable land. We
on the subject lot's classification as certified by the Department of
similarly ruled in Republic v. Court of Appeal and intoned therein that
the certification enjoys a presumption of regularity in the absence of Environment and Natural Resources (DENR) Community and Environment
contradictory evidence. and Natural Resources Office (CENRO). The CENRO Report dated
Both the DENR-CENRO Certification and Report constitute a positive September 23, 2002 stated, among others, that (1) Lot 972-A is not within
government act, an administrative action, validly classifying the land in a reservation or forest zone; (2) there is no previously issued patent, decree,
question. As adverted to by the petitioner, the classification or or title; (3) there is no public land application filed for the same by the
reclassification of public lands into alienable or disposable, mineral, or applicant (petitioner) or any other person; (4) the land applied for is
forest lands is now a prerogative of the Executive Department of the commercial in nature and is used as warehouse of feeds for animals; and (5)
government. Clearly, the petitioner has overcome the burden of proving
the land does not encroach upon an established watershed, riverbank
the alienability of the subject lot.
protection, creek, or right of way. Olivia further reiterated that the subject
lot is classified as commercial and within the alienable and disposable zone.
FACTS:
(Merely additional info on the land and testimonies. Not really necessary; In turn, ​Arsenia Alcantara identified Lot 972 and declared that it was
just in case.) previously owned by her parents-in-law, the Spouses Andres and Trinidad,
Olivia P. Gomez​, petitioner's Assistant General Manager, testified that she who had occupied and possessed said lot, in the concept of owner, since
knows the subject lot which has been occupied and used by the petitioner 1938. In 1982, Lot 972 was donated by the couple to their son, Venustiano.
from the date of purchase as its sales and warehouse office in Rosario, She came to know of her in-laws' ownership of Lot 972 a year after she and
Batangas. The subject lot was bought by petitioner from the Spouses Venustiano were married in 1953. In this regard, Arsenia testified that the
Spouses Andres and Trinidad planted and harvested several coconut trees, continuous, and exclusive, that they planted citrus on the said lot, and that
other crops, and vegetables thereon. She and Venustiano, likewise, stored the perimeter area was surrounded by madre de cacao, bamboos, and some
thereat some of their equipment and things used in their bakery. When wire. The subject lot is not tenanted, is located outside a military reservation
Venustiano became the owner thereof, they put up a perimeter fence on Lot or forest zone, and is without adverse claimants. Likewise, Lorenzo verified
972 and continued to use the property as a storage site for materials utilized Arsenia's testimony on the Spouses Andres and Trinidad's donation of the
in their bakery. Thereafter, Lot 972 was segregated into Lots 972-A and subject lot to Venustiano in 1982. Initially, Lorenzo only knew from his
972-B. Lot 972-A was the subject of the petitioner's application for wife that Lot 972 was donated to Venustiano by the latter's parents.
registration before the RTC while Lot 972-B was donated by Venustiano to Subsequently, however, he was able to confirm that the ownership of the
their daughter, Trinidad Alcantara. Finally, Arsenia identified and ratified subject lot had been transferred to Venustiano because it was of public
the Deed of Sale evidencing petitioner's acquisition of the subject lot. On knowledge and they became his neighbors in the same place.
cross-examination, Arsenia confirmed that there were no adverse claimants
over the subject lot, and her in-laws' possession thereof was peaceful, WHEREFORE, premises considered, the petition is GRANTED. The
adversarial, continuous, and open, which they (Venustiano and her) January 31, 2005 Decision of the Court of Appeals is hereby
eventually continued in like manner. REVERSED, and the April 10, 2003 Order of the Regional Trial Court
granting the petitioner's application for registration of the subject lot,
Lorenzo Limbo corroborated Arsenia's testimony on the Spouses Andres is hereby REINSTATED. No costs.
and Trinidad's possession and ownership of the subject lot since 1938.
Lorenzo testified that he was born and raised in Poblacion, Rosario, SO ORDERED.
Batangas, the same place where the subject lot is located. He was a resident
thereof for 59 years. In addition, Lorenzo declared that he knows the subject
lot which was originally part of Lot 972, and ownership thereof by the
Spouses Andres and Trinidad, because Lorenzo's family used to own a
parcel of land near the property. In fact, the Spouses Andres and Trinidad
were compadres of Lorenzo's father. Lorenzo was already a teenager in
1938 when the Spouses Andres and Trinidad possessed and tilled Lot 972
encompassing the subject lot. As a boy, Lorenzo frequented the property
near Lot 972, and, from 1938, he observed the Spouses Andres and Trinidad
working on and utilizing the land as storage site for their business. ASHaDT
Lorenzo's familiarity with Lot 972 and its previous owners was solidified by
his marriage to Trinidad's sister, which makes Lorenzo, Venustiano's uncle
by affinity. Indeed, Lorenzo asseverated that the Spouses Andres and
Trinidad were the recognized and acknowledged owners of Lot 972, and
they remained owners thereof when he got married on January 18, 1951. He
recalled that the Spouses Andres and Trinidad's possession was open,
42. REPUBLIC v. T.A.N. PROPERTIES
26 June 2008 | J. Carpio | Sec. 3, Art. XII of the Constitution and the The two certificates are INSUFFICIENT. The CENRO certificate
Judicial Confirmation of Imperfect or Incomplete Titles presented is beyond the authority of the CENRO to certify as alienable
M. del Rosario & T. dela Rosa and disposable based on DENR Administrative Order No. 20, series of
1988, the CENRO issues certificates of land classification status for areas
below 50 hectares. Moreover, the Provincial Environment and Natural
PETITIONER: ​Republic of the Philippines
Resources Offices (PENRO) is the one issuing certificates of land
RESPONDENT: ​T.A.N. Properties classification status for lands covering over 50 hectares.

RECIT-READY: ​T.A.N. Properties filed from an Application for Likewise, the FMS-DENR Memorandum is insufficient for having no
Original Registration of title covering a 56.4007 hectares of a lot in a probative value for lack of authority of the Regional Technical Director
subdivision plan which is a portion of a consolidated Sto. Tomas to issue the same under DAO Nos. 20 and 38.
Cadastre in Batangas. The only oppositor was the Republic of the
Philippines represented by the Director of Lands. The CENRO and Regional Technical Director, FMS-DENR,
certifications do not prove that the lot falls within the alienable and
ISSUE:​ (Note: The issues most relevant to the syllabus topic are in bold.) disposable land as proclaimed by the DENR Secretary. Such government
1. Whether the land is alienable and disposable (NO) certifications do not, by their mere issuance, prove the facts stated
2. Whether T.A.N. or its predecessors-in-interest had open, therein. In the present case, only Torres, respondent's Operations
continuous, exclusive, and notorious possession and occupation Manager, identified the certifications submitted by respondent. The
of the land in the concept of an owner since June 1945 or earlier government officials who issued the certifications were not presented
(NO) before the trial court to testify on their contents.
3. Whether T.A.N. Properties, a private corporation, cannot apply
for registration of the land of the public domain (NO) DOCTRINE: ​It is not enough for the PENRO or CENRO to certify that
4. Whether respondent is qualified to apply for registration of the a land is alienable and disposable. The applicant for land registration
land under the Public Land Act (NO) must prove that the DENR Secretary had approved the land classification
and released the land of the public domain as alienable and disposable,
RULING: ​The land is not alienable and disposable. and that the land subject of the application for registration falls within the
approved area per verification through survey by the PENRO or CENRO.
The rule is that all lands not appearing to be clearly of private dominion In addition, the applicant for land registration must present a copy of the
presumably belong to the State. It is upon the respondent to overturn such original classification approved by the DENR Secretary and certified as a
presumption. In this case, T.A.N submitted two certifications issued by true copy by the legal custodian of the official records. These facts must
the DENR. The first one is the 3 June 1997 Certification by CENRO be established to prove that the land is alienable and disposable.
certified that the disputed lot falls within ​the alienable and disposable
zone. The second certification in the form of a memorandum to the trial
court, which was issued by the Regional Technical Director, Forest
Management Services of the DENR (FMS-DENR), stated "that the
subject area falls within an alienable and disposable land, Project No. 30
of Sto. Tomas, Batangas.
FACTS: ISSUES:
● T.A.N. Properties filed from an Application for Original 1. Whether the land is alienable and disposable (NO)
Registration of title covering a 56.4007 hectares of a lot in a 2. Whether T.A.N. Properties, a private corporation, cannot apply for
subdivision plan which is a portion of a consolidated Sto. Tomas registration of the land of the public domain (NO)
Cadastre. 3. Whether respondent is qualified to apply for registration of the
● After publishing the Notice of Initial Hearing, the trial court, on 11 land under the Public Land Act (NO)
November 1999 called the case for initial hearing. The only
oppositor was the Republic of the Philippines represented by the RELEVANT ARGUMENTS: ​See ratio.
Director of Lands.
● T.A.N. Properties presented three witnesses: Anthony Dimayuga RATIO:
Torres (Torres), its Operations Manager and its authorized 1. ​T.A.N. Properties failed to prove that the land is alienable and
representative in the case; Primitivo Evangelista (Evangelista), a disposable.
72-year old resident of San Bartolome, Sto. Tomas, Batangas since ● The rule is that all lands not appearing to be clearly of private
birth; and Regalado Marquez, Records Officer II of the Land dominion presumably belong to the State. It is upon the respondent
Registration Authority (LRA), Quezon City. to overturn such presumption.· ​In this case, T.A.N submitted
● T.A.N.’s witnesses testified for the transfer of the lands: two certifications issued by the DENR.

1942 – 1960 Kabesang Puroy had peaceful, adverse, open, 1. ​The first one is the 3 June 1997 Certification by the

and continuous possession in the concept of Community Environment and Natural Resources Offices
an owner. (CENRO)​, Batangas City certified that the disputed lot falls within
the alienable and disposable zone.
1960 Upon Kabesang Puroy’s death, he was 2. ​The second certification in the form of a memorandum to the
succeeded by his son Antonio.
trial court, which was issued by the Regional Technical Director,
1960 Antonio executed a Deed of Donation in Forest Management Services of the DENR ​(FMS-DENR)​, stated
favour of one of his children, Fortunato. "that the subject area falls within an alienable and disposable land,
Later, however, Antonio gave Fortunato Project No. 30 of Sto. Tomas, Batangas.
another piece of land. ● ​The two certificates are INSUFFICIENT.
1. The ​CENRO certificate presented by T.A.N. covered the
1961 Antonio executed a Partial Revocation of
entire lot with an area of 596,116 square meters which, as per DAO
Donation, and the land was adjudicated to one
No. 38, series of 1990, is ​beyond the authority of the CENRO to
of Antonio’s children, Porting.
certify as alienable and disposable.
1997 Porting sold the land to T.A.N. Properties.
BASIS: Under DAO No. 20, series of 1988, the CENRO issues · ​It is not enough for the PENRO or CENRO to certify that a
certificates of land classification status for areas below 50 hectares. land is alienable and disposable. The applicant for land
The Provincial Environment and Natural Resources Offices registration must prove that the DENR Secretary had approved the
(PENRO) issues certificate of land classification status for lands land classification and released the land of the public domain as
covering over 50 hectares. alienable and disposable, and that the land subject of the application
for registration falls within the approved area per verification
2. The ​FMS-DENR Memorandum has ​no probative value through survey by the PENRO or CENRO. In addition, the
for lack of authority of the Regional Technical Director to issue the applicant for land registration must present a copy of the original
same. classification approved by the DENR Secretary and certified as a
true copy by the legal custodian of the official records. These facts
BASIS: under DAO Nos. 20 and 38 to issue certificates of land must be established to prove that the land is alienable and
classification. Under DAO No. 20, the Regional Technical Director, disposable.
FMS-DENR:
1. Issues original and renewal of ordinary minor products (OM) Only Torres, respondent's Operations Manager, identified the
permits except rattan; certifications submitted by respondent. The government officials
2. Approves renewal of resaw/mini-sawmill permits; who issued the certifications were not presented before the trial
3. Approves renewal of special use permits covering over five court to testify on their contents. The trial court should not have
hectares for public infrastructure projects; andcralawlibrary accepted the contents of the certifications as proof of the facts stated
4. Issues renewal of certificates of registration for logs, poles, piles, therein. Even if the · ​certifications are presumed duly issued and
and lumber dealers. admissible in evidence, they have no probative value in establishing
that the land is alienable and disposable.
Under DAO No. 38, the Regional Technical Director, FMS-DENR:
1. Issues original and renewal of ordinary minor [products] (OM)
permits except rattan; 2. There was No Open, Continuous, Exclusive, and Notorious
2. Issues renewal of certificate of registration for logs, poles, and Possession and Occupation in the Concept of an Owner
piles and lumber dealers;
3. Approves renewal of resaw/mini-sawmill permits; Ruling was discussed in Case #26 (same case).
4. Issues public gratuitous permits for 20 to 50 cubic meters within
calamity declared areas for public infrastructure projects; 3. ​T.A.N. Properties, a private corporation, cannot apply for
andcralawlibrary registration of the land of the public domain.
5. Approves original and renewal of special use permits covering
over five hectares for public infrastructure projects. Ruling was discussed in Case #26 (same case).
4. T.A.N. is NOT qualified to apply for registration of the land under
the Public Land Act

Ruling was discussed in Case #26 (same case).

WHEREFORE, ​we SET ASIDE ​the 21 August 2002 Decision of the


Court of Appeals in CA-G.R. CV No. 66658 and the 16 December 1999
Decision of the Regional Trial Court of Tanauan, Batangas, Branch 6 in
Land Registration Case No. T-635. We ​DENY ​the application for
registration filed by T.A.N. Properties, Inc.
43. REPUBLIC v. CA
were deemed alienable. In this case, petitioner has not alleged that the
August 6, 2008 | G.R. No. 155450 | Carpio | Land of Public Domain:
disputed portion had been declared as mineral or forest zone, or reserved
When Alienable and Disposable
for some public purpose in accordance with law, during the Spanish
LUCES | MIRANDA
regime or thereafter. When the trial court issued the decision for the
issuance of Decree No. 381928 in 1930, the trial court had jurisdiction to
PETITIONER: ​Republic of the Philippines represented by the Regional determine whether the subject property, including the disputed portion,
Executive Director, Department of Environment and Natural Resources, applied for was agricultural, timber or mineral land. The trial court
Regional Office No. 2 determined that the land was agricultural and that spouses Carag proved
RESPONDENT: ​Court of Appeals, Heirs of Antonio Carag and Victoria that they were entitled to the decree and a certificate of title. The
Turingan, The Register of Deeds of Cagayan, and the Court of First government, which was a party in the original proceedings in the trial
Instance of Cagayan court as required by law, did not appeal the decision of the trial court
declaring the subject land as agricultural. Since the trial court had
RECIT-READY: ​The CFI of Cagayan issued a Decree in favor of jurisdiction over the subject matter of the action, its decision rendered in
Spouses Carag covering a parcel of land situated in Cagayan. A 1930, or 78 years ago, is now final and beyond review.
letter-petition was filed with the DENR by a certain Bienvenida Taguiam
requesting the DENR to initiate the filing of an action for the annulment DOCTRINE:
of the decree on the ground that the trial court (CFI Cagayan) did not During the Spanish regime, Crown lands were per se alienable unless
have jurisdiction to adjudicate a portion of the subject property still falling under timber or mineral zones, or otherwise reserved for some
allegedly classified as timber land at the time of the issuance of the public purpose in accordance with law.
Decree. ​68 years after the issuance of Decree No. 381928​, the Republic
of the Philippines filed with the Court of Appeals a complaint to annul
FACTS:
and declare the nullity of titles on the same ground of lack of jurisdiction
● The CFI of Cagayan issued Decree No. 381928 in favor of spouses
of the CFI Cagayan. Petitioner claims that the disputed portion was still
Antonio Carag and Victoria Turingan, covering a parcel of land
classified as timber land, and thus not alienable and disposable, when
situated in Tuguegarao, Cagayan. Pursuant to said Decree, the
Decree No. 381928 was issued in 1930. Petitioner argues that in 1930 or
Register of Deeds of Cagayan issued Original Certificate of Title
in 1938, only the Executive Branch of the Government, not the trial
No. 11585 (OCT No. 11585) in the name of spouses Carag.
courts, had the power to declassify or reclassify lands of the public
● Bienvenida Taguiam Vda. De Dayag and others filed with the
domain.
Regional Office No. 2 of the Department of Environment and
Natural Resources (DENR), Tuguegarao, Cagayan, a letter-petition
ISSUE:
requesting the DENR to initiate the filing of an action for the
● W/N the CFI of Cagayan had jurisdiction to adjudicate a tract of
annulment of Decree No. 381928 on the ground that the trial court
timberland in favor of respondent spouses Antonio Carag and
did not have jurisdiction to adjudicate a portion of the subject
Victoria Turingan - ​YES
property which was allegedly still classified as timber land at the
time of the issuance of Decree No. 381928.
RULING:
● In a Memorandum, the Legal Division of the Land Management
Under the Spanish regime, all Crown lands were per se alienable. Thus,
Bureau recommended to the Director of Lands that an action for
unless specifically declared as mineral or forest zone, or reserved by the
the cancellation of OCT No. 11585, as well as its derivative titles,
State for some public purpose in accordance with law, all Crown lands
be filed with the proper court. The Director of Lands approved certification of the National Mapping and Resources Information
the recommendation. Authority, dated 27 May 1994, contained no statement that the
● 68 years after the issuance of Decree No. 381928​, petitioner filed disputed portion was declared and classified as timber land.
with the Court of Appeals a complaint for annulment of judgment, ● The law prevailing when Decree No. 381928 was issued in 1930
cancellation and declaration of nullity of titles on the ground that was Act No. 2874, which provides:
in 1930 the trial court had no jurisdiction to adjudicate a portion of
the subject property, which portion consists of 2,640,000 square SECTION 6. The Governor-General, upon the recommendation of
meters (disputed portion). The disputed portion was allegedly still the Secretary of Agriculture and Natural Resources, shall from
classified as timber land at the time of issuance of Decree No. time to time classify the lands of the public domain into —
381928 and, therefore, was not alienable and disposable until 22 (a) Alienable or disposable
February 1982 when the disputed portion was classified as (b) Timber and
alienable and disposable. (c) Mineral lands
● Petitioner contends that the trial court had no jurisdiction to and may at any time and in a like manner transfer such lands from
adjudicate to spouses Carag the disputed portion of the subject one class to another, for the purposes of their government and
property. Petitioner claims that the disputed portion was still disposition.
classified as timber land, and thus not alienable and disposable,
when Decree No. 381928 was issued in 1930. In effect, petitioner ● Petitioner has not alleged that the Governor-General had declared
admits that the adjacent 4,407,673 square meters of the subject the disputed portion of the subject property timber or mineral land
property, outside of the disputed portion, were alienable and pursuant to Section 6 of Act No. 2874.
disposable in 1930. Petitioner argues that in 1930 or in 1938, only ● It is true that Section 8 of Act No. 2874 opens to disposition only
the Executive Branch of the Government, not the trial courts, had those lands which have been declared alienable or disposable.
the power to declassify or reclassify lands of the public domain. Section 8 provides:

ISSUE: SECTION 8. Only those lands shall be declared open to


● W/N the CFI of Cagayan had jurisdiction to adjudicate a tract of disposition or concession which have been officially delimited and
timberland in favor of respondent spouses Antonio Carag and classified and, when practicable, surveyed, and which have not
Victoria Turingan - ​YES been reserved for public or quasi- public uses, not appropriated by
the Government, nor in any manner become private property, nor
RULING: those on which a private right authorized and recognized by this
● Under the Spanish regime, all Crown lands were per se alienable. Act or any other valid law may be claimed, or which, having been
Thus, unless specifically declared as mineral or forest zone, or reserved or appropriated, have ceased to be so. However, the
reserved by the State for some public purpose in accordance with Governor-General may, for reasons of public interest, declare lands
law, all Crown lands were deemed alienable. of the public domain open to disposition before the same have had
● In this case, petitioner has not alleged that the disputed portion had their boundaries established or been surveyed, or may, for the same
been declared as mineral or forest zone, or reserved for some reasons, suspend their concession or disposition by proclamation
public purpose in accordance with law, during the Spanish regime duly published or by Act of the Legislature.
or thereafter. The land classification maps petitioner attached to the
complaint also do not show that in 1930 the disputed portion was ● However, Section 8 provides that lands which are already private
part of the forest zone or reserved for some public purpose. The lands, as well as lands on which a private claim may be made
under any law, are not covered by the classification requirement in
Section 8 for purposes of disposition. This exclusion in Section 8
recognizes that during the Spanish regime, Crown lands were per
se alienable unless falling under timber or mineral zones, or
otherwise reserved for some public purpose in accordance with
law.
● Clearly, with respect to lands excluded from the classification
requirement in Section 8, trial courts had jurisdiction to adjudicate
these lands to private parties. Petitioner has not alleged that the
disputed portion had not become private property prior to the
enactment of Act No. 2874. Neither has petitioner alleged that the
disputed portion was not land on which a private right may be
claimed under any existing law at that time.
● When the trial court issued the decision for the issuance of Decree
No. 381928 in 1930, the trial court had jurisdiction to determine
whether the subject property, including the disputed portion,
applied for was agricultural, timber or mineral land. The trial court
determined that the land was agricultural and that spouses Carag
proved that they were entitled to the decree and a certificate of
title. The government, which was a party in the original
proceedings in the trial court as required by law, did not appeal the
decision of the trial court declaring the subject land as agricultural.
Since the trial court had jurisdiction over the subject matter of the
action, its decision rendered in 1930, or 78 years ago, is now final
and beyond review.

WHEREFORE, we DENY the petition. We DISMISS petitioner


Republic of the Philippines' complaint for reversion, annulment of
decree, cancellation and declaration of nullity of titles for lack of merit.
44. SECRETARY OF DEPARTMENT OF ENVIRONMENT AND
proclamation or an executive order; an administrative action;
NATURAL RESOURCES v. YAP
investigation reports of Bureau of Lands investigators; and a legislative
8 October 2008 | G.R. No. 167707 | R.T. Reyes | Regalian Doctrine
act or a statute.
Made by: Billie F. Blanco (“Ruling” added by J. Miranda)
ISSUE:
● Whether or not the claimants have a right to secure titles over
PETITIONER: ​Secretary of DENR their occupied portions in Boracay - ​NO
RESPONDENT: ​Mayor Jose Yap
RULING:
Recit Ready: ​At stake in these consolidated cases is the right of the A positive act declaring land as alienable and disposable is required. In
present occupants of Boracay Island to secure titles over their occupied keeping with the presumption of State ownership, the Court has time and
lands. again emphasized that there must be a positive act of the government,
such as an official proclamation, 80 declassifying inalienable public land
In ​Yap p​ etition, then President, the late dictator, Ferdinand Marcos issued into disposable land for agricultural or other purposes. In fact, Section 8
Proclamation No. 1801, which declared Boracay Islands, among others as of CA No. 141 limits alienable or disposable lands only to those lands
tourist zones and marine reserves. Philippine Tourism Authority (PTA) which have been "officially delimited and classified."
Circular 3-82 was issued to implement the said Proclamation.
Respondent-claimants Mayor Jose Yap, Jr., et.al filed a petition for The burden of proof in overcoming the presumption of State ownership
declaratory relief with the Regional Trial Court in Kalibo, Aklan. They of the lands of the public domain is on the person applying for
contended that the Proclamation and Circular precluded them from filing registration (or claiming ownership), who must prove that the land
an application for judicial confirmation of imperfect title or survey of subject of the application is alienable or disposable. To overcome this
land for titling purposes. presumption, incontrovertible evidence must be established that the land
During the pendency of the Yap petition, on May 22 2006, President subject of the application (or claim) is alienable or disposable. There
Gloria Macapagal-Arroyo issued Proclamation No. 1064, classifying must still be a positive act declaring land of the public domain as
Boracay Island into 400 hectares of reserved forest land and 628 hectares alienable and disposable. To prove that the land subject of an application
of agricultural land. Respondent-claimants Orlando Sacay, et. al filed for registration is alienable the applicant must establish the existence of a
with the Supreme Court a petition for prohibition, mandamus, and positive act of the government such as a presidential proclamation or an
nullification of the said Proclamation. They alleged that the Proclamation executive order; an administrative action; investigation reports of Bureau
infringed on their “prior vested rights” over portions of Boracay having of Lands investigators; and a legislative act or a statute. The applicant
been in continued possession of their respective lots since time may also secure a certification from the government that the land claimed
immemorial. to have been possessed for the required number of years is alienable and
The issue of this case is whether or not the claimants have a right to disposable.
secure titles over their occupied portions in Boracay. The Supreme Court
ruled that, claimants are not entitled to apply for judicial confirmation of In the case at bar, no such proclamation, executive order, administrative
imperfect title under CA No. 141. Neither do they have vested rights over action, report, statute, or certification was presented to the Court. The
the occupied lands under the said law. In keeping with the presumption records are bereft of evidence showing that, prior to 2006, the portions of
of State ownership, the Court has time and again emphasized that there Boracay occupied by private claimants were subject of a government
must be a positive act of the government, such as a presidential
o ​On April 14, 1976, the Department of Environment and
proclamation that the land is alienable and disposable. Absent such
Natural Resources (DENR) approved the National Reservation
well-nigh incontrovertible evidence, the Court cannot accept the
Survey of Boracay Island. This identified several lots as
submission that lands occupied by private claimants were already open to
occupied or claimed by named persons.
disposition before 2006. Matters of land classification or reclassification
cannot be assumed. They call for proof.
o ​Then President, the late dictator, Ferdinand Marcos issued
Proclamation No. 1801, ​which declared Boracay Islands,
While they may not be eligible to apply for judicial confirmation of
among others as tourist zones and marine reserves. Philippine
imperfect title under Section 48 (b) of CA No. 141, as amended, this does
Tourism Authority (PTA) Circular 3-82 was issued to
not denote their automatic ouster from the residential, commercial, and
implement the said Proclamation.
other areas they possess now classified as agricultural. Neither will this
mean the loss of their substantial investments on their occupied alienable
o ​Respondent-claimants Mayor Jose Yap, Jr., et.al filed a
lands. Lack of title does not necessarily mean lack of right to possess. For
petition for declaratory relief with the Regional Trial Court in
one thing, those with lawful possession may claim good faith as builders
Kalibo, Aklan. They contended that:
of improvements. They can take steps to preserve or protect their
possession. For another, they may look into other modes of applying for
§ ​The Proclamation and Circular precluded them from
original registration of title, such as by homestead 131 or sales patent,
filing an application for judicial confirmation of imperfect
132 subject to the conditions imposed by law.
title or survey of land for titling purposes.
More realistically, Congress may enact a law to entitle private claimants
§ ​They themselves, or through their
to acquire title to their occupied lots or to exempt them from certain
predecessors-in-interest, have been in ​open, continuous,
requirements under the present land laws. There is one such bill 133 now
exclusive, notorious possession and occupation of
pending in the House of Representatives. Whether that bill or a similar
Boracay since 12 June, 1945 or since time immemorial.
bill will become a law is for Congress to decide..
Doctrine: All lands not otherwise appearing to be clearly within private
§ ​The Proclamation and Circulation did not place
ownership are ​presumed t​ o belong to the State. It determines if lands of
Boracay ​beyond the commerce of man and as a tourist
public domain will be disposed of for private ownership. A positive act
zone, it was susceptible of private ownership. In fact,
declaring land as alienable and disposable is required.
under Sec. 48 (b) of Commonwealth Act No. 141 or
otherwise known as the Public Land Act, they had the
right to have the lots registered in their names through
FACTS: judicial confirmation of imperfect itles.

● At the core of this case are two consolidated cases (herein, “Yap o ​The Office of the Solicitor-General (OSG), on the other
petition” and “Sacay petition”) challenging ​the right of the present hand, argued that Boracay was an ​unclassified land o​ f the
occupants Boracay Island to secure titles over their occupied public domain. It formed part of the mass lands classified as
lands. “public forests”, which was not available for disposition under
● Yap petition Sec. 3 (a) of Presidential Decree No. 705, or the Revised
Forestry Code. Since Boracay Islands had not been classified
as alienable and disposable, whatever possession they had ● Court ordered the consolidation of the two petition as they
cannot ripen into ownership. involved the same issues on the land classification of Boracay
Island.
o ​After submitting the case for the resolution upon
submission of memoranda, Yap, et.al were granted a favorable RATIO:
decision in the RTC, which upheld their right to have their
occupied lands titled in their name. OSG moved for 1. ​Except for lands already with existing titles, Boracay has ​never
reconsideration, but was denied. On appeal to the Court of been expressly and administratively classified before Proclamation No.
Appeals, CA affirmed the RTC decision. Hence, the present 1​064. It was an unclassified land of public domain. Such unclassified
petition. lands are considered public forest under PD 705.

● Sacay petition 2. ​Regalian Doctrine dictates that all lands of the public dominion
belongs to the State, and that the State is the source of any asserted
o ​During the pendency of the Yap petition, on May 22 2006, right to ownership of land and charged with the conservation of such
President Gloria Macapagal-Arroyo issued Proclamation No. patrimony.
1064, ​classifying Boracay Island into 400 hectares of reserved
forest land and 628 hectares of agricultural land. 3. All lands not otherwise appearing to be clearly within private
ownership are ​presumed t​ o belong to the State. It determines if lands of
o ​Respondent-claimants Orlando Sacay, et. al filed with the public domain will be disposed of for private ownership.
Supreme Court a petition for prohibition, mandamus, and
nullification of the said Proclamation They alleged that: 4. The Court held that, a positive act declaring land as alienable and
disposable is required. In keeping with the presumption of State
§ ​The Proclamation infringed on their “prior vested ownership, the Court has time and again emphasized that there must be
rights” over portions of Boracay having been in continued a positive act of the government, such as a presidential proclamation or
possession of their respective lots since time immemorial. an executive order; an administrative action; investigation reports of
Bureau of Lands investigators; and a legislative act or a statute.
§ ​They invested billions in developing their lands and
building international renowned first class resorts. 5. The applicant may also secure a certification from the government
that the land claimed to have been possessed for the required number of
§ ​There is no need for the proclamation reclassifying it to years is alienable and disposable. The burden of proof in overcoming
an agricultural land because it is ​deemed ​agricultural such presumption is on the person applying for registration (or claiming
under Philippine Bill of 1902 and Act No. 926. ownership), who must prove that the land subject of the application is
alienable or disposable.
o ​OSG, on the contrary, argued that they do not have a vested
right as Boracay is an unclassified public forest land and as 6. No such proclamation, executive order, administrative action,
such, portions are inalienable and cannot be subject of judicial report, statute, or certification was presented to the Court. The records
confirmation of imperfect title. are bereft of evidence showing that, prior to 2006, the portions of
Boracay occupied by private claimants were subject of a government
proclamation that the land is alienable and disposable. Matters of land 2. the classification of the land as alienable and disposable land of
classification or reclassification cannot be assumed. They call for proof. the public domain.

7. Ankron v. Government of the Philippines and ​de Aldecoa v. Insular 11. ​Claimants’ bid for judicial confirmation of imperfect title, relying
Government d​ id not make the whole of Boracay Island, or portions of on the Philippine Bill of 1902, Act No. 926, and Proclamation No. 1801,
it, agricultural. must fail because of the absence of the second element of alienable and
disposable land. Their entitlement to a government grant under our present
o These cases were decided under the provisions of Public Land Act presupposes that the land possessed and applied for is
Philippine Bill of 1902 and Act No. 926. already alienable and disposable. This is clear from the wording of the law
itself. ​Where the land is not alienable and disposable, possession of the
o In these cases, the Supreme Court ruled that, “in the land, no matter how long, cannot confer ownership or possessory rights.
absence of evidence to the contrary, that in each case the lands are
agricultural lands ​until the contrary is shown​.” ​BUT, the Court 12. ​Neither may claimants apply for judicial confirmation of
clarified the matter in the present case, stating that the Philippine imperfect title under Proclamation No. 1064, with respect to those lands
Bill of 1902 and Act No. 926 merely provided for the manner which were classified as agricultural lands. ​Private claimants failed to
through which land registration courts would classify lands of the prove the first element of open, continuous, exclusive, and notorious
public domain. During this time, the President had no power to possession of their lands in Boracay since June 12, 1945.
classify lands of the public domain. Further, the assumption is
absolute and in the end, land classification is dependent on proof. WHEREFORE, judgment is rendered as follows:

8. Proclamation No. 1801 cannot be deemed the positive act needed 1. The petition for certiorari in G.R. No. 167707 is GRANTED and the
to classify Boracay Island as alienable and disposable land. If Court of Appeals Decision in CA-G.R. CV No. 71118 REVERSED AND
President Marcos intended to classify the island as alienable and SET ASIDE.
disposable or forest, or both, he would have identified the specific
limits of each, as President Arroyo did in Proclamation No. 1064. This 2. The petition for certiorari in G.R. No. 173775 is DISMISSED for lack
was not done in Proclamation No. 1801. of merit.

9. Claimants then are not entitled to apply for judicial confirmation of


imperfect title under CA No. 141. ​Neither do they have vested rights
over the occupied lands under the said law.

10. There are two requisites for judicial confirmation of imperfect or


incomplete title under CA No. 141, namely:

1. open, continuous, exclusive, and notorious possession and


occupation of the subject land by himself or through his
predecessors-in-interest under a bona fide claim of ownership
since time immemorial or from June 12, 1945; and
45. HEIRS OF MARIO MALABANAN v. REPUBLIC
the Property Registration Decree. Likewise, the land continues to be
April 29, 2009 | J. Tinga | Land of Public Domain: When Alienable and
ineligible for land registration under Section 14(2) of the Property
Disposable
Registration Decree
F. QUIJANO & J. SALAMAT
DOCTRINE: ​Section 48(b) of the Public Land Act used the words
PETITIONER: ​Heirs of Mario Malabanan "lands of the public domain" or "alienable and disposable lands of the
RESPONDENT: ​Republic of the Philippines public domain" to clearly signify that lands otherwise classified, i.e.,
mineral, forest or timber, or national parks, and lands of patrimonial or
RECIT-READY: ​Mario Malabanan, who had purchased the property private ownership, are outside the coverage of the Public Land Act. What
from Eduardo Velazco, filed an application for land registration covering the law does not include, it excludes. The use of the descriptive phrase
the property in the Regional Trial Court (RTC) in Tagaytay City, Cavite, "alienable and disposable" further limits the coverage of Section 48(b) to
claiming that the property formed part of the alienable and disposable only the agricultural lands of the public domain as set forth in Article
land of the public domain, and that he and his predecessors-in-interest XII, Section 2 of the 1987 Constitution. Bearing in mind such limitations
had been in open, continuous, uninterrupted, public and adverse under the Public Land Act, the applicant must satisfy the following
possession and occupation of the land for more than 30 years, thereby requirements in order for his application to come under Section 14(1) of
entitling him to the judicial confirmation of his title. The RTC rendered the Property Registration Decree,​ ​to wit:
judgment granting Malabanan’s application for land registration. The CA 1. The applicant, by himself or through his
promulgated its decision reversing the RTC and dismissing the predecessor-in-interest, has been in possession and
application for registration of Malabanan. The CA declared that under occupation of the property subject of the application;
Section 14(1) of the Property Registration Decree, any period of 2. The possession and occupation must be open,
possession prior to the classification of the land as alienable and continuous, exclusive, and notorious;
disposable was inconsequential and should be excluded from the 3. The possession and occupation must be under a bona
computation of the period of possession. fide claim of acquisition of ownership;
4. The possession and occupation must have taken
ISSUE: place since June 12, 1945, or earlier; and
● Whether or not petitioners were able to prove that the property 5. The property subject of the application must be an
was an alienable and disposable land of the public domain? - agricultural land of the public domain.
NO
FACTS:
RULING: ​The petitioners failed to present sufficient evidence to
● On February 20, 1998, applicant Mario Malabanan, who had
establish that they and their predecessors-in-interest had been in
purchased the property from Eduardo Velazco, filed an application
possession of the land since June 12, 1945. Without satisfying the
for land registration covering the property in the Regional Trial
requisite character and period of possession - possession and occupation
Court (RTC) in Tagaytay City, Cavite, claiming that the property
that is open, continuous, exclusive, and notorious since June 12, 1945, or
formed part of the alienable and disposable land of the public
earlier - the land cannot be considered ipso jure converted to private
domain, and that he and his predecessors-in-interest had been in
property even upon the subsequent declaration of it as alienable and
open, continuous, uninterrupted, public and adverse possession and
disposable. Prescription never began to run against the State, such that
occupation of the land for more than 30 years, thereby entitling
the land has remained ineligible for registration under Section 14(1) of
him to the judicial confirmation of his title.
● To prove that the property was an alienable and disposable land of ● Section 48(b) of the Public Land Act used the words "lands of the
the public domain, Malabanan presented during trial a certification public domain" or "alienable and disposable lands of the public
issued by the Community Environment and Natural Resources domain" to clearly signify that lands otherwise classified, i.e.,
Office (CENRO) of the Department of Environment and Natural mineral, forest or timber, or national parks, and lands of
Resources (DENR) patrimonial or private ownership, are outside the coverage of the
● The RTC rendered judgment granting Malabanan’s application for Public Land Act. What the law does not include, it excludes. The
land registration. use of the descriptive phrase "alienable and disposable" further
● The Office of the Solicitor General (OSG) appealed the judgment limits the coverage of Section 48(b) to only the agricultural lands
to the CA, arguing that Malabanan had failed to prove that the of the public domain as set forth in Article XII, Section 2 of the
property belonged to the alienable and disposable land of the 1987 Constitution. Bearing in mind such limitations under the
public domain Public Land Act, the applicant must satisfy the following
● The CA promulgated its decision reversing the RTC and requirements in order for his application to come under Section
dismissing the application for registration of Malabanan. The CA 14(1) of the Property Registration Decree,​ ​to wit:
declared that under Section 14(1) of the Property Registration
Decree, any period of possession prior to the classification of the 1. The applicant, by himself or through his
land as alienable and disposable was inconsequential and should be predecessor-in-interest, has been in possession and
excluded from the computation of the period of possession. Noting occupation of the property subject of the application;
that the CENRO-DENR certification stated that the property had 2. The possession and occupation must be open,
been declared alienable and disposable only on March 15, 1982, continuous, exclusive, and notorious;
Velazco’s possession prior to March 15, 1982 could not be tacked 3. The possession and occupation must be under a bona
for purposes of computing Malabanan’s period of possession. fide claim of acquisition of ownership;
4. The possession and occupation must have taken place
ISSUES: since June 12, 1945, or earlier; and
● Whether or not petitioners were able to prove that the property was 5. The property subject of the application must be an
an alienable and disposable land of the public domain? - ​NO agricultural land of the public domain.

RATIO: ● An examination of Section 48(b) of the Public Land Act indicates


● Alienable and disposable lands of the State fall into two categories, that Congress prescribed no requirement that the land subject of
to wit: (a) patrimonial lands of the State, or those classified as the registration should have been classified as agricultural since
lands of private ownership under Article 425 of the Civil Code, June 12, 1945, or earlier. As such, the applicant’s imperfect or
without limitation; and (b) lands of the public domain, or the incomplete title is derived only from possession and occupation
public lands as provided by the Constitution, but with the since June 12, 1945, or earlier. This means that the character of the
limitation that the lands must only be agricultural. Consequently, property subject of the application as alienable and disposable
lands classified as forest or timber, mineral, or national parks are agricultural land of the public domain determines its eligibility for
not susceptible of alienation or disposition unless they are land registration, not the ownership or title over it.
reclassified as agricultural. A positive act of the Government is ● Alienable public land held by a possessor, either personally or
necessary to enable such reclassification, ​and the exclusive through his predecessors-in-interest, openly, continuously and
prerogative to classify public lands under existing laws is vested in exclusively during the prescribed statutory period is converted to
the Executive Department, not in the courts. private property by the mere lapse or completion of the period. In
fact, by virtue of this doctrine, corporations may now acquire lands
of the public domain for as long as the lands were already
converted to private ownership, by operation of law, as a result of
satisfying the requisite period of possession prescribed by the
Public Land Act.
● The declaration that land is alienable and disposable also serves to
determine the point at which prescription may run against the
State. The imperfect or incomplete title being confirmed under
Section 48(b) of the Public Land Act is title that is acquired by
reason of the applicant’s possession and occupation of the
alienable and disposable agricultural land of the public domain.
Where all the necessary requirements for a grant by the
Government are complied with through actual physical, open,
continuous, exclusive and public possession of an alienable and
disposable land of the public domain, the possessor is deemed to
have acquired by operation of law not only a right to a grant, but a
grant by the Government, because it is not necessary that a
certificate of title be issued in order that such a grant be sanctioned
by the courts.
● The petitioners failed to present sufficient evidence to establish
that they and their predecessors-in-interest had been in possession
of the land since June 12, 1945. Without satisfying the requisite
character and period of possession - possession and occupation that
is open, continuous, exclusive, and notorious since June 12, 1945,
or earlier - the land cannot be considered ipso jure converted to
private property even upon the subsequent declaration of it as
alienable and disposable. Prescription never began to run against
the State, such that the land has remained ineligible for registration
under Section 14(1) of the Property Registration Decree. Likewise,
the land continues to be ineligible for land registration under
Section 14(2) of the Property Registration Decree unless Congress
enacts a law or the President issues a proclamation declaring the
land as no longer intended for public service or for the
development of the national wealth.

WHEREFORE, ​the Court DENIES the petitioners' Motion for


Reconsideration and the respondent's Partial Motion for
Reconsideration for their lack of merit.
46. REPUBLIC v. LEE TSAI
possession and occupation for 30 years or more is not sufcient.
June 22, 2009 | CARPIO, J.| Land of Public Domain: When Alienable Therefore, since the effectivity of PD 1073 on 25 January 1977, it must
and Disposable now be shown that possession and occupation of the piece of land by the
SALAMAT, J. & SORIANO, P. applicant, by himself or through his predecessors-in-interest, started on
12 June 1945 or earlier. This provision is in total conformity with Section
14(1) of PD 1529. There are three requisites for the ling of an
PETITIONER: ​Republic of the Philippines application for registration of title under Section 14(1) of PD 1529: (1)
RESPONDENT: ​Ruby Lee Tsai that the property in question is ​alienable and disposable land of the
public domain;​ (2) that the applicant by himself or through his
RECIT-READY: ​Ruby Lee Tsai filed an application for confirmation predecessors-in-interest ​have been in open, continuous, exclusive and
and registration of her property under PD No. 1529. She claimed that she notorious possession and occupation​; and (3) that such possession is
bought the property back in 1993 from Carungcong and that she and her under a ​bona de claim of ownership since 12 June 1945 or earlier .
predecessors-in-interest have been in open, continuous, exclusive, and We agree with the Republic that respondent’s evidence was not enough
notorious possession and occupation of the property for more than 30 to prove that her possession of the subject property started since 12 June
years. For proof she submitted among others Tax Declarations (earliest of 1945 or ​earlier because Lee Tsai’s earliest evidence can be traced
which was dated 1948 under the name of Carungcong). The Republic back to a tax declaration issued in the name of her
opposed saying that Lee Tsai failed to present sufficient evidence that she predecessors-in-interest only in the year 1948.
has been in open, continuous, exclusive and notorious possession and
occupation of the subject property since 12 June 1945. Republic also We note that Lee Tsai also failed to prove that the subject property has
countered that tax declarations and tax receipts do not constitute
been declared alienable and disposable by the President or the Secretary
sufficient evidence of bona fide acquisition. The RTC ruled in favor of
Lee Tsai. The CA affirmed the RTC, and added that Lee Tsai need not of the Department of Environment and Natural Resources.
prove that she and her predecessors-in-interest have been in possession of
the subject property since 12 June 1945 or earlier because Section 48(b) DOCTRINE: ​[T]he applicant for land registration must prove that the
of CA 141 was already superseded by Republic Act No. 1942 (RA 1942), DENR Secretary had approved the land classication and released the
which provides for a simple 30 year prescriptive period of occupation by land of the public domain as alienable and disposable, and that ​the land
an applicant for judicial conrmation of title. subject of the application for registration falls within the approved
area per verication through survey by the PENRO or CENRO. In
ISSUE: addition, the applicant for land registration must present a copy of
● Whether the trial court can grant the application for registration the original classication approved by the DENR Secretary and
despite the lack of proof of Lee Tsai’s open, continuous, certied as a true copy by the legal custodian of the ofcial records.
These facts must be established to prove that the land is alienable
exclusive and notorious possession of the subject property since
and disposable.
12 June 1945 or earlier - NO (Lee Tsai did not satisfy requisites
under PD 1529)
FACTS:
RULING: ​The Court of Appeals have an erroneous interpretation of ● On 3 December 1996, Lee Tsai led an application for the
Section 48(b) of CA 141. As the law now stands, a mere showing of conrmation and registration of the subject property under
Presidential Decree No. 1529. She alleged that she is the owner of
the subject property and the improvements thereon. She stated that ● On 21 September 1998, the trial court granted Lee Tsai’s
on 31 May 1993, she purchased the subject property from Manolita application for registration. The Republic appealed to the Court of
Gonzales Vda. De Carungcong (Carungcong), through Wendy Appeals on the ground that the trial court erred in granting the
Mitsuko Sato, Carungcong’s daughter and attorney in fact. She application for registration despite Lee Tsai’s failure to prove
also declared that she and her predecessors-in-interest have been in open, continuous, exclusive and notorious possession of the subject
open, continuous, exclusive and notorious possession and property since 12 June 1945 or earlier. According to the Republic,
occupation of the subject property for more than 30 years it is not sufcient that respondent proved possession of the subject
● Except for the Republic, there were no other oppositors to the property for more than 30 years. The Court of Appeals afrmed the
application. The Republic opposed Lee Tsai’s application on the trial court’s decision. According to the Court of Appeals, Lee Tsai
following grounds: need not prove that she and her predecessors-in-interest have been
○ (1) that Lee Tsai and her predecessors-in-interest failed to in possession of the subject property since 12 June 1945 or earlier
present sufcient evidence to show that they have been in because Section 48(b) of CA 141 was already superseded by
open, continuous, exclusive and notorious possession and Republic Act No. 1942 (RA 1942), which provides for a simple 30
occupation of the subject property since 12 June 1945 or year prescriptive period of occupation by an applicant for judicial
earlier as required by Section 48(b) of Commonwealth conrmation of title.
Act No. 141 (CA 141), as amended by Presidential
Decree No. 1073 ISSUES:
○ (2) that the tax declarations and tax receipt payments ● Whether the trial court can grant the application for registration
attached to the application do not constitute competent despite the lack of proof of Lee Tsai’s open, continuous, exclusive
and sufcient evidence of a bona de acquisition of the and notorious possession of the subject property since 12 June
land applied for or of Lee Tsai’s open, continuous, 1945 or earlier - NO
exclusive and notorious possession and occupation of the
subject property in the concept of an owner since 12 June RELEVANT ARGUMENTS (if any):
1945 or earlier; and ● Petitioner: Lee Tsai only proved possession since 1948, which is in
○ (3) that the subject property forms part of the public violation of Section 48(b) of CA 141, as amended by PD 1073
domain and is not subject to private appropriation ● Respondent: insists that it is sufcient that she proved that she and
● Lee Tsai presented the following documents to support her her predecessors-in-interest have been in open, continuous,
application: Deed of Absolute Sale (Yr 1993); Tax Declarations exclusive and notorious possession and occupation of the subject
(Yrs (under the name of Carungcong) 1948, 1965, 1960, 1974, property under a bona de claim of ownership for more than 30
1980, 1992); Tax declaration (Yr 1994 under the name of Lee years
Tsai); Official Receipts of real property taxes under the name of
Carungcong (Yr 1991-1993); Official Receipts of Real Property RATIO:
Taxes under the name of Lee Tsai Yrs 1994-1995) ● The Court notes that in Lee Tsai’s original application before the
trial court, she claimed that she was entitled to the conrmation
and registration of her title to the subject property under PD 1529. since July 26, 1894. This was superseded by R.A. No.
However, Lee Tsai did not specify under what paragraph of 1942, which provided for a simple thirty-year prescriptive
Section 14 of PD 1529 she was ling the application. But going period of occupation by an applicant for judicial
over Lee Tsai’s application and the evidence she presented before conrmation of imperfect title. The same, however, has
the trial court, it appears that Lee Tsai led her application under already been amended by Presidential Decree No. 1073
Section 14(1) of PD 1529: ● As the law now stands, a mere showing of possession and
○ “SEC. 14. Who may apply: (1) Those who by themselves occupation for 30 years or more is not sufcient. Therefore, since
or through their predecessors-in-interest have been in the effectivity of PD 1073 on 25 January 1977, it must now be
open, continuous, exclusive and notorious possession and shown that possession and occupation of the piece of land by the
occupation of alienable and disposable lands of the public applicant, by himself or through his predecessors-in-interest,
domain under a bona de claim of ownership since June started on 12 June 1945 or earlier. This provision is in total
12, 1945, or earlier conformity with Section 14(1) of PD 1529
● Thus, there are three requisites for the ling of an application for ● Lee Tsai failed to comply with the period of possession and
registration of title under Section 14(1) of PD 1529: occupation of the subject property, as required by both PD 1529
○ (1) that the property in question is alienable and and CA 141. We agree with the Republic that respondent’s
disposable land of the public domain; evidence was not enough to prove that her possession of the
○ (2) that the applicant by himself or through his subject property started since 12 June 1945 or ​earlier because Lee
predecessors-in-interest have been in open, continuous, Tsai’s earliest evidence can be traced back to a tax declaration
exclusive and notorious possession and occupation; and issued in the name of her predecessors-in-interest only in the
○ (3) that such possession is under a bona de claim of year 1948. In view of the lack of sufcient showing that
ownership since 12 June 1945 or earlier respondent and her predecessors-in-interest possessed the subject
● A similar right is given under Section 48(b) of CA 141, as property under a bona de claim of ownership since 12 June 1945
amended by PD 1073. According to the Court of Appeals, or earlier, respondent’s application for conrmation and
respondent need not prove possession of the subject property since registration of the subject property under PD 1529 and CA 141
12 June 1945 or earlier because Section 48(b) of CA 141 was should be denied
amended by RA 1942, which provided for a simple 30-year ● Finally, we note that respondent also failed to prove that the
prescriptive period. The Court of Appeals appears to have an subject property has been declared alienable and disposable by the
erroneous interpretation of Section 48(b) of CA 141. President or the Secretary of the Department of Environment and
● Through the years, Section 48(b) of the CA 141 has been amended Natural Resources. In Republic v. T.A.N. Properties, Inc:
several times. 21 The Court of Appeals failed to consider the ○ “[T]he applicant for land registration must prove that the
amendment introduced by PD 1073. In Republic v. Doldol, 22 the DENR Secretary had approved the land classication and
Court provided a summary of these amendments: released the land of the public domain as alienable and
○ “The original Section 48(b) of C.A. No.141 provided for disposable, and that ​the land subject of the application
possession and occupation of lands of the public domain for registration falls within the approved area per
verication through survey by the PENRO or
CENRO. In addition, the applicant for land
registration must present a copy of the original
classication approved by the DENR Secretary and
certied as a true copy by the legal custodian of the
ofcial records. These facts must be established to
prove that the land is alienable and disposable.

WHEREFORE, we GRANT the petition. We SET ASIDE the 30


January 2004 Decision of the Court of Appeals in CA G.R. CV No.
70006 and the 21 September 1998 Decision of the Regional Trial Court
of Tagaytay City, Branch 18, in LRC Case No. TG-788. We DENY
respondent Ruby Lee Tsai’s application for conrmation and
registration of Lot No. 7062 described in plan Ap-04-010084, Cad355,
Tagaytay Cadastre. SO ORDERED
47. DIAZ v. REPUBLIC FACTS:
2 Feb. 2010 | J.Corona | Land of public domain ● Diaz’s late mother, Flora Garcia (Garcia), filed an application for
P. SORIANO & COMPANY registration of a vast tract of land She alleged that she possessed
the land as owner and worked, developed and harvested the
agricultural products and benefits of the same continuously,
PETITIONER: ​Florencia Diaz
publicly and adversely for more or less 26 years.
RESPONDENT: ​Republic of the Philippines
● The Republic of the Philippines, represented by the OSG opposed
the application because the land in question was within the Fort
RECIT-READY: ​Garcia filed an application to register a parcel of land
Magsaysay Military Reservation (FMMR), established by virtue of
which she had alleged owned for 27 years.. The Republic opposed this on
Proclamation No. 237 (Proclamation 237) 3 in 1955. Thus, it was
the ground that the land was within a military reservation. In another
inalienable as it formed part of the public domain.
case, ​Director of Lands v. Reyes,​ the court ruled that someone else’s land
● Significantly, this Court already ruled in ​Director of Lands v.
which was likewise within the reservation was inalienable. Nonetheless,
Reyes that the property subject of Garcia's application was
the court granted the application. Republic appealed in CA arguing that
inalienable as it formed part of a military reservation. Moreover,
the land has been proclaimed to be inalienable. Garcia died, daughter
the existence of Possessory Information Title No. 216 (allegedly
Diaz substituted her. CA reversed its decision. Diaz appealed on the
registered in the name of a certain Melecio Padilla on March 5,
ground that she has “private rights” over the property.
1895), on which therein respondent Parañaque Investment and
Development Corporation anchored its claim on the land, was not
ISSUE: ​[None explicitly stated]
proven. Accordingly, the decree of registration issued in its favor
● W/N Diaz has private rights to the property. -NO
was declared null and void.
● W/N the Compromise Agreement between Diaz and OSG is
● Reyes n​ otwithstanding, the CFI ruled in Garcia's favor.
valid. -NO
● The Republic eventually appealed the decision of the CFI to the
CA. In its decision penned by Justice Vicente V.
RULING: ​In this case, Diaz was not able to produce such evidence.
Mendoza(Mendoza decision), the appellate court reversed and set
Accordingly, her occupation thereof, and that of her
aside the decision of the CFI. The CA found that Reyes was
predecessors-in-interest, could not have ripened into ownership of the
applicable to Diaz’'s case as it involved the same property. The CA
subject land. This is because prior to the conversion of forest land as
observed that Garcia also traced her ownership of the land in
alienable land, any occupation or possession thereof cannot be counted in
question to Possessory Information Title No. 216. As Garcia's right
reckoning compliance with the thirty-year possession requirement under
to the property was largely dependent on the existence and validity
the Public Land Act.
of the possessory information title the probative value of which
had already been passed upon by this Court in Reyes, and
DOCTRINE: ​Forest lands are not registrable under the Public Land Act.
inasmuch as the land was situated inside a military reservation, the
However, forest lands may be registered when they have been
CA concluded that she did not validly acquire title thereto.
reclassified as alienable by the President in a clear and categorical
● During the pendency of the case in the CA, Garcia passed away
manner (upon the recommendation of the proper department head who
and was substituted by her heirs, one of whom was petitioner
has the authority to classify the lands of the public domain into alienable
Florencia G. Diaz.
or disposable, timber and mineral lands) coupled with possession by the
● the CA encouraged the parties to reach an amicable settlement on
claimant as well as that of her predecessors-in- interest.
the matter and even gave the parties sufficient time to draft and
finalize the same. The parties ultimately entered into a compromise
agreement with the Republic withdrawing its claim on the more or clear and categorical manner (upon the recommendation of the
less 4,689 hectares supposedly outside the FMMR. For her part, proper department head who has the authority to classify the
Diaz withdrew her application for the portion of the property inside lands of the public domain into alienable or disposable, timber
the military reservation. They filed a motion for approval of the and mineral lands) coupled with possession by the claimant as
amicable settlement in the CA. CA approved the compromise well as that of her predecessors-in- interest.
agreement. CA directed the Land Registration Administration to ● Unfortunately for Diaz, she was not able to produce such
issue the corresponding decree of registration in petitioner's favor evidence. Accordingly, her occupation thereof, and that of her
● OSG filed a motion for reconsideration of the CA resolution. The predecessors-in-interest, could not have ripened into
OSG informed CA that the tract of land subject of the amicable ownership of the subject land. This is because prior to the
settlement was still within the military reservation. conversion of forest land as alienable land, any occupation or
● CA issued an amended resolution (amended resolution) annulling possession thereof cannot be counted in reckoning compliance
the compromise agreement entered into between the parties with the thirty-year possession requirement under the Public
● Diaz .moved for reconsideration. Diaz filed a petition for review Land Act.
on certiorari. Diaz insisted on filing a motion to lift entry of ● The rules on the confirmation of imperfect titles do not apply
judgment and motion for leave to file a second motion for unless and until the land classified as forest land is released
reconsideration and to refer the case to theSupreme Court en banc. through an official proclamation to that effect. Then and only then
Diaz wrote identical letters, first addressed to Justice Leonardo A. will it form part of the disposable agricultural lands of the public
Quisumbing (then Acting Chief Justice) and then to Chief Justice domain.
Reynato S. Puno himself. ● Coming now to Diaz's contention that her "private rights" to the
property, meaning her and her predecessors' possession thereof
ISSUES: [​None explicitly stated​] prior to the establishment of the FMMR, must be respected, the
● W/N Diaz has private rights to the property. ​-NO same is untenable. As earlier stated, we had already recognized the
● W/N the Compromise Agreement between Diaz and OSG is valid​. same land to be public forest even before the FMMR was
-NO established.

RELEVANT ARGUMENTS (if any): To reiterate:


● Petitioner:
● Respondent: Before the military reservation was established, the evidence is
inconclusive as to possession, for it is shown by the evidence that
RATIO: the land involved is largely mountainous and forested. As a matter
of fact, at the time of the hearing, it was conceded that
Diaz does not have “private rights” to the property approximately 13,957 hectares of said land consist of public forest.
● Forest lands are not registrable under the Public Land Act. It
is well-settled that forest land is incapable of registration; and ● Therefore, even if possession was for more than 30 years, it could
its inclusion in a title, whether such title be one issued using the never ripen to ownership.
Spanish sovereignty or under the present Torrens system of ● But even assuming that the land in question was alienable land
registration, nullifies the title. before it was established as a military reservation, there was
● However, it is true that forest lands may be registered when nevertheless still a dearth of evidence with respect to its occupation
they have been reclassified as alienable by the President in a
by petitioner and her predecessors-in-interest for more than 30 WHEREFORE, the letter-motion dated January 26, 2009 of petitioner
years. is NOTED and is hereby treated as a third motion for reconsideration.
● Furthermore, the fact that the possessory information title on which The motion is DENIED considering that a third motion for
Diaz also bases her claim of ownership was found to be inexistent reconsideration is a prohibited pleading and the plea utterly lacks
in Reyes, 39 thus rendering its probative value suspect, further merit. Petitioner is found GUILTY of contempt of court. Accordingly, a
militates against granting her application for registration. FINE of Five Thousand Pesos is hereby imposed on her, payable within
ten days from receipt of this resolution. She is hereby WARNED that
Compromise agreement is null and void. any repetition hereof shall be dealt with more severely.
● There is no contract unless the following requisites concur:
○ Consent of the contracting parties;
○ Object certain which is the subject matter of the contract;
○ Cause of the obligation which is established.
● In this case, although the OSG was authorized to appear as counsel
for the Republic, it was never given the specific or special
authority to enter into a compromise agreement with petitioner.
This is in violation of the provisions of Rule 138 Section 23, of the
Rules of Court which requires "special authority" for attorneys to
bind their clients.
● Diaz was not able to provide any proof that the consent of the
Republic, through the appropriate government agencies, i.e., the
Department of Environment and Natural Resources, Land
Management Bureau, Land Registration Authority, and the Office
of the President, was secured by the OSG when it executed the
agreement with her.
● Moreover, the land in question could not have been a valid subject
matter of a contract because, being forest land, it was inalienable.
Article 1347 of the Civil Code provides: all things which are not
outside the commerce of men, including future things, may be the
object of a contract.
● Finally, the Court finds the cause or consideration of the obligation
contrary to law and against public policy. The agreement provided
that, in consideration of Diaz's withdrawal of her application for
registration of title from that portion of the property located within
the military reservation, Republic was withdrawing its claim on
that part of the land situated outside said reservation. The Republic
could not validly enter into such undertaking as the subject matter
of the agreement was outside the commerce of man.
48. REPUBLIC v. HANOVER WHEREFORE, start here.
April 29, 2009 | J. Tinga | Judicial Confirmation of Imperfect or
Incomplete Titles; When is a land of public domain alienable and
Disposable
A. ALVERO & V. BARLONGAY

[For AINE]: Case link - ​LTD - Republic v. Hanover Worldwide


Trading Corp GR 172102

PETITIONER: ​Republic of the Philippines


RESPONDENT: ​Hanover Worldwide Trading Corporation

RECIT-READY: ​Start here.

ISSUE:
● W/N? ​YES/NO
● W/N? Y​ ES/NO

RULING: ​Start here.

DOCTRINE: ​Start here.

FACTS:
● Fact #1
● Fact #2

ISSUES:
● Issue #1
● Issue #2

RELEVANT ARGUMENTS (if any):


● Petitioner:
● Respondent:

RATIO:
● Ratio #1
● Ratio #2
49. REPUBLIC v. VEGA
alienable government land must be proven. Here, the Republic does not
January 17,, 2011 | J. Sereno | Land Of Public Domain: When
question the fact of occupation, but that of the alienability of the land.
Alienable And Disposable
They also contended that the testimony of the CENRO officer is
BARLONGAY & BARRALES
insufficient. It has been held in Jurisprudence that a ​CENRO certificate is
inadequate proof that the land is alienable​. ​There must also be a
PETITIONER: ​Republic of the Philippines certification from the Secretary of DENR that a lot was alienable and
RESPONDENT: ​Carlos R. Vega, Marcos R. Vega, Rogelio R. Vega, disposable​ as ruled in Republic v. Tan.
Lubin R. Vega, Heirs Of Gloria R. Vega, Namely: Fracisco L. Yap, Ma.
Winona Y. Rodriguez, Ma. Wendelyn V. Yap And Francisco V. Yap, Jr. However, in light of a recent ruling in Republic v. Serrano, the CENRO
certification is held to be substantial compliance to the needed proof. In
RECIT-READY: ​Respondents sought to register a parcel of land, the present case, the Court, has nonetheless recognized and affirmed
claiming that they inherited the same from their deceased mother. applications for land registration on other substantial and convincing
Respondent-intervenors Buhay claimed a portion of the lot in question evidence duly presented without any opposition from the LRA or the
and offered in evidence a Subdivision Plan which indicated the portion of DENR on the ground of ​substantial compliance.
the subject land, which they claimed was sold to their
predecessors-in-interest. On their part, Vegas presented as witness an Respondents here substantially complied.
officer from CENRO who testified that the land in question is indeed
alienable. ● First, respondents Vegas were able to present Mr. Gonzales of
the CENRO who testified that the subject land is alienable and
The Republic, through the Office of the Solicitor General, opposed the disposable.
claim and argued that Vegas failed to prove that the subject land was ● Second, the Subdivision Plan formally offered as evidence by
alienable and disposable, since the testimony of Mr. Gonzales did not respondents-intervenors Buhays, ​expressly indicates that the
contain the date when the land was declared as such. The RTC ruled in land is alienable and disposable.
favor of the respondents and ordered titles to be issued in favor of Vega ● Finally, upon being informed of respondents Vegas’ application
and Buhay. The Republic appealed the case to the Court of Appeals, for original registration, the LRA never raised the issue that the
which affirmed the findings of the lower court. Hence, this petition. land subject of registration was not alienable and disposable.

ISSUE: It must be emphasized that the present ruling on substantial compliance


applies pro hac vice (this time only).
● W/N based on the evidence on record, respondents Vegas have
sufficiently established that the subject land is alienable and
disposable. ​Yes ​(But this case falls under the exception, not the DOCTRINE: ​To establish that the land subject of the application is
general rule) alienable and disposable public land, the general rule remains: all
applications for original registration under the Property Registration
Decree must include both (1) a CENRO or PENRO certification and (2) a
RULING: ​The rule for registration of government land is that there must certified true copy of the original classification made by the DENR
be open, continuous, exclusive and notorious possession and occupation Secretary.
of alienable government land. The fact of occupation and that the land is
was no public land application filed for the same land by the
applicant or by any other person.
As an exception, however, the courts - in their sound discretion and based
● During the trial, respondents-intervenors Romea, Francisco,
solely on the evidence presented on record - may approve the application,
Orlando etc. all surnamed buhay (respondents-intervenors
pro hac vice, on the ground of ​substantial compliance. ​This exception
Buhays) entered their appearance and moved to intervene in
shall only apply to applications for registration currently pending before
respondents Vegas’ application for registration.
the trial court prior to this Decision and shall be inapplicable to all future
Respondents-intervenors Buhays claimed a portion of the subject
applications.
land , purportedly sold by respondents Vegas’ mother. ​They
likewise formally offered in evidence Subdivision Plan
Csd-04-024336-D, which indicated the portion of the subject land,
FACTS: which they claimed was sold to their predecessors-in-interest.
● The trial court granted respondents Vegas’ application and directed
● On 26 May 1995, respondents filed an application for the the Land Registration Authority (LRA) to issue the corresponding
registration of title. decree of registration in the name of respondents Vegas and
● The application covered a parcel of land in Los Baños, Laguna, respondents-intervenors Buhays’ predecessors, in proportion to
with a total area of 6,902 square meters their claims over the subject land.
● Respondents Vegas alleged that they inherited the subject land ● Petitioner Republic appealed the Decision of the trial court,
from their mother, Maria Revilleza Vda. de Vega, who in turn arguing that respondents Vegas failed to prove that the subject land
inherited it from her father, Lorenzo Revilleza. Their mother’s was alienable and disposable, since the testimony of Mr. Gonzales
siblings (two brothers and a sister) died intestate, all without did not contain the date when the land was declared as such.
leaving any offspring. ● The Court of Appeals affirmed in toto the earlier Decision of the
● On 21 June 1995, petitioner Republic filed an opposition to trial court. Aggrieved by the ruling, petitioner filed the instant Rule
respondents Vegas’ application for registration on the ground, inter 45 Petition with this Court.
alia, that the subject land or portions thereof were lands of the
public domain and, as such, not subject to private appropriation. ISSUES:
● During the trial court hearing on the application for registration,
respondents Vegas presented several exhibits in compliance with ● W/N based on the evidence on record, respondents Vegas have
the jurisdictional requirements, as well as witnesses to prove sufficiently established that the subject land is alienable and
respondents Vegas’ ownership, occupation and possession of the disposable. ​Yes ​(This case falls under the exception, not the
land subject of the registration. general rule)
● Significant was the testimony of Mr. Rodolfo Gonzales, a Special
Investigator of the Community Environment and Natural RATIO:
Resources Office (CENRO) of Los Baños, Laguna, under the ● Presidential Decree No. 1529, otherwise known as the Property
Department of Environment and Natural Resources (DENR). He Registration Decree, provides for the instances when a person may
attested to having conducted an inspection of the subject land and file for an application for registration of title over a parcel of land.
identified the corresponding Report dated 13 January 1997, which Applicants for registration of title must prove the following: (1)
he had submitted to the Regional Executive Director, Region IV. that the subject land forms part of the disposable and alienable
The report stated that the area subject of the investigation was lands of the public domain; and (2) that they have been in open,
entirely within the alienable and disposable zone, and that there
continuous, exclusive and notorious possession and occupation of denial of an application for registration. ​Significantly, however, the
the land under a bona fide claim of ownership since 12 June 1945 Court’s pronouncement in Republic v. T.A.N. Properties, Inc. was
or earlier. issued after the decisions of the trial court and the appellate court ​in
● Raising no issue with respect to respondents Vegas’ open, this case.
continuous, exclusive and notorious possession of the subject land ● Recently, however, in Republic v. Serrano, ​the Court affirmed the
in the present Petition, the Court will limit its focus on the first findings of the trial and appellate courts that the parcel of land
requisite: specifically, whether it has sufficiently been subject of registration was alienable and disposable. The Court
demonstrated that the subject land is alienable and disposable. held that a DENR Regional Technical Director’s certification,
● To prove that the land subject of an application for registration is which is annotated on the subdivision plan submitted in evidence,
alienable, an applicant must conclusively establish the existence of constitutes substantial compliance with the legal requirement.
a positive act of the government, such as any of the following: a ● The ​DENR certification enjoys the presumption of regularity
presidential proclamation or an executive order; other absent any evidence to the contrary. ​It bears noting that no
administrative actions; investigation reports of the Bureau of Lands opposition was filed or registered by the Land Registration
investigator; or a legislative act or statute. ​The applicant may also Authority or the DENR to contest respondents' applications on the
secure a certification from the government that the lands applied ground that their respective shares of the lot are inalienable. There
for are alienable and disposable. being no substantive rights which stand to be prejudiced, the
● Previously, a certification from the DENR that a lot was alienable benefit of the Certification may thus be equitably extended in favor
and disposable was sufficient to establish the true nature and of respondents.
character of the property and enjoyed the presumption of regularity ● Indeed, the best proofs in registration proceedings that a land is
in the absence of contradictory evidence. alienable and disposable are a certification from the CENRO or
● However, in ​Republic v. T.A.N. Properties, Inc., the Supreme Provincial Environment and Natural Resources Office (PENRO)
Court overturned the grant by the lower courts of an original and a certified true copy of the DENR’s original classification of
application for registration over a parcel of land in Batangas and the land. The Court, however, has nonetheless recognized and
ruled that a CENRO certification is not enough to certify that a affirmed applications for land registration on other substantial and
land is alienable and disposable. convincing evidence duly presented without any opposition from
● As it now stands, aside from a CENRO certification, an the LRA or the DENR on the ground of ​substantial compliance.
application for original registration of title over a parcel of land ● Applying these precedents, the Court finds that despite the absence
must be accompanied by a copy of the ​original classification of a certification by the CENRO and a certified true copy of the
approved by the DENR Secretary ​and certified as a true copy by original classification by the DENR Secretary, there has been
the legal custodian of the official records in order to establish that substantial compliance with the requirement to show that the
the land indeed is alienable and disposable. subject land is indeed alienable and disposable based on the
● To comply with the first requisite for an application for original evidence on record.
registration of title under the Property Registration Decree, ● First, respondents Vegas were able to present Mr. Gonzales of the
respondents Vegas should have submitted a CENRO certification CENRO who testified that the subject land is alienable and
and a certified true copy of the original classification by the DENR disposable, and who identified his written report on his inspection
Secretary that the land is alienable and disposable, together with of the subject land.
their application. ● Second, Subdivision Plan Csd-04-02433-6, formally offered as
● If the stringent rule imposed in Republic v. T.A.N. Properties, Inc. evidence by respondents-intervenors Buhays, ​expressly indicates
is to be followed, the absence of these twin certifications justifies a
that the land is alienable and disposable. Mr. Samson G. de Leon’s CENRO or PENRO certification and (2) a certified true copy of
(the officer-in-charge of the Office of the Assistant Regional the original classification made by the DENR Secretary.
Executive Director for Operations of the DENR) ​annotation ● As an exception, however, the courts - in their sound discretion
pertaining to the identification of the land as alienable and and based solely on the evidence presented on record - may
disposable coincides with the investigation report of Mr. Gonzales. approve the application, pro hac vice, on the ground of ​substantial
● Finally, upon being informed of respondents Vegas’ application compliance showing that there has been a positive act of
for original registration, the LRA never raised the issue that the government to show the nature and character of the land and an
land subject of registration was not alienable and disposable. ​In absence of effective opposition from the government. This
addition, not only did the government fail to cross-examine Mr. exception shall only apply to applications for registration currently
Gonzales, it likewise chose not to present any countervailing pending before the trial court prior to this Decision and shall be
evidence to support its opposition. In contrast to the other cases inapplicable to all future applications.
brought before this Court, ​no opposition was raised by any
interested government body, aside from the pro forma opposition WHEREFORE, premises considered, the instant Petition is DENIED.
filed by the OSG. The Court of Appeals’ Decision dated 30 April 2007 and the trial
● Petitioner Republic also assails the failure of Mr. Gonzales to court’s Decision dated 18 November 2003 are hereby AFFIRMED.
testify as to when the land was declared as alienable and
disposable. However, these matters could have been dealt with
extensively during cross-examination, which petitioner Republic
waived because of its repeated absences and failure to present
counter evidence. In any event, the Report, as well as the
Subdivision Plan, readily reveals that the subject land was certified
as alienable and disposable as early as 31 December 1925 and was
even classified as residential and commercial in nature.
● Thus, the Court finds that the evidence presented by respondents
Vegas, coupled with the absence of any countervailing evidence by
petitioner Republic, substantially establishes that the land applied
for is alienable and disposable and is the subject of original
registration proceedings under the Property Registration Decree.
● It must be emphasized that the present ruling on substantial
compliance applies pro hac vice (this time only). It does not in any
way detract from our rulings in Republic v. T.A.N. Properties, Inc.,
and similar cases which impose a strict requirement to prove that
the public land is alienable and disposable, especially in this case
when the Decisions of the lower court and the Court of Appeals
were rendered prior to these rulings. To establish that the land
subject of the application is alienable and disposable public land,
the general rule remains: all applications for original registration
under the Property Registration Decree must include both (1) a
50. VICENTE YU CHANG AND SOLEDAD YU CHANG v.
actually be covered with grass or planted with crops by kaingin
REPUBLIC OF THE PHILIPPINE
cultivators or other farmers. "Forest lands" do not have to be on
February 23, 2011 | Villarama, Jr., J. | Topic
mountains or in out-of-the-way places. ​The classification of land is
CHECKER & MAKER
descriptive of its legal nature or status and does not have to be descriptive
of what the land actually looks like. Unless and until the land classified
PETITIONER: ​Vicente Yu Chang and Soledad Yu Chang as forest land is released in an official proclamation to that effect so that
RESPONDENT: ​Republic of the Philippines it may form part of the disposable agricultural lands of the public
domain, the rules on confirmation of imperfect title does not apply.
RECIT-READY: ​On February 21, 1997, petitioner Soledad Yu Chang, Petitioners' possession of the subject forest land prior to the date when it
and Vicente Yu Chang, filed a petition for registration of title over the was classified as alienable and disposable is inconsequential and should
aforementioned lots under the Property Registration Decree. They be excluded from the computation of the period of possession. (The
declared that they are the co-owners of the subject lots; that they and subject lots were declared alienable and disposable only on October 30,
their predecessors-in-interest "have been in actual, physical, material, 1986.)
exclusive, open, occupation and possession of the above described
parcels of land for more than 100 years" The Republic, filed an DOCTRINE: ​A forested area classified as forest land of the public
Opposition to the application, alleging that: (1) neither the applicants nor domain does not lose such classification simply because loggers or
their predecessors-in-interest have been in open, continuous, exclusive settlers may have stripped it of its forest cover. Parcels of land classified
and notorious possession of the land since June 12, 1945 or prior thereto; as forest land may actually be covered with grass or planted with crops
(2) the muniments of title, tax declarations and tax receipts do not by ​kaingin cultivators or other farmers. "Forest lands" do not have to be
constitute competent and sufficient evidence of a bona fide acquisition of on mountains or in out-of-the-way places. The classification of land is
the land; and (3) that the parcels of land applied for are portions of the descriptive of its legal nature or status and does not have to be descriptive
public domain and are not subject to private appropriation. of what the land actually looks like. ​Unless and until the land classified
as forest land is released in an official proclamation to that effect so
ISSUE: ​W/N the subject properties could no longer be considered and that it may form part of the disposable agricultural lands of the
classified as forest land - ​NO public domain, the rules on confirmation of imperfect title do not
apply.
RULING: ​The Court Ruled that petitioners' application for registration
of title may be granted, they must first establish the following: (1) that
FACTS:
the subject land forms part of the disposable and alienable lands of the
● On March 22, 1949, petitioners' father, L. Yu Chang and the
public domain and (2) that they have been in open, continuous, exclusive
Municipality of Pili, Camarines Sur, through its then Mayor, Justo
and notorious possession and occupation of the same under a bona fide
Casuncad, executed an Agreement to Exchange Real Property
claim of ownership, since June 12, 1945, or earlier. In the instant case,
● Wherein the former assigned and transferred to the Municipality of
petitioners did not adduce any evidence to the effect that the lots subject
Pili his 400-square-meter residential lot in Barrio San Roque, Pili,
of their application are alienable and disposable land of the public
Camarines Sur, in exchange for a 400-square-meter piece of land
domain. Instead, petitioners contend that the subject properties could no
located in San Juan, Pili.
longer be considered and classified as forest land since there are building
● Thereafter, L. Yu Chang and his family took possession of the
structures, residential houses and even government buildings existing and
property thus obtained and erected a residential house and a
standing on the area. Parcels of land classified as forest land may
gasoline station thereon. (it was still a forest land during this time)
● When L. Yu Chang died on September 30, 1976, his wife, Donata existing and standing on the land. In their Memorandum,
Sta. Ana and his seven children inherited the property and petitioners point out that the original owner and possessor of the
succeeded in the possession of the property. subject land was the Municipal Government of Pili which was
● On March 1, 1978, a Deed of Transfer and Renunciation of their established in 1930. The land was originally part of the
rights over the property was executed by L. Yu Chang's five municipal ground adjacent to the Municipal Building located
children, Rafaela, Catalina, Flaviana, Esperanza, and Antonio, in at the right side of the Naga-Legaspi National Highway​. From
favor of herein petitioners. 1949, when L. Yu Chang acquired the property through barter and
● On February 21, 1997, petitioner Soledad Yu Chang, for herself up to the filing of petitioners' application in 1997, petitioners and
and in representation of her brother and co-petitioner, Vicente Yu their predecessors-in-interest had been in actual physical and
Chang, filed a petition for registration of title over the material possession of the land in the concept of an owner,
aforementioned lots under the Property Registration Decree. notorious and known to the public and adverse to the whole world.
● In their petition, they declared that they are the co-owners of the ● Respondent: The Republic, through the OSG, for its part,
subject lots; that they and their predecessors-in-interest "have been maintains that petitioners failed to prove their open, continuous,
in actual, physical, material, exclusive, open, occupation and exclusive and notorious possession of the subject lots for the
possession of the above described parcels of land for more than period of time required by law. ​The OSG also submits that the
100 years"; and that allegedly, they have continuously, peacefully, subject lands were declared as alienable and disposable only on
and adversely possessed the property in the concept of owners. October 30, 1986.
● Hence, they are entitled to confirmation of ownership and issuance
and registration of title in their names. RATIO:
● The Republic, through the Office of the Solicitor General (OSG), ● Section 48(b) of the Public Land Act, as amended by P.D. 1073,
filed an Opposition to the application, alleging, inter alia, that: (1) under which petitioners' application was filed, provides:
neither the applicants nor their predecessors-in-interest have been ● SEC. 48. The following described citizens of the Philippines,
in open, continuous, exclusive and notorious possession of the land occupying lands of the public domain or claiming to own any such
since June 12, 1945 or prior thereto; (2) the muniments of title, tax lands or an interest therein, but whose titles have not been
declarations and tax receipts do not constitute competent and perfected or completed, may apply to the Regional Trial Court of
sufficient evidence of a bona fide acquisition of the land; and (3) the province or city where the land is located for confirmation of
that the parcels of land applied for are portions of the public their claims and the issuance of a certificate of title therefor, under
domain and are not subject to private appropriation. the Property Registration Decree, to wit:
xxxx
ISSUES: (b) Those who by themselves or through their
● W/N the subject properties could no longer be considered and predecessors-in-interest have been in the open, continuous,
classified as forest land since there are building structures, exclusive, and notorious possession and occupation of alienable
residential houses and even government buildings existing and and disposable agricultural lands of the public domain, under a
standing on the area. - ​NO bona fide claim of acquisition or ownership, since June 12, 1945,
except when prevented by war or force majeure. These shall be
RELEVANT ARGUMENTS (if any): conclusively presumed to have performed all the conditions
● Petitioner: Petitioners insist that the subject properties could no essential to a Government grant and shall be entitled to a certificate
longer be considered and classified as forest land since there are of title under the provisions of this chapter.
buildings, residential houses and even government structures
● Under this provision, in order that petitioners' application for ● The adverse possession which can be the basis of a grant of title in
registration of title may be granted, they must first establish the confirmation of imperfect title cases cannot commence until after
following: (1) that the subject land forms part of alienable and forest land has been declared and alienable.
disposable lands of the public domain and (2) that they have been
in open, continuous, exclusive and notorious possession and WHEREFORE, the petition is hereby DENIED.
occupation of the same under a bona fide claim of ownership, since
June 12, 1945, or earlier.
● In the instant case, petitioners did not adduce any evidence to the
effect that the lots subject of their application are alienable and
disposable land of the public domain.
● Instead, petitioners contend that the subject properties could no
longer be considered and classified as forest land since there are
building structures, residential houses and even government
buildings existing and standing on the area. This, however, is
hardly the proof required under the law.
● As clarified by this Court in ​Heirs of Jose Amunategui v. Director
of Forestry, a forested area classified as forest land of the public
domain does not lose such classification simply because loggers or
settlers may have stripped it of its forest cover.
● Parcels of land classified as forest land may actually be covered
with grass or planted with crops by kaingin cultivators or other
farmers. "Forest lands" do not have to be on mountains or in
out-of-the-way places.
● The classification of land is descriptive of its legal nature or status
and does not have to be descriptive of what the land actually looks
like.
● Unless and until the land classified as forest land is released in an
official proclamation to that effect so that it may form part of the
disposable agricultural lands of the public domain, the rules on
confirmation of imperfect title do not apply.
● Petitioners' possession of the subject forest land prior to the date
when it was classified as alienable and disposable is
inconsequential and should be excluded from the computation of
the period of possession. (The subject lots were declared alienable
and disposable only on October 30, 1986.)
● To reiterate, it is well settled that possession of forest land, prior to
its classification as alienable and disposable land, is ineffective
since such possession may not be considered as possession in the
concept of owner.
51. TAN v. REPUBLIC
DOCTRINE: Possession and occupation of an alienable and disposable
April 16, 2012 ​| J. Reyes | Land of Public Domain: When Alienable and
public land for the periods provided under the Civil Code will not convert
Disposable
it to patrimonial or private property. There must be an express declaration
B. BLANCO & N. CALDOZO
that the property is no longer intended for public service or the
development of national wealth. In the absence thereof, the property
PETITIONER: ​Jean Tan, Roseller Anacinto, Carlo Lolo Espineda, and remains to be alienable and disposable and may not be acquired by
Daisy Aliado Manaois, represented in this act by their Attorney-in-Fact prescription under Section 14(2) of P.D. No. 1529.
Ma. Wilhelmina E. Tobias
RESPONDENT: ​Republic of the Philippines Moreover, tax declarations per se do not qualify as competent evidence
of actual possession for purposes of prescription. More so, if the payment
RECIT-READY: Petitioners Jean Tan, et.al filed with the Regional of the taxes due on the property is episodic, irregular and random.
Trial Court (RTC) of Naic, Cavite, an application for land registration Adverse, continuous, open, public possession in the concept of an owner
covering a parcel of land identified as Lot 9972 in Indang, Cavite. Tan, is a conclusion of law and the burden to prove it by clear, positive and
et.al ​alleged that they acquired the subject property from Gregonio convincing evidence is on the applicant. A claim of ownership will not
Gatdula pursuant to a Deed of Absolute Sale dated April 25, 1996. proper on the basis of tax declarations if unaccompanied by proof of
Moreover, they and their predecessors-in-interest contended that they actual possession.
have been in open, continuous and exclusive possession of the subject
property in the concept of an owner for more than 30 years.
FACTS:
● Petitioners Jean Tan, et.al ​filed with the Regional Trial Court
ISSUE:
(RTC) of Naic, Cavite, an application for land registration
● Whether or not Tan et.al have proven themselves qualified to
covering a parcel of land ​identified as Lot 9972 ​in Indang
the benefits under the relevant laws on the confirmation of
Cadastre, situated in Barangay Bancod, Indang, Cavite and
imperfect or incomplete titles. - ​NO.
with an area of 6,920 square meters.
● Tan, et.al ​alleged that they acquired the subject property from
RULING: ​The Court finds the evidence presented by Tan, et.al to be
Gregonio Gatdula pursuant to a Deed of Absolute Sale dated
wanting. They failed to demonstrate that they and their
April 25, 1996. Moreover, they and their predecessors-in-interest
predecessors-in-interest possessed the property in the requisite manner in
contended that they have been in open, continuous and exclusive
that possession should be in the concept of an owner, public, peaceful,
possession of the subject property in the concept of an owner
uninterrupted and adverse. In this case, the taxes were paid only on 11
for more than 30 years.
occasions within the 40-year period from 1961 to 2001. Moreover, the
● RTC granted Tan et.al’s petition.
application was filed after only 1 year from the time the subject property
● On appeal by the Republic of the Philippines, the Court of Appeals
may be considered patrimonial. DARCO Conversion Order No.
held that the petitioners failed to prove that they and their
040210005-(340)-99 was issued by the DAR only on July 13, 2000,
predecessors-in-interest have been in possession of the subject
which means that the counting of the 30-year prescriptive period for
property for the requisite period of 30 years.
purposes of acquiring ownership of a public land under Section 14(2) can
● Tan, et.al moved for reconsideration but this was denied by the
only start from such date. Before the property was declared patrimonial
CA. Hence, the present petition.
by virtue of such conversion order, it cannot be acquired by prescription.
ISSUE: 7. Certification issued by the Department of
● Whether or not Tan et.al have proven themselves qualified to the Environment and Natural Resources (DENR) –
benefits under the relevant laws on the confirmation of imperfect CALABARZON ​dated October 29, 2002, stating that
or incomplete titles? ​- NO. “the subject area falls within the Alienable and
Disposable Land Project No. 13-A of Indang, Cavite per
RELEVANT ARGUMENT: LC Map 3091 certified on June 21, 1983.”
● Tan, et.al contend that the following sufficed to demonstrate that
they acquired title over the subject property ​by prescription​: RATIO:
1. Testimony of their attorney-in-fact, Ma. Wilhelmina ● Commonwealth Act No. 141, otherwise known as the “Public
Tobias​, stating that: Land Act” governs ​the classification and disposition of lands
○ Tan et.al have been in actual, notorious, and open forming part of the public domain.
possession of the property since the time they ○ Section 11 thereof provides that ​one of the modes of
purchased it; disposing public lands suitable for agricultural
○ They regularly paid their taxes; purposes is by “confirmation of imperfect or
○ Their predecessors-in-interest, Victorio Garcia, incomplete titles.”
Felipe Gatdula, and Gregonio Gatdula have been in ○ Section 48 thereof ​enumerates those who are considered
possession of property for more than 30 years and to have acquired an imperfect or incomplete title over an
likewise religiously paid taxes; alienable and disposable public land.
○ The subject property is agricultural, alienable, and ● Presidential Decree No. 1529​, otherwise known as the “Property
disposable. Registration Decree”, is a codification of all the laws relative to
2. Testimony of Margarito Pena, the caretaker of the the registration of property​.
property; ○ Section 14 thereof ​specifies those who are qualified to
○ He witnessed the execution of the deed of sale that register their incomplete title over an alienable and
Tan et.al entered into with Gregonio Gatdula; and disposable public land under the Torrens system,
○ Tan, et.al and their predecessors-in-interest have been particularly:
in possession of the property for more than 30 years.
3. Testimony of Ferdinand Encarnacion, a clerk in the Section 14. ​Who may apply. The following persons may file in the
proper Court of First Instance an application for registration of title to
Docket Division of the Land Registration Authority
land, whether personally or through their authorized representatives:
(LRA);
4. Tax Declarations; (1) Those who by themselves or through their predecessors-in-interest
5. Resolution No. 69, Series of 1998, of the Sangguniang have been in open, continuous, exclusive and notorious possession and
Bayan of Indang, Cavite, ​which approved the occupation of alienable and disposable lands of the public domain under
a bona fide claim of ownership since June 12, 1945, or earlier.
reclassification of several lots, including the subject
property, from agricultural to residential/commercial; (2) Those who have acquired ownership of private lands by
6. DARCO Conversion Order No. 040210005-(340)-99, prescription under the provision of existing laws.
Series of 2000, issued by the Department of Agrarian
(3) Those who have acquired ownership of private lands or abandoned
Reform ​on July 13, 2000, which converted several river beds by right of accession or accretion under the existing laws.
parcels of land, including the subject property, from
agricultural to residential/commercial; and
(4) Those who have acquired ownership of land in any other manner the burden to prove it by clear, positive and convincing
provided for by law [emphasis supplied].
evidence is on the applicant. A claim of ownership will not
proper on the basis of tax declarations if unaccompanied by
● Possession and occupation of an alienable and disposable proof of actual possession.
public land for the periods provided under the Civil Code will ● Finally, the application was filed after only 1 year from the time
not convert it to patrimonial or private property. ​There must be the subject property may be considered patrimonial. ​DARCO
an ​express declaration that the property is no longer intended for Conversion Order No. 040210005-(340)-99 was issued by the
public service or the development of national wealth. In the DAR only on July 13, 2000, which means that the counting of the
absence thereof, the property remains to be alienable and thirty 30-year prescriptive period for purposes of acquiring
disposable and may not be acquired by prescription under Section ownership of a public land under Section 14(2) can only start from
14(2) of P.D. No. 1529. such date. ​Before the property was declared patrimonial by
● Tan’s application is obviously anchored on Section 14(2) of P.D. virtue of such conversion order, it cannot be acquired by
No. 1529 as they do not claim to have possessed, by themselves or prescription.
their predecessors-in-interest, the subject property since June 12,
1945 or earlier; thus, they posit that ​through prescription that WHEREFORE, ​premises considered, the instant petition is DENIED
they had acquired an imperfect title over the subject property for lack of merit. The July 6, 2009 Decision and August 12, 2010
is the foundation upon which they rest their application. Resolution of the Court of Appeals are AFFIRMED.
● However, the Court finds the evidence presented by the Tan,
et.al to be wanting. They failed to demonstrate that they and their
predecessors-in-interest possessed the property in the requisite
manner:
It is concerned with lapse of time in the manner and under conditions laid down by
law, namely, that ​the possession should be in the concept of an owner, public,
peaceful, uninterrupted and adverse. ​Possession is open when it is patent,
visible, apparent, notorious and not clandestine. It is continuous when
uninterrupted, unbroken and not intermittent or occasional; exclusive when
the adverse possessor can show exclusive dominion over the land and an
appropriation of it to his own use and benefit; and notorious when it is so
conspicuous that it is generally known and talked of by the public or the people
in the neighborhood. ​The party who asserts ownership by adverse possession must
prove the presence of the essential elements of acquisitive prescription.

● The Court ruled that, tax declarations ​per se do not qualify as


competent evidence of actual possession for purposes of
prescription. ​More so, if the payment of the taxes due on the
property is episodic, irregular and random such as in this case.
In this case, the taxes were paid only on 11 occasions within the
40-year period from 1961 to 2001.
● It was emphasized that adverse, continuous, open, public
possession in the concept of an owner is a conclusion of law and
52. REPUBLIC OF THE PHILIPPINES v. BANTIGUE POINT
RULING:
DEVELOPMENT COPORATION
(A) The RTC's failure to issue the Order setting the date and hour of the
Mar 14 2012 | J. Sereno | 52. Forms and contents of petition for original
initial hearing within five days from the filing of the application for
application of title: Where to file. / 59. Publication, Opposition, Order
registration, as provided in the Property Registration Decree, did not
of General default, Hearing, Judgement, and decree of registration:
affect the court's jurisdiction. Observance of the five- day period was
Notice of initial hearing
merely directory, and failure to issue the Order within that period did not
N. CALDOZO & CHECKER
deprive the RTC of its jurisdiction over the case. To rule that compliance
with the five-day period is mandatory would make jurisdiction over the
PETITIONER: ​Republic of the Philippines subject matter dependent upon the trial court. Jurisdiction over the
RESPONDENT: ​Bantigue Point Development Corporation subject matter is conferred only by the Constitution or the law. It cannot
be contingent upon the action or inaction of the court.
RECIT-READY: ​On 17 July 1997, Bantigue Point Development
Corporation filed with the RTC of Rosario, Batangas an application for (B) The value of the property must therefore be ascertained with
original registration of title over a parcel of land. On 18 July 1997, the reference to the corresponding ​Tax Declarations submitted by
RTC issued an Order setting the case for initial hearing on 22 October respondent Corporation together with its application for registration.
1997. On 7 August 1997, it issued a second Order setting the initial From the records, we finds that the assessed value of the property is
hearing on 4 November 1997. On 8 January 1998, Republic filed its P4,330, P1,920 and P8,670, or a total assessed value of P14,920 for the
Opposition to the application for registration while the records were still entire property. ​Based on these Tax Declarations, it is evident that the
with the RTC. In assailing the jurisdiction of the lower courts, Republidc total value of the land in question does not exceed P100,000. ​Clearly,
argued ​first​, the lower court failed to acquire jurisdiction over the the MTC may exercise its delegated jurisdiction under the Judiciary
application, because the RTC set the date and hour of the initial hearing Reorganization Act, as amended.
beyond the 90-day period provided under the Property Registration
Decree. ​Second​, petitioner contended that since the selling price of the DOCTRINE:
property based on the Deed of Sale annexed to respondent's application (A) Observance of the five- day period was merely directory, and failure
for original registration was P160,000,the MTC did not have jurisdiction to issue the Order within that period did not deprive the RTC of its
over the case. Under Section 34 of the Judiciary Reorganization Act, as jurisdiction over the case. To rule that compliance with the five-day
amended,the MTC's delegated jurisdiction to try cadastral and land period is mandatory would make jurisdiction over the subject matter
registration cases is limited to lands, the value of which should not dependent upon the trial court.
exceed P100,000. (B) The MTC has delegated jurisdiction in cadastral and land registration
cases in two instances: ​first​, where there is no controversy or opposition;
ISSUE: or, ​second​, over contested lots, the value of which does not exceed
● W/N The municipal trial court acquired jurisdiction over the P100,000.
application for original registration of land title? YES Section 34 of the Judiciary Reorganization Act provides that the value of
○ (a) the period for setting the date and hour of the initial the property sought to be registered may be ascertained in three ways:
hearing; and first, by the affidavit of the claimant; ​second, by agreement of the
○ (b) the value of the land to be registered. respective claimants, if there are more than one; ​or​, ​third​, ​from the
corresponding tax declaration of the real property.
FACTS: Second, petitioner contended that since the selling price of the property
● On 17 July 1997, Bantigue Point Development Corporation filed based on the Deed of Sale annexed to respondent's application for original
with the Regional Trial Court (RTC) of Rosario, Batangas an registration was P160,000,the MTC did not have jurisdiction over the case.
application for original registration of title over a parcel of land Under Section 34 of the Judiciary Reorganization Act, as amended,the
with an assessed value of P4,330, P1,920 and P8,670, or a total MTC's delegated jurisdiction to try cadastral and land registration cases is
assessed value of P14,920 for the entire property, more particularly limited to lands, the value of which should not exceed P100,000.
described as Lot 8060 of Cad 453-D, San Juan Cadastre, with an
area of more or less 10,732 square meters, located at Barangay RATIO:
Barualte, San Juan, Batangas. (not super relevant) Republic is not estopped from raising the issue of
● On 18 July 1997, the RTC issued an Order setting the case for jurisdiction in this case
initial hearing on 22 October 1997. On 7 August 1997, it issued a ● In this case, petitioner Republic has not displayed such
second Order setting the initial hearing on 4 November 1997. unreasonable failure or neglect that would lead us to conclude that
● Petitioner Republic filed its Opposition to the application for it has abandoned or declined to assert its right to question the lower
registration on 8 January 1998 while the records were still with the court's jurisdiction.
RTC.
● On 31 March 1998, the RTC Clerk of Court transmitted motu The municipal trial court acquired jurisdiction over the application for
proprio the records of the case to the MTC of San Juan, because original registration of land title
the assessed value of the property was allegedly less than ● In assailing the jurisdiction of the lower courts, petitioner Republic
P100,000. raised two points of contention: ​(a) the period for setting the date
● Thereafter, the MTC entered an Order of General Default and and hour of the initial hearing and ​(b) the value of the land to be
commenced with the reception of evidence.Among the documents registered.
presented by respondent in support of its application are Tax ● A. The period for setting the date and hour of the initial hearing
Declarations,a Deed of Absolute Sale in its favor,and a ● The Property Registration Decree provides:
Certification from the Department of Environment and Natural ○ Sec. 23. Notice of initial hearing, publication, etc. — The court shall,
within five days from filing of the application, issue an order setting the
Resources (DENR) Community Environment and Natural
date and hour of the initial hearing which shall not be earlier than
Resources Office (CENRO) of Batangas City that the lot in forty-five days nor later than ninety days from the date of the order.
question is within the alienable and disposable zone.Thereafter, it ● In this case, the application for original registration was filed on 17
awarded the land to respondent Corporation. July 1997. On 18 July 1997, or a day after the filing of the
application, the RTC immediately issued an Order setting the case
ISSUES: for initial hearing on 22 October 1997, which was 96 days from the
● W/N the Republic is estopped from raising the issue of jurisdiction Order. While the date set by the RTC was beyond the 90-day
in this case? NO period provided for in Section 23, this fact did not affect the
● W/N the municipal trial court acquired jurisdiction over the jurisdiction of the trial court. In Republic v. Manna Properties,
application for original registration of land title? YES Inc., petitioner Republic therein contended that there was failure
to comply with the jurisdictional requirements for original
RELEVANT ARGUMENTS: registration, because there were 125 days between the Order
Petitioner: First, the lower court failed to acquire jurisdiction over the setting the date of the initial hearing and the initial hearing itself.
application, because the RTC set the date and hour of the initial hearing We ruled that the lapse of time between the issuance of the Order
beyond the 90-day period provided under the Property Registration Decree.
setting the date of initial hearing and the date of the initial hearing law.
itself was not fatal to the application. ​Thus, we held: ● B. the value of the land to be registered.
○ . . . [A] party to an action has no control over the Administrator or the ● The delegated jurisdiction of the MTC over cadastral and land
Clerk of Court acting as a land court; he has no right to meddle unduly
registration cases is indeed set forth in the Judiciary
with the business of such official in the performance of his duties. A
party cannot intervene in matters within the exclusive power of the trial Reorganization Act, which provides:
court. No fault is attributable to such party if the trial court errs on ○ Sec. 34. Delegated Jurisdiction in Cadastral and Land Registration
matters within its sole power. It is unfair to punish an applicant for an Cases. — Metropolitan Trial Courts, Municipal Trial Courts, and
act or omission over which the applicant has neither responsibility nor Municipal Circuit Trial Courts may be assigned by the Supreme Court to
control, especially if the applicant has complied with all the hear and determine ​cadastral or land registration cases covering lots
requirements of the law. where there is no controversy or opposition, or contested lots where the
● Indeed, it would be the height of injustice to penalize respondent value of which does not exceed One hundred thousand pesos
(P100,000.00)​, such value to be ascertained by the affidavit of the
Corporation by dismissing its application for registration on claimant or by agreement of the respective claimants if there are more
account of events beyond its control. than one, or from the corresponding tax declaration of the real property.
● Moreover, since the RTC issued a second Order on 7 August 1997 Their decision in these cases shall be appealable in the same manner as
setting the initial hearing on 4 November 1997,within the 90-day decisions of the Regional Trial Courts. (As amended by R.A. No. 7691)
period provided by law, petitioner Republic argued that the (Emphasis supplied.) ​CAaEDH
jurisdictional defect was still not cured, as the second Order was ● Thus, the MTC has delegated jurisdiction in cadastral and
issued more than five days from the filing of the application, again land registration cases in two instances: first, where there is no
contrary to the prescribed period under the Property Registration controversy or opposition; ​or, second, over contested lots, the
Decree. value of which does not exceed P100,000.
● Petitioner is incorrect. T​he RTC's failure to issue the Order setting ● The case at bar does not fall under the first instance, because
the date and hour of the initial hearing within five days from the petitioner opposed respondent Corporation's application for
filing of the application for registration, as provided in the Property registration on 8 January 1998.
Registration Decree, did not affect the court's jurisdiction. ● However, the MTC had jurisdiction under ​the second instance,
Observance of the five- day period was merely directory, and because the value of the lot in this case does not exceed P100,000.
failure to issue the Order within that period did not deprive the ● Contrary to petitioner's contention, the value of the land should not
RTC of its jurisdiction over the case. To rule that compliance with be determined with reference to its selling price. ​Rather, Section 34
the five-day period is mandatory would make jurisdiction over the of the Judiciary Reorganization Act provides that the value of the
subject matter dependent upon the trial court. Jurisdiction over the property sought to be registered may be ascertained in three ways:
subject matter is conferred only by the Constitution or the law. It first, by the affidavit of the claimant; second, by agreement of the
cannot be contingent upon the action or inaction of the court. respective claimants, if there are more than one; or, third, from the
● This does not mean that courts may disregard the statutory periods corresponding tax declaration of the real property.
with impunity. We cannot assume that the law deliberately meant ● In this case, the value of the property cannot be determined using
the provision "to become meaningless and to be treated as a dead the first method, because the records are bereft of any affidavit
letter." However, the records of this case do not show such blatant executed by respondent as to the value of the property. Likewise,
disregard for the law. In fact, the RTC immediately set the case for valuation cannot be done through the second method, because this
initial hearing a day after the filing of the application for method finds application only where there are multiple claimants
registration, except that it had to issue a second Order because the who agree on and make a joint submission as to the value of the
initial hearing had been set beyond the 90-day period provided by property. Here, only respondent Bantigue Point Development
Corporation claims the property.
● The value of the property must therefore be ascertained with
reference to the corresponding ​Tax Declarations submitted by
respondent Corporation together with its application for
registration. From the records, we finds that the assessed value of
the property is P4,330, P1,920 and P8,670, or a total assessed value
of P14,920 for the entire property. ​Based on these Tax
Declarations, it is evident that the total value of the land in
question does not exceed P100,000.
● Clearly, the MTC may exercise its delegated jurisdiction under
the Judiciary Reorganization Act, as amended.
A certification from the CENRO is not sufficient proof that the
property in question is alienable and disposable land of the public
domain.
● Even as we affirm the propriety of the MTC's exercise of its
delegated jurisdiction, we find that the lower court erred in
granting respondent Corporation's application for original
registration in the absence of sufficient proof that the property in
question was alienable and disposable land of the public domain.
● Thus, the present rule is that an application for original registration
must be accompanied by (1) a CENRO or PENRO Certification;
and (2) a copy of the original classification approved by the DENR
Secretary and certified as a true copy by the legal custodian of the
official records.
● Here, respondent Corporation only presented a CENRO
certification in support of its application. Clearly, this falls short of
the requirements for original registration.

WHEREFORE, premises considered, the instant Petition for Review is


DENIED. Let this case be REMANDED to the Municipal Trial Court
of San Juan, Batangas, for reception of evidence to prove that the
property sought to be registered is alienable and disposable land of the
public domain.
53. FORTUNA VS. REPUBLIC
The nature of Lot No. 4457 as alienable and disposable public land has
5 March 2014 | J. Brion |
not been sufficiently established.
‘When is a land of public domain alienable and disposable?’ and the
Judicial Confirmation of Imperfect or Incomplete Titles.
The Constitution declares that all lands of the public domain are owned
Calilung & Checker
by the State. Of the four classes of public land, i.e., agricultural lands,
forest or timber lands, mineral lands, and national parks, only agricultural
PETITIONER:​SPS. ANTONIO FORTUNA and ERLINDA lands may be alienated. Public land that has not been classified as
FORTUNA alienable agricultural land remains part of the inalienable public domain.
RESPONDENT: ​REPUBLIC OF THE PHILIPPINES Thus, it is essential for any applicant for registration of title to land
derived through a public grant to establish foremost the alienable and
RECIT-READY: ​Spouses Fortuna filed an application with the RTC for disposable nature of the land. The PLA provisions on the grant and
registration of a 2,597-square meter land identified as (Lot) Lot No. disposition of alienable public lands, specifically, Sections 11 and 48 (b),
4457, situated in Bo. Canaoay, San Fernando, La Union. They stated that will find application only from the time that a public land has been
the Lot was originally owned by Pastora Vendiola, upon whose death classified as agricultural and declared as alienable and disposable. Under
was succeeded by her children, Clemente and Emeteria Nones. Through Section 6 of the PLA, the classification and reclassification of public
an affidavit of adjudication dated August 3, 1972, Emeteria renounced all lands are the prerogative of the Executive Department. The President,
her interest in Lot No. 4457 in favor of Clemente. Clemente later sold the through a presidential proclamation or executive order, can classify or
lot in favor of Rodolfo Cuenca on May 23, 1975. Rodolfo sold the same reclassify a land to be included or excluded from the public domain. The
lot to the spouses Fortuna. Hence they claim that they, through Department of Environment and Natural Resources (DENR) Secretary is
themselves and their predecessors in-interest, have been in quiet, likewise empowered by law to approve a land classification and declare
peaceful, adverse and uninterrupted possession of the Lot for more than such land as alienable and disposable. Accordingly, jurisprudence has
50 years, and submitted as evidence the lot's survey plan, technical required that an applicant for registration of title acquired through a
description, and certificate of assessment. RTC granted. public land grant must present incontrovertible evidence that the land
Republic appealed the RTC decision with the CA, arguing that the subject of the application is alienable or disposable by establishing the
spouses Fortuna did not present an official proclamation from the existence of a positive act of the government, such as a presidential
government that the lot has been classified as alienable and disposable proclamation or an executive order; an administrative action;
agricultural land. It also claimed that the spouses Fortuna's evidence — investigation reports of Bureau of Lands investigators; and a legislative
Tax Declaration No. 8366 — showed that possession over the lot dates act or a statute.
back only to 1948, thus, failing to meet the June 12, 1945 cut-off period
provided under Section 14 (1) of Presidential Decree (PD) No. 1529 or In this case, the CA declared that the alienable nature of the land was
the Property Registration Decree (PRD). CA reversed and set aside, established by the​ notation in the survey plan​, which states:
hence the present petition. “This survey is inside alienable and disposable area as per Project
No. 13 L.C. Map No. 1395 certified August 7, 1940. It is outside any
ISSUE: civil or military reservation.”
W/N the nature of Lot No. 4457 is alienable and disposable public land.
It also relied on the ​Certification from the DENR-CENRO that "there is,
RULING: per record, neither any public land application led nor title previously
issued for the subject parcel[.]
only from the time that a public land has been classified as agricultural
However, we nd that neither of the above documents is evidence of a and declared as alienable and disposable. (Note: Take note of the notation
positive act from the government reclassifying the lot as alienable and in the survey plan and the DENR-CENRO Certification. Why they are
disposable agricultural land of the public domain Mere notations insufficient in this case.)
appearing in survey plans are inadequate proof of the covered properties'
alienable and disposable character.
Ruling: ​(On another relevant issue, however not the main issue)
These notations, at the very least, only establish that the land subject of
the application for registration falls within the approved alienable and In judicial confirmation of imperfect or incomplete title, the period of
disposable area per verification through survey by the proper government possession should commence, at the latest, as of May 9, 1947. Although the
office. The applicant, however, must also present a copy of the original above finding that the spouses Fortuna failed to establish the alienable and
classification of the land into alienable and disposable land, as declared disposable character of Lot No. 4457 serves as sufficient ground to deny the
by the DENR Secretary or as proclaimed by the President. In Republic v. petition and terminate the case, we deem it proper to continue to address the
Heirs of Juan Fabio the Court ruled that [t]he applicant for land other important legal issues raised in the petition. As mentioned, the PLA is
registration must prove that the DENR Secretary had approved the land
the law that governs the grant and disposition of alienable agricultural lands.
classification and released the land of the public domain a alienable and
disposable, and that the land subject of the application fo registration falls Under Section 11 of the PLA, alienable lands of the public domain may be
within the approved area per verification through survey by the PENRO disposed of, among others, by judicial confirmation of imperfect or
or CENRO. In addition, the applicant must present a copy of the original incomplete title. This mode of acquisition of title is governed by Section 48
classification of the land into alienable and disposable, as declared by the (b) of the PLA, the original version of which states:
DENR Secretary, or as proclaimed by the President.
The survey plan and the DENR-CENRO certification are not proof that Sec. 48. The following-described citizens of the Philippines,
the President or the DENR Secretary has reclassified and released the occupying lands of the public domain or claiming to own any such
public land as alienable and disposable. The oces that prepared these
lands or an interest therein, but whose titles have not been
documents are not the ocia repositories or legal custodian of the
issuances of the President or the DENR Secretary declaring the public perfected or completed, may apply to the Court of First Instance of
land as alienable and disposable. For failure to present incontrovertible the province where the land is located for confirmation of their
evidence that Lot No. 4457 has been reclassified as alienable and claims and the issuance of a certificate of title therefor, under the
disposable land of the public domain though a positive act of the Land Registration Act, to wit:
Executive Department, the spouses Fortuna's claim of title through a xxx xxx xxx
public land grant under the PLA should be denied. (b) Those who by themselves or through their
predecessors-in-interest
DOCTRINE: ​Public land that has not been classified as alienable
have been in open, continuous, exclusive, and notorious possession
agricultural land remains part of the inalienable public domain. Thus, it is
essential for any applicant for registration of title to land derived through and
a public grant to establish foremost the alienable and disposable nature of occupation of agricultural lands of the public domain, under a bona
the land. The PLA provisions on the grant and disposition of alienable fide claim of acquisition or ownership, except as against the
public lands, specifically, Sections 11 and 48 (b), will find application Government, ​since July twenty sixth, eighteen hundred and
ninety-four​, except when prevented by war or force majeure. These commenced only after the cut-off date of June 12, 1945 was established by
shall be conclusively presumed to have performed all the the PD No. 1073 amendment. To remedy this, the Court ruled in Abejaron
conditions essential to a government grant and shall be entitled to a v. Nabasa 30 that "Filipino citizens who by themselves or their
certificate of title under the provisions of this chapter. [emphasis predecessors-in interest have been, prior to the effectivity of P.D. 1073 on
supplied] January 25, 1977 , in open, continuous, exclusive and notorious possession
and occupation of agricultural lands of the public domain, under a bona fide
On June 22, 1957, the cut-off date of July 26, 1894 was replaced by a claim of acquisition of ownership, for at least 30 years, or at least since
30-year period of possession under RA No. 1942. Section 48 (b) of the January 24, 1947 may apply for judicial confirmation of their imperfect or
PLA, as amended by RA No. 1942, read: incomplete title under Sec. 48 (b) of the [PLA]." January 24, 1947 was
(b) Those who by themselves or through their predecessors in considered as the cut-off date as this was exactly 30 years counted
interest have been in open, continuous, exclusive and notorious backward from January 25, 1977 — the effectivity date of PD No. 1073.
possession and occupation of agricultural lands of the public
domain, under a bona fide claim of acquisition of ownership, ​for at It appears, however, that January 25, 1977 was the date PD No. 1073 was
least thirty years​, immediately preceding the ruling of the enacted ; based on the certification from the National Printing O ce, 31 PD
application for confirmation of title, except when prevented by war No. 1073 was published in Vol. 73, No. 19 of the O cial Gazette , months
or force majeure. [emphasis supplied] later than its enactment or on May 9, 1977. This uncontroverted fact
materially affects the cut-off date for applications for judicial confirmation
On January 25, 1977, PD No. 1073 replaced the 30-year period of of incomplete title under Section 48 (b) of the PLA.
possession by requiring possession since June 12, 1945. Section 4 of PD
No. 1073 reads: Although Section 6 of PD No. 1073 states that "[the] Decree shall take
SEC. 4. The provisions of Section 48(b) and Section 48(c), effect upon its promulgation," the Court has declared in Tañada, et al. v.
Chapter Hon. Tuvera, etc., et al. 32 that the publication of laws is an indispensable
VIII of the Public Land Act are hereby amended in the sense that requirement for its effectivity. "[A]ll statutes, including those of local
these provisions shall apply only to alienable and disposable lands application and private laws, shall be published as a condition for their
of the public domain which have been in open, continuous, effectivity, which shall begin fifteen days after publication unless a different
exclusive and notorious possession and occupation by the applicant effectivity date is fixed by the legislature." 33 Accordingly, Section 6 of PD
himself or thru his predecessor-in-interest, under a bona fide claim No. 1073 should be understood to mean that the decree took effect only
of acquisition of ownership, ​since June 12, 1945​. [emphasis upon its publication, or on May 9, 1977. ​This, therefore, moves the cut-off
supplied] date for applications for judicial confirmation of imperfect or incomplete
title under Section 48 (b) of the PLA to May 8, 1947. In other words,
Under the PD No. 1073 amendment, possession of at least 32 years — from applicants must prove that they have been in open, continuous, exclusive
1945 up to its enactment in 1977 — is required. This effectively impairs the and notorious possession and occupation of agricultural lands of the public
vested rights of applicants who had complied with the 30-year possession domain, under a bona fide claim of acquisition of ownership, for at least 30
required under the RA No. 1942 amendment, but whose possession years, or at least since May 8, 1947​.
WHEREFORE,the petition is DENIED. The decision dated May 16,
2005 and the resolution dated June 27, 2006 of the Court of Appeals in
CA-G.R. CV No. 71143 are AFFIRMED insofar as these dismissed the
spouses Antonio and Erlinda Fortuna's application of registration of
title on the basis of the grounds discussed above. Costs against the
spouses Fortuna.
SO ORDERED.
54. REPUBLIC v. HEIRS OF SIN
it was susceptible of private ownership ​before 1960. The lower courts
March 26, 2014 | J. Leonardo-De Castro | Land of ruled that the land was alienable and disposable before 1960, since the
Public Domain: When Alienable and Disposable Republic failed to prove otherwise. However, SC held that ​the
E. CANCEKO & S. CERILLA presumption is that ​unclassified lands are inalienable public lands.
To overcome this presumption, there must be a positive act (like an
official proclamation) ​declaring land of the public domain as alienable
PETITIONER: ​Republic of the Philippines (represented by Aklan and disposable. ​The burden of proof should fall on the one alleging that
National College Fisheries and Dr. Elenita Andrade) it is alienable and disposable, not the other way around.
RESPONDENT: ​Heirs of Maxima Lachica Sin
DOCTRINE: ​All lands of the public domain belong to the State and
RECIT-READY: ​Maxima Sin bought a parcel of land in 1932. She lands not appearing to be clearly within private ownership are
planted coconut trees, banana plants, and nipa palms on such land until presumed to belong to the State. ​(Regalian Doctrine)
the day she died. Her heirs then inherited the land. In 1988, a portion of
the land was occupied by ANCF and was converted into a fishpond.
Because of this, the heirs instituted a complaint against ANCF for FACTS:
recovery of possession​. ● Maxima Sin acquired a parcel of land by virtue of a Deed of Sale
in 1932, and then developed the same by planting coconut trees,
The Republic, through ANCF, countered that (1) the land was the subject banana plants, mango trees and nipa palms and usufructing the
of Proclamation No. 2074 allocating such land as ​civil reservation f​ or produce of said land until her death in 1945.
educational purposes, and that (2) the land had also been declared a
● The heirs of Sin asserted that they were previously in possession of
timberland i​ n 1960 and therefore not susceptible of private ownership.
the disputed land (in the concept of an owner) before a portion of
The MCTC, RTC, and CA ruled in favor of the heirs of Sin. It was held the land was occupied by ANCF and converted into a fishpond for
that the Republic failed to show that the land was ​timberland ​before educational purposes in 1988.
1960​. ​Therefore, it was susceptible of private ownership ​until 1960. ● To prove possession, the heirs presented several ​tax declarations,​
Since such land had been privately possessed for more than 30 years the earliest of which was in the year 1945.
before it was declared a ​timberland, ​Maxima’s possession had ripened ● The ANCF Superintendent countered that the parcel of land being
​ herefore, the Proclamation cannot be
into an ownership (imperfect title). T claimed by respondents was the subject of ​Proclamation No. 2074
applied on the disputed land.
of then President Ferdinand E. Marcos allocating land within the
SC reversed and held that the land was never susceptible of area, which included said portion of private respondents' alleged
​ ence, neither
ownership even before it was declared a ​timberland. H property, as ​civil reservation​ for educational purposes of ANCF.
Maxima nor her heirs acquired private rights over the land. ● The ANCF Superintendent furthermore averred that the parcel of
land was declared a ​timberland in 1960 and therefore not
Aside from open, continuous, exclusive, and notorious possession, the susceptible of private ownership.
other requisite for an ​imperfect title is ​the classification of the land as ● The MCTC, the RTC and the Court of Appeals unanimously held
alienable and disposable land of the public domain. ​It is clear that the that the heirs of Sin retain private rights to the disputed property.
disputed land became a ​timberland in 1960. The issue was whether or not
● The private right referred to is an alleged ​imperfect title, which the land was declared a ​timberland before its formal classification as
heirs supposedly acquired by possession of the property, through such on said year.
their predecessors-in-interest, for 30 years ​before it was declared as ● However, under the​ ​Regalian Doctrine:
a ​timberland​. ○ All lands of the public domain belong to the State and
● The Court of Appeals held: lands not appearing to be clearly within private
○ “It should be noted that Maxima Sin purchased the ownership are presumed to belong to the State.
subject property from its previous owners in 1932 (28 ● Property of the public domain is beyond the commerce of man and
years before it was declared a ​timberland in 1960). not susceptible of private appropriation and acquisitive
Tacking, therefore, the possession of the previous owners prescription. ​Occupation thereof in the concept of owner no
and that of Maxima Sin over the disputed property, it does matter how long cannot ripen into ownership and be registered
not tax one’s imagination to conclude that the subject as a title.
property had been privately possessed for more than 30 ● To overcome this presumption, incontrovertible evidence must be
years before it was declared a timberland. This being the established that the land subject of the application (or claim) is
case, the said possession has ripened into an ownership alienable or disposable. There must be a positive act declaring
against​ the State, albeit an imperfect one.” land of the public domain as alienable and disposable.
● The failure of the Republic to show competent evidence that the
ISSUES: ​W/N the disputed property was alienable and disposable - ​NO subject land was declared a ​timberland before its formal
classification does ​not lead to the presumption that said land was
RATIO: alienable and disposable. ​On the contrary, the presumption is
● At the outset, it must be noted that the heirs have not filed an that unclassified lands are inalienable public lands.
application for ​judicial confirmation of imperfect title under the ● It is therefore the ​respondent heirs ​which have the burden to
Public Land Act or the Property Registration Decree. identify a ​positive act of the government, such as an official
● There are two requisites for ​judicial confirmation of imperfect or proclamation, declassifying inalienable public land into disposable
incomplete title​ under CA No. 141, namely: land for agricultural or other purposes.
○ (1) open, continuous, exclusive, and notorious possession ● Since respondents failed to do so, the alleged possession by
and occupation of the subject land by himself or through them and by their predecessors-in-interest is inconsequential
his predecessors-in-interest under a bona fide claim of and could never ripen into ownership.
ownership since time immemorial or from June 12, 1945;
○ (2) ​the classification of the land as alienable and WHEREFORE, ​premises considered, the Petition for Review is
disposable land of the public domain. GRANTED. The Decision of the Court of Appeals, which upheld the
● With respect to the second requisite, the lower courts held that the Decisions of the Regional Trial Court and the First Municipal Circuit Trial
disputed property was ​alienable and disposable before 1960, citing Court, segregating from the ANCF reservation the portion of land being
the Republic’s failure to show competent evidence that the subject claimed by respondents is ​REVERSED​ and ​SET ASIDE.
55.
56.
57. DUMO v. REPUBLIC
Whether or not the Subject Property is alienable and disposable (NO)
In Re: Application for Land Registration
Whther or not the Subject Property is a private land (NO)
6 June 2018 | G.R. No. 218269 | Carpio | Land of Public Domain: When
Alienable and Disposable
RULING:
M. del Rosario | T. dela Rosa
● The requirements for judicial confirmation of imperfect title are
found in ​Section 14​ o​f Presidential Decree No. 1529​ (PD No.
PETITIONER: ​Suprema T. Dumo 1529), which provides:
RESPONDENT: ​Republic of the Philippines
Section 14. Who may apply. The following persons
RECIT-READY: ​In 1943, Espinas purchased from Calica a parcel of may file in the proper Court of First Instance an
land located in Bauang, La Union classified as unirrigated riceland. In application for registration of title to land, whether
1963, Espinas executed an affidavit stating his claim of ownership over personally or through their duly authorized
the property. He had also been paying realty taxes on the property. In representatives:
1987, the heirs of Trinidad executed a Deed of Partition with Absolute
Sale over another parcel of land on the North of the Subject Property. (1) Those who by themselves or through their
The heirs of Espinas filed a Complaint for Recovery of Ownership, predecessors-in-interest have been in ​open, continuous,
Possession, and Damages upon finding that the Deed of Partition with exclusive and notorious possession and occupation​ of
Absolute Sale executed by the heirs of Trinidad included the Subject alienable and disposable lands of the public domain​ under a
Property. Dumo (an heir of Trinidad) filed an application for registration bona fide claim of ownership since June 12, 1945, or earlier.
of two parcels of land. She alleged that the lots belonged to her mother
and that she and her siblings inherited them upon their mother’s death. (2) Those who have acquired ownership of ​private
lands​ by prescription under the provision of existing laws.
The RTC found that the Subject Property was owned by the heirs of
Espinas and ordered the dismissal of Dumo’s land and registration (3) Those who have acquired ownership of private
application. CA affirmed the RTC decision dismissing the application for lands or abandoned river beds by right of accession or accretion
land registration. The CA also found that the Subject Property did not under the existing laws.
belong to the heirs of Espinas as it still belonged to the public domain.
(4) Those who have acquired ownership of land in any
Dumo seeks to reverse the CA decision by alleging her compliance with other manner provided for by law.
Sec. 14(1) and (2) of PD 1529 that she and her predecessors-in-interest
have been in open, continuous, exclusive, and notorious possession and
occupation of alienable and disposable lands of the public domain under ● Dumo failed to submit any of the documents required to
a bona fide claim of ownership since June 12, 1945. She also alleged that prove that the land she seeks to register is alienable and
her ownership is based on prescription. disposable land of the public domain.
● The CA found that Dumo and her predecessors-in-interest
ISSUE: Whether or not Dumo shall be allowed to apply for land
have been in possession of the land only from 1948, which is
registration (NO)
the earliest date of the tax declaration presented. ​Thus, by
Absolute Sale dated 6 February 1987, she acquired the subject lots
admission, it is clear that she failed to prove her and her
from her siblings.
predecessors-in-interest’s possession and occupation of the land ● Dumo traces her title from her mother, Trinidad, who purchased
for the duration required by law -- from June 12, 1945 or earlier. the lots from Florencio Mabalay in August 1951. Mabalay was
● Dumo cannot argue that she may be allowed to register due to Dumo's maternal grandfather. Mabalay, on the other hand,
prescription. Dumo not only failed to prove that the land sought purchased the properties from Carlos Calica.
to be registered is alienable and disposable, but also utterly ● The heirs of Espinas opposed Dumo's application for land
failed to submit any evidence to establish that such land has registration on the ground that the properties sought to be
been converted into patrimonial property by an express registered by Dumo are involved in the ​accion reivindicatoria ( is
an action whereby plaintiff alleges ownership over a parcel of land
declaration by the State.
and seeks recovery of its full possession) ​case. Thus, the RTC
consolidated the land registration case with the Complaint for
DOCTRINE:
Recovery of Ownership, Possession and Damages.
As it is only the President or the DENR Secretary who may classify as
● The RTC found that the Subject Property was owned by the heirs
alienable and disposable lands of the public domain, an applicant for land
of Espinas. The RTC ordered the dismissal of Dumo's land
registration must prove that the land sought to be registered has been
registration application on the ground of lack of registrable title,
declared by the President or DENR Secretary as alienable and disposable
and ordered Dumo to restore ownership and possession of the lots
land of the public domain.
to the heirs of Espinas.
● The CA affirming the RTC's decision dismissing the application
FACTS: for land registration of Dumo, and finding that she failed to
● October 19, 1943 - Espinas purchased from Calica through a Deed demonstrate that she and her predecessors-in¬ interest possessed
of Absolute Sale a parcel of land located in Paringao, Bauang, La the property in the manner required by law to merit the grant of her
Union classified as unirrigated Riceland with an area of 1,065 application for land registration.
square meters. ● The CA, however, modified the decision of the RTC insofar as it
● 1963 - Espinas executed an affidavit stating his claim of ownership found that the Subject Property belonged to the heirs of Espinas.
over the Subject Property. Espinas had also been paying realty The CA found that since the property still belonged to the public
taxes on the Subject Property. domain, and the heirs of Espinas were not able to establish their
open, continuous, exclusive and notorious possession and
● February 6, 1987 - the heirs of Trinidad executed a Deed of
occupation of the land under a bona fide claim of ownership since
Partition with Absolute Sale over another parcel of land on the
12 June 1945 or earlier, it was erroneous for the RTC to declare the
North of the Subject Property. heirs of Espinas as the owners of the Subject Property.
● Finding that the Deed of Partition with Absolute Sale executed by ● Dumo argues that the Certification from the Regional Surveys
the heirs of Trinidad included the Subject Property, the heirs of Division and not opposed by the Republic, should be considered
Espinas filed a Complaint for Recovery of Ownership, Possession substantial compliance with the requirement that the applicant
and Damages to protect their interests. must submit the certified true copy of the original classification of
● Additionally, Dumo filed an application for registration of two the land as approved by the DENR Secretary.
parcels of land. Dumo alleged that the lots belonged to her mother ● Dumo also argues that it does not matter that her possession dates
and that she and her siblings inherited them upon their mother's only back to 1948 because this Court has allegedly stated that even
death. She further alleged that through a Deed of Partition with
if the possession or occupation started after 12 June 1945, this does (4) Those who have acquired ownership of land in any
not bar the grant of an application for registration of land. other manner provided for by law.

ISSUE: ● Dumo failed to submit any of the documents required to prove


● Whether or not Dumo should be allowed to proceed with the that the land she seeks to register is alienable and disposable
application for land registration land of the public domain.
○ By proving that she or her predecessors-in-interet were in ● The first requirement is to prove that the land sought to be
open, continuous, exclusive and notorious possession and
registered is ​alienable and disposable land of the public domain.
occupation of alienable and disposable lands of the public
domain under a ​bona fide claim of ownership since June This is because under the Regalian Doctrine, as embodied in the
12, 1945 or earlier (NO); or 1987 Philippine Constitution, lands which do not clearly appear to
○ By proving that she has acquired ownership of private be within private ownership are presumed to belong to the State.
lands by prescription under the provision of existing laws Thus, in an application for land registration, the applicant has the
(NO) burden of overcoming the presumption that the State owns the land
applied for, and proving that the land has already been classified as
RULING: alienable and disposable. To overcome the presumption that the
● The requirements for judicial confirmation of imperfect title are
land belongs to the State, the applicant must prove by ​clear and
found in ​Section 14 o​f Presidential Decree No. 1529 (PD No.
1529), which provides: incontrovertible evidence at the time of application that the land
has been classified as alienable and disposable land of the public
Section 14. Who may apply. The following persons may domain.
file in the proper Court of First Instance an application for ● DISCUSSION ON THE CLASSIFICATION OF LANDS OF THE
registration of title to land, whether personally or through PUBLIC DOMAIN:
their duly authorized representatives: ○ Section 3 of Article XII classifies lands of the public
domain into
(1) Those who by themselves or through their ■ Agricultural
predecessors-in-interest have been in open, continuous, exclusive ■ forest or timber
and notorious possession and occupation of alienable and ■ mineral lands, and
disposable lands of the public domain under a bona fide claim of ■ national parks.
ownership since June 12, 1945, or earlier.
Of these four classifications, only agricultural lands may
(2) Those who have acquired ownership of private lands be alienated and disposed of by the State.
by prescription under the provision of existing laws.
○ How are the lands of public domain classified into
(3) Those who have acquired ownership of private lands agricultural lands, as well as their further classification
or abandoned river beds by right of accession or accretion under into alienable and disposable lands of the public domain?
the existing laws. Answer: LEGISLATIVE PREROGATIVE! ​This
prerogative has long been exercised through the
enactment of ​Commonwealth Act No. 141 (CA No. 141) been officially delimited and classified and,
or the ​Public Land Act of 1936. when practicable, surveyed, and which have not
been reserved for public or quasi-public uses, nor
○ Section 1827 of the Administrative Code of 1917 appropriated by the Government, nor in any
merely authorizes the Department Head to classify as manner become private property, nor those on
agricultural lands those forrest lands which are better which a private right authorized and recognized
adapted and more valuable for agricultural purposes. by this Act or any other valid law may be
NOTE: Sec. 1827 does not authorize the Department claimed, or which, having been reserved or
Head to classify agricultural lands ​as alienable and appropriated, have ceased to be so. ​However,
disposable lands. ​This power is expressly delegated by the President may, for reasons of public
the same Code solely to the ​Governor-General. interest, declare lands of the public domain
open to disposition before the same have had
○ At present, the existing Administrative Code is ​EO 292 ​or their boundaries established or been
the ​Administrative Code of 1987. ​The Admin Code of surveyed, or may, for the same reason,
1987 did not reenact Sec. 1827 of the Admin Code of suspend their concession or disposition until
1917. However, in the absence of incompatibility as to the they are again declared open to concession or
two statutes, Sec. 1827 has NOT been repealed. disposition by proclamation duly published or
by Act of the National Assembly.
○ There is nothing in Section 1827 that authorizes the
Department Head to classify agricultural lands into ○ Section 13 ​of the ​Revised Forestry Code of the
alienable or disposable lands of the public domain. Thus, Philippines of 1975, ​however, delegates discretionary
public lands classified as agricultural and used by the power to the DENR Secretary to classify as alienable and
Bureau of Plant Industry of the Department of Agriculture disposable forest lands of the public domain no longer
for plant research or plant propagation are not necessarily needed for forest reserves. He shall declare those
alienable and disposable lands of the public domain classified and determined not to be needed for forest
despite being classified as agricultural lands. purposes as alienable and disposable lands, the
administrative jurisdiction and management of which
○ For such agricultural lands to be alienable and shall be transferred to the Bureau of Lands: Provided,
disposable, there must be an ​express proclamation by That mangrove and other swamps not needed for shore
the President ​declaring such agricultural lands as protection and suitable for fishpond purposes shall be
alienable and disposable. released to, and be placed under the administrative
jurisdiction and management of the Bureau of Fisheries
○ Furthermore, agricultural land may still be reserved for and Aquatic Resources. Those still to be classified under
public or quasi-public purposes which would prohibit the present system shall continue to remain as part of the
the alienation or disposition of such land. CA 141 public forest.
provides: ○ Section 3, Article XII of the 1987 Philippine Constitution
states: "x x x. Alienable lands of the public domain shall
Section 8. Only those lands shall be declared be limited to agricultural lands. x x x." Thus, the
open to disposition or concession which have unclassified lands of the public domain, not needed for
forest reserve purposes, must first be declared agricultural accompanied by the original or certified true copy of the
lands of the public domain before the DENR Secretary original classification approved by the DENR Secretary
can declare them alienable and disposable. Under the or the President.
foregoing Section 13 of PD No. 705, the DENR Secretary
has no discretionary power to classify unclassified lands ● The CA found that Dumo and her predecessors-in-interest
of the public domain, not needed for forest reserve have been in possession of the land only from 1948, which is
purposes, into agricultural lands. However, the DENR the earliest date of the tax declaration presented.
Secretary can invoke his power under Section 1827 of the ● Thus, by admission, it is clear that she failed to prove her and her
Revised Administrative Code of 1917 to classify forest predecessors-in-interest’s possession and occupation of the land
lands into agricultural lands. Once so declared as for the duration required by law -- from June 12, 1945 or earlier.
agricultural lands of the public domain, the DENR ● Dumo cannot argue that she may be allowed to register due to
Secretary can then invoke his delegated power under prescription. Dumo not only failed to prove that the land
Section 13 of PD No. 705 to declare such agricultural sought to be registered is alienable and disposable, but also
lands as alienable and disposable lands of the public utterly failed to submit any evidence to establish that such land
domain. has been converted into patrimonial property by an express
○ As it is only the President or the DENR Secretary who declaration by the State.
may classify as alienable and disposable lands of the ● Section 14(2) of PD No. 1529 puts into operation the entire regime
public domain, an applicant for land registration must of prescription under the Civil Code, particularly Article 1113 in
prove that the land sought to be registered has been relation to Article 1137.55 Article 1113 provides that "[p]roperty
declared by the President or DENR Secretary as alienable of the State or any of its subdivisions ​not patrimonial in character
and disposable land of the public domain. shall not be the object of prescription​." Thus, it is clear that the
○ The applicant for land registration must prove that the land must be patrimonial before it may be susceptible of
DENR Secretary had approved the land classification and acquisitive prescription. Indeed, Section 14(2) of PD No. 1529
released the land of the public domain as alienable and provides that one may acquire ownership of private lands by
disposable, and that the land subject of the application for prescription.
registration falls within the approved area per verification ● Mere classification of agricultural land as alienable and disposable
through survey by the PENRO or CENRO. In addition, does not make such land patrimonial property of the State – an
the applicant for land registration must present a copy of express declaration by the State that such land is no longer
the original classification approved by the DENR intended for public use, public service or the development of
Secretary and certified as a true copy by the legal national wealth is imperative.
custodian of the official records. These facts must be ● Alienable and disposable lands of the public domain are those that
established to prove that the land is alienable and are to be disposed of to private individuals by sale or application,
disposable. because their disposition to private individuals is for the
○ Submitting a mere certification by the CENRO or development of the national wealth. However, until the lands are
PENRO with references to the original classification alienated or disposed of to private individuals, they remain
made by the President or the DENR Secretary is sorely "alienable lands of the public domain," as expressly classified by
inadequate since it has no probative value as a public the 1987 Philippine Constitution.
document to prove the alienable and disposable character ● Lands of the public domain become patrimonial property ​only
of the public land. The certification should always be when they are no longer intended for public use or public
service or the development of national wealth​. Articles 421 and
422 of the Civil Code expressly provide:
○ Article 421. All other property of the State, which is not
of the character stated in the preceding article, is
patrimonial property
○ Article 422. Property of public dominion, when no longer
intended for public use or for public service, shall form
part of the patrimonial property of the State.

WHEREFORE, the petition is ​DENIED​. The assailed decision and


resolution of the Court of Appeals are AFFIRMED.
58. REPUBLIC v. VEGA
question the fact of occupation, but that of the alienability of the land.
January 17,, 2011 | J. Sereno | Land Of Public Domain: When
They also contended that the testimony of the CENRO officer is
Alienable And Disposable
insufficient. It has been held in Jurisprudence that a ​CENRO certificate is
BARLONGAY & BARRALES
inadequate proof that the land is alienable​. ​There must also be a
certification from the Secretary of DENR that a lot was alienable and
PETITIONER: ​Republic of the Philippines disposable​ as ruled in Republic v. Tan.
RESPONDENT: ​Carlos R. Vega, Marcos R. Vega, Rogelio R. Vega,
Lubin R. Vega, Heirs Of Gloria R. Vega, Namely: Fracisco L. Yap, Ma. However, in light of a recent ruling in Republic v. Serrano, the CENRO
Winona Y. Rodriguez, Ma. Wendelyn V. Yap And Francisco V. Yap, Jr. certification is held to be substantial compliance to the needed proof. In
the present case, the Court, has nonetheless recognized and affirmed
RECIT-READY: ​Respondents sought to register a parcel of land, applications for land registration on other substantial and convincing
claiming that they inherited the same from their deceased mother. evidence duly presented without any opposition from the LRA or the
Respondent-intervenors Buhay claimed a portion of the lot in question DENR on the ground of ​substantial compliance.
and offered in evidence a Subdivision Plan which indicated the portion of
the subject land, which they claimed was sold to their Respondents here substantially complied.
predecessors-in-interest. On their part, Vegas presented as witness an
officer from CENRO who testified that the land in question is indeed ● First, respondents Vegas were able to present Mr. Gonzales of
alienable. the CENRO who testified that the subject land is alienable and
disposable.
The Republic, through the Office of the Solicitor General, opposed the ● Second, the Subdivision Plan formally offered as evidence by
claim and argued that Vegas failed to prove that the subject land was respondents-intervenors Buhays, ​expressly indicates that the
alienable and disposable, since the testimony of Mr. Gonzales did not land is alienable and disposable.
contain the date when the land was declared as such. The RTC ruled in ● Finally, upon being informed of respondents Vegas’ application
favor of the respondents and ordered titles to be issued in favor of Vega for original registration, the LRA never raised the issue that the
and Buhay. The Republic appealed the case to the Court of Appeals, land subject of registration was not alienable and disposable.
which affirmed the findings of the lower court. Hence, this petition.
It must be emphasized that the present ruling on substantial compliance
ISSUE: applies pro hac vice (this time only).

● W/N based on the evidence on record, respondents Vegas have DOCTRINE: ​To establish that the land subject of the application is
sufficiently established that the subject land is alienable and alienable and disposable public land, the general rule remains: all
disposable. ​Yes ​(But this case falls under the exception, not the applications for original registration under the Property Registration
general rule) Decree must include both (1) a CENRO or PENRO certification and (2) a
certified true copy of the original classification made by the DENR
RULING: ​The rule for registration of government land is that there must Secretary.
be open, continuous, exclusive and notorious possession and occupation
of alienable government land. The fact of occupation and that the land is
alienable government land must be proven. Here, the Republic does not
was no public land application filed for the same land by the
applicant or by any other person.
As an exception, however, the courts - in their sound discretion and based
● During the trial, respondents-intervenors Romea, Francisco,
solely on the evidence presented on record - may approve the application,
Orlando etc. all surnamed buhay (respondents-intervenors
pro hac vice, on the ground of ​substantial compliance. ​This exception
Buhays) entered their appearance and moved to intervene in
shall only apply to applications for registration currently pending before
respondents Vegas’ application for registration.
the trial court prior to this Decision and shall be inapplicable to all future
Respondents-intervenors Buhays claimed a portion of the subject
applications.
land , purportedly sold by respondents Vegas’ mother. ​They
likewise formally offered in evidence Subdivision Plan
Csd-04-024336-D, which indicated the portion of the subject land,
FACTS: which they claimed was sold to their predecessors-in-interest.
● The trial court granted respondents Vegas’ application and directed
● On 26 May 1995, respondents filed an application for the the Land Registration Authority (LRA) to issue the corresponding
registration of title. decree of registration in the name of respondents Vegas and
● The application covered a parcel of land in Los Baños, Laguna, respondents-intervenors Buhays’ predecessors, in proportion to
with a total area of 6,902 square meters their claims over the subject land.
● Respondents Vegas alleged that they inherited the subject land ● Petitioner Republic appealed the Decision of the trial court,
from their mother, Maria Revilleza Vda. de Vega, who in turn arguing that respondents Vegas failed to prove that the subject land
inherited it from her father, Lorenzo Revilleza. Their mother’s was alienable and disposable, since the testimony of Mr. Gonzales
siblings (two brothers and a sister) died intestate, all without did not contain the date when the land was declared as such.
leaving any offspring. ● The Court of Appeals affirmed in toto the earlier Decision of the
● On 21 June 1995, petitioner Republic filed an opposition to trial court. Aggrieved by the ruling, petitioner filed the instant Rule
respondents Vegas’ application for registration on the ground, inter 45 Petition with this Court.
alia, that the subject land or portions thereof were lands of the
public domain and, as such, not subject to private appropriation.
● During the trial court hearing on the application for registration, ISSUES:
respondents Vegas presented several exhibits in compliance with
the jurisdictional requirements, as well as witnesses to prove ● W/N based on the evidence on record, respondents Vegas have
respondents Vegas’ ownership, occupation and possession of the sufficiently established that the subject land is alienable and
land subject of the registration. disposable. ​Yes ​(This case falls under the exception, not the
● Significant was the testimony of Mr. Rodolfo Gonzales, a Special general rule)
Investigator of the Community Environment and Natural
Resources Office (CENRO) of Los Baños, Laguna, under the
Department of Environment and Natural Resources (DENR). He RATIO:
attested to having conducted an inspection of the subject land and ● Presidential Decree No. 1529, otherwise known as the Property
identified the corresponding Report dated 13 January 1997, which Registration Decree, provides for the instances when a person may
he had submitted to the Regional Executive Director, Region IV. file for an application for registration of title over a parcel of land.
The report stated that the area subject of the investigation was Applicants for registration of title must prove the following: (1)
entirely within the alienable and disposable zone, and that there
that the subject land forms part of the disposable and alienable ● If the stringent rule imposed in Republic v. T.A.N. Properties, Inc.
lands of the public domain; and (2) that they have been in open, is to be followed, the absence of these twin certifications justifies a
continuous, exclusive and notorious possession and occupation of denial of an application for registration. ​Significantly, however, the
the land under a bona fide claim of ownership since 12 June 1945 Court’s pronouncement in Republic v. T.A.N. Properties, Inc., was
or earlier. issued after the decision of the trial court and the appellate court ​in
● Raising no issue with respect to respondents Vegas’ open, this case.
continuous, exclusive and notorious possession of the subject land ● Recently, however, in Republic v. Serrano, ​the Court affirmed the
in the present Petition, the Court will limit its focus on the first findings of the trial and appellate courts that the parcel of land
requisite: specifically, whether it has sufficiently been subject of registration was alienable and disposable. The Court
demonstrated that the subject land is alienable and disposable. held that a DENR Regional Technical Director’s certification,
● To prove that the land subject of an application for registration is which is annotated on the subdivision plan submitted in evidence,
alienable, an applicant must conclusively establish the existence of constitutes substantial compliance with the legal requirement.
a positive act of the government, such as any of the following: a ● The ​DENR certification enjoys the presumption of regularity
presidential proclamation or an executive order; other absent any evidence to the contrary. ​It bears noting that no
administrative actions; investigation reports of the Bureau of Lands opposition was filed or registered by the Land Registration
investigator; or a legislative act or statute. ​The applicant may also Authority or the DENR to contest respondents' applications on the
secure a certification from the government that the lands applied ground that their respective shares of the lot are inalienable. There
for are alienable and disposable. being no substantive rights which stand to be prejudiced, the
● Previously, a certification from the DENR that a lot was alienable benefit of the Certification may thus be equitably extended in favor
and disposable was sufficient to establish the true nature and of respondents.
character of the property and enjoyed the presumption of regularity ● Indeed, the best proofs in registration proceedings that a land is
in the absence of contradictory evidence. alienable and disposable are a certification from the CENRO or
● However, in ​Republic v. T.A.N. Properties, Inc., the Supreme Provincial Environment and Natural Resources Office (PENRO)
Court overturned the grant by the lower courts of an original and a certified true copy of the DENR’s original classification of
application for registration over a parcel of land in Batangas and the land. The Court, however, has nonetheless recognized and
ruled that a CENRO certification is not enough to certify that a affirmed applications for land registration on other substantial and
land is alienable and disposable. convincing evidence duly presented without any opposition from
● As it now stands, aside from a CENRO certification, an the LRA or the DENR on the ground of ​substantial compliance.
application for original registration of title over a parcel of land ● Applying these precedents, the Court finds that despite the absence
must be accompanied by a copy of the ​original classification of a certification by the CENRO and a certified true copy of the
approved by the DENR Secretary ​and certified as a true copy by original classification by the DENR Secretary, there has been
the legal custodian of the official records in order to establish that substantial compliance with the requirement to show that the
the land indeed is alienable and disposable. subject land is indeed alienable and disposable based on the
● To comply with the first requisite for an application for original evidence on record.
registration of title under the Property Registration Decree, ● First, respondents Vegas were able to present Mr. Gonzales of the
respondents Vegas should have submitted a CENRO certification CENRO who testified that the subject land is alienable and
and a certified true copy of the original classification by the DENR disposable, and who identified his written report on his inspection
Secretary that the land is alienable and disposable, together with of the subject land.
their application.
● Second, Subdivision Plan Csd-04-02433-6, formally offered as the general rule remains: all applications for original registration
evidence by respondents-intervenors Buhays, ​expressly indicates under the Property Registration Decree must include both (1) a
that the land is alienable and disposable. Mr. Samson G. de Leon’s CENRO or PENRO certification and (2) a certified true copy of
(the officer-in-charge of the Office of the Assistant Regional the original classification made by the DENR Secretary.
Executive Director for Operations of the DENR) ​annotation ● As an exception, however, the courts - in their sound discretion
pertaining to the identification of the land as alienable and and based solely on the evidence presented on record - may
disposable coincides with the investigation report of Mr. Gonzales. approve the application, pro hac vice, on the ground of ​substantial
● Finally, upon being informed of respondents Vegas’ application compliance showing that there has been a positive act of
for original registration, the LRA never raised the issue that the government to show the nature and character of the land and an
land subject of registration was not alienable and disposable. ​In absence of effective opposition from the government. This
addition, not only did the government fail to cross-examine Mr. exception shall only apply to applications for registration currently
Gonzales, it likewise chose not to present any countervailing pending before the trial court prior to this Decision and shall be
evidence to support its opposition. In contrast to the other cases inapplicable to all future applications.
brought before this Court, ​no opposition was raised by any
interested government body, aside from the pro forma opposition WHEREFORE, premises considered, the instant Petition is DENIED.
filed by the OSG. The Court of Appeals’ Decision dated 30 April 2007 and the trial
● Petitioner Republic also assails the failure of Mr. Gonzales to court’s Decision dated 18 November 2003 are hereby AFFIRMED.
testify as to when the land was declared as alienable and
disposable. However, these matters could have been dealt with
extensively during cross-examination, which petitioner Republic
waived because of its repeated absences and failure to present
counter evidence. In any event, the Report, as well as the
Subdivision Plan, readily reveals that the subject land was certified
as alienable and disposable as early as 31 December 1925 and was
even classified as residential and commercial in nature.
● Thus, the Court finds that the evidence presented by respondents
Vegas, coupled with the absence of any countervailing evidence by
petitioner Republic, substantially establishes that the land applied
for is alienable and disposable and is the subject of original
registration proceedings under the Property Registration Decree.
● It must be emphasized that the present ruling on substantial
compliance applies pro hac vice (this time only). It does not in any
way detract from our rulings in Republic v. T.A.N. Properties, Inc.,
and similar cases which impose a strict requirement to prove that
the public land is alienable and disposable, especially in this case
when the Decisions of the lower court and the Court of Appeals
were rendered prior to these rulings. To establish that the land
subject of the application is alienable and disposable public land,
59.
60.
61.
62.
63. OPPEN v COMPAS
conferred by Section 2 of P.D. No. 1529. It expressly provides:
Oct 21, 2015 | Mendoza | Where to file
R. Dysico & Section 2. Nature of registration proceedings; jurisdiction of courts​.
Judicial proceedings for the registration of lands throughout the
PETITIONER: ​Ernesto Oppen Inc. Philippines shall be in rem and shall be based on the generally accepted
RESPONDENT: ​Alberto Compas principles underlying the Torrens system.

RECIT-READY: ​The case involves two (2) parcels of land in Las Piñas Courts of First Instance shall have exclusive jurisdiction over all
City. applications for original registration of title to lands, including
On August 10, 1987, pursuant to the writ of execution issued by the improvements and interests therein, and over ​all petitions filed after
MeTC, EOI annotated its lien on TCT No. S-100162. A certificate of sale original registration of title​, with power to hear and determine all
was issued in its favor on October 19, 1987, and entered on TCT No.
questions arising upon such applications or petitions. The court through
S-100612 on August 24, 1989. The said property was later sold in a
public auction where EOI was the highest bidder and the Final Deed of its clerk of court shall furnish the Land Registration Commission with
Sale, dated September 28, 1990, was issued after the lapse of the two certified copies of all pleadings, exhibits, orders, and decisions filed
redemption period. Subsequently, EOI filed for the cancellation of or issued in applications or petitions for land registration, with the
PMMSl's title and the issuance of a new one under EOI's name. Pursuant exception of stenographic notes, within five days from the filing or
to a writ of execution, dated December 9, 2003, TCT No. 95712 in the issuance thereof.
name of EOI, was issued cancelling on March 18, 2004 TCT No.
S-100612. From a perusal of the above-quoted provision, it is apparent that courts of
Meanwhile, on September 2, 2002, an alias writ of execution11 was first instance, now the RTC, have exclusive jurisdiction over registration
issued by MeTC- Branch 7 in connection with the case between PMMSI
proceedings, including petitions filed after the original registration of
and MBI. On November 8, 2002, the properties covered by TCT Nos.
S-100612 and S-100613 were sold in a public auction in which title.
respondent Alberto Compas (Compas) was the winning bidder and had
the sale annotated on both titles on November 11, 2002. The Final Deed DOCTRINE: ​Section 2 of P.D. No. 1529. It expressly provides:
of Sale12 was issued to Compas after PMMSI failed to redeem the said
properties during the redemption period which expired on November 11, Section 2. Nature of registration proceedings; jurisdiction of courts.
2003. Judicial proceedings for the registration of lands throughout the
Philippines shall be in rem and shall be based on the generally accepted
ISSUE: principles underlying the Torrens system.

● Whether the RTC has jurisdiction to hear the amended petition Courts of First Instance shall have exclusive jurisdiction over all
of EOI. (YES) applications for original registration of title to lands, including
improvements and interests therein, and over all petitions filed after
RULING: ​It is basic in law that the jurisdiction of courts is conferred by original registration of title, with power to hear and determine all
law.​[24] The jurisdiction of regional trial courts in land registration cases is questions arising upon such applications or petitions. The court through
PMMSI and MBI. On November 8, 2002, the properties covered
its clerk of court shall furnish the Land Registration Commission with
two certified copies of all pleadings, exhibits, orders, and decisions filed by TCT Nos. S-100612 and S-100613 were sold in a public auction
or issued in applications or petitions for land registration, with the in which respondent Alberto Compas (Compas) was the winning
exception of stenographic notes, within five days from the filing or bidder and had the sale annotated on both titles on November 11,
issuance thereof. 2002. The Final Deed of Sale was issued to Compas after PMMSI
failed to redeem the said properties during the redemption period
FACTS: which expired on November 11, 2003.

● The case involves two (2) parcels of land, each with an area of 11,
452 square meters, located in Las Piñas City, covered by Transfer ISSUES:
Certificate of Title (TCT) No. S-100612 and TCT No. S-100613, ● Whether the RTC has jurisdiction to hear the amended petition of
and previously registered in the name of Philippine Merchant EOI. (YES)
Marine School Inc. (P MMSI).
● On May 21, 1984, the said properties were levied upon pursuant to RELEVANT ARGUMENTS (if any):
the decision rendered, and the writ of execution issued,by the ● Petitioner:
Metropolitan Trial Court, Branch 7, Manila which approved the ● Respondent:
compromise agreement between Manufacturers Building, Inc.
(MBI) and PMMSI. Thereafter, the Notice of Levy in favor of RATIO:
MBI was annotated at the back of TCT Nos. S-100612 and
● It is basic in law that the jurisdiction of courts is conferred by law.
S100613 on August 22, 1986.
The jurisdiction of regional trial courts in land registration cases is
● On August 10, 1987, pursuant to the writ of execution issued by
conferred by Section 2 of P.D. No. 1529. It expressly provides:
the MeTC, EOI annotated its lien on TCT No. S-100162. A
● Section 2. Nature of registration proceedings; jurisdiction of
certificate of sale was issued in its favor on October 19, 1987, and
courts​. Judicial proceedings for the registration of lands throughout
entered on TCT No. S-100612 on August 24, 1989. The said
the Philippines shall be in rem and shall be based on the generally
property was later sold in a public auction where EOI was the
accepted principles underlying the Torrens system.
highest bidder and the Final Deed of Sale, dated September 28,
● Courts of First Instance shall have exclusive jurisdiction over all
1990, was issued after the lapse of the redemption period.
applications for original registration of title to lands, including
Subsequently, EOI filed for the cancellation of PMMSl's title and
improvements and interests therein, and over ​all petitions filed
the issuance of a new one under EOI's name. Pursuant to a writ of
after original registration of title​, with power to hear and
execution, dated December 9, 2003, TCT No. 95712 in the name
determine all questions arising upon such applications or petitions.
of EOI, was issued cancelling on March 18, 2004 TCT No.
The court through its clerk of court shall furnish the Land
S-100612.
Registration Commission with two certified copies of all
● Meanwhile, on September 2, 2002, an alias writ of execution was
pleadings, exhibits, orders, and decisions filed or issued in
issued by MeTC- Branch 7 in connection with the case between
applications or petitions for land registration, with the exception of considering that the case involved controversial issues. The parties
stenographic notes, within five days from the filing or issuance obviously lacked unanimity as EOI even filed a motion to dismiss
thereof. for failure to state a cause of action, claiming that its Torrens Title
● From a perusal of the above-quoted provision, it is apparent that was indefeasible and could not be collaterally attacked.
courts of first instance, now the RTC, have exclusive jurisdiction ● Even granting that Section 108 of P.D. No. 1529 was applicable,
over registration proceedings, including petitions filed after the EOI's second motion to dismiss should still be denied. The second
original registration of title. paragraph of Section 108 provides that all petitions or motions, as
● On the other hand, Section 108 states that: well as under any other provision of P.D. No. 1529 after original
registration, shall be filed in the original case in which the decree
Sec. 108. ​Amendment and alterations of certificates.​ - No erasure, or registration was made.
alteration, or amendment shall be made upon the registration book ● A closer scrutiny of Section 2 and Section 108 of P.D. No. 1529
after the entry of a certificate of title or of a memorandum thereon will show that the former pertains to the grant of jurisdiction to
and the attestation of the same by the Register of Deeds, except by regional trial courts while the latter refers to the venue where the
order of the proper Court of First Instance. action is to be instituted. EOI's second motion to dismiss was
supposed to be on the ground of lack of jurisdiction. It, however,
● All petitions or motions filed under this Section as well as any alleges that the petition should not have been filed with the RTC of
other provision of this Decree after original registration shall be Las Piñas under LRC Case No. LP-05-0089, but with the RTC
filed and entitled in the original case in which the decree or where the original title was filed and issued under LRC No.
registration was entered. N-1238. Based on the allegations thereof, it appeared that the
● The CA was correct in stating that EOI's reliance on Section 108 of second motion was invoking the ground of improper venue.
P.D. No. 1529 was misplaced. The appellate court aptly cited ● Granting it to be so, the second motion to dismiss was rightfully
Philippine Veteran's Bank v. Valenzuela​where the Court held that denied as EOI waived the ground of improper venue after it had
the prevailing rule was that proceedings under Section 108 were filed its first motion to dismiss pursuant to the Omnibus Motion
summary in nature, contemplating corrections or insertions of Rule. Section 8 of Rule 15 of the Revised Rules of Court provides
mistakes which were only ​clerical but certainly not controversial that a motion attacking a pleading, order, judgment or proceeding
issues. Relief under the said legal provision can only be granted if shall include all objections then available, and all objections not so
there is unanimity among the parties, or that there is no adverse included shall be deemed waived.
claim or serious objection on the part of any party in interest.Thus, ● In ​Spouses de Guzman v. Spouses Ochoa,​ a second motion to
the petition was properly filed with the RTC-Las Piñas where it dismiss on the ground of defective verification was denied
was docketed as LRC Case No. LP-05-0089, and not before the pursuant to the Omnibus Motion Rule. The Court held:
court which heard the original registration proceeding under LRC ● Section 8, Rule 15 of the Rules of Court defines an omnibus
No. N-1238, as the petition involved adversarial issues. motion as a motion attacking a pleading, judgment or proceeding.
● EOI cannot insist that the action should have been filed with the A motion to dismiss is an omnibus motion because it attacks a
RTC where the original registration was filed and issued pleading, that is the complaint. For this reason, a motion to
dismiss, like any other omnibus motion, must raise and include all
objections available at the time of the filing of the motion because
under Section 8, "all objections not so included shall be deemed
waived." As inferred from the provision, only the following
defenses Under Section 1, Rule 9, are excepted from its
application: [a] lack of jurisdiction over the subject matter; [b]
there is another action pending between the same parties for the
same cause (​litispendentia​); [c] the action is barred by prior
judgment (​res judicata)​ ; and the action is barred by the statute of
limitations or prescription.
● In the case at bench, the petitioners raised the ground of defective
verification and certification of forum shopping only when they
filed their second motion to dismiss, despite the fact that this
ground was existent and available at the time of the filing of their
first motion to dismiss. Absent any justifiable reason to explain
this fatal omission, the ground of defective verification and
certification of forum shopping was deemed waived and could no
longer be questioned by the petitioners in their second motion to
dismiss.
● Similar to the above-cited case, EOI erroneously filed a second
motion to dismiss raising improper venue as basis—one which is
susceptible of being waived—after the first motion to dismiss was
denied. EOI only insisted that the proper venue was the RTC
where the original case in which the decree or registration was
entered and not with the RTC Las Piñas after its first motion to
dismiss alleging the failure to state a cause of action was filed and
denied. Consequently, the ground of improper venue was deemed
waived and could no longer be questioned by EOI because the
issue on venue was not raised in its prior motion to dismiss.

WHEREFORE​, the petition is ​DENIED​.


63. Sps. Hwa Ping and Mary Gaw, et. al. v. Ayala Land, Inc.
vest title because registration is not a mode of acquiring ownership. A
July 26, 2017 | G.R. No. 173120 & 173141 | Mendoza | Priority in Time
certificate of title is merely an evidence of ownership or title over the
Made by: Kia Opinion
particular property described therein. Merely relying on the date of
registration of the original titles is insufficient because it is the surveys
Recit Ready​: ​A survey plan for a parcel of land was submitted to the therein that are being assailed.
General Land Registration Office for approval of the Director of Lands
by ​SPOUSES DIAZ ​in 1921​. Subsequently, the same land was again
FACTS:
submitted for subsequent surveys by various proponents, including
● The following survey plans ​of the same land ​were submitted to the
MAYUGA. ​In 1950 and 1958, an OCT was issued to Mayuga (OCT
General Land Registration Office for approval of the Director of Lands:
Nos. 242, 244, 1609). An OCT would be issued to Spouses Diaz only in
○ (1921) Petitioners Spouses Andres Diaz and Josefa Mia (Spouses
1970 (OCT No. 8510). Spouses Diaz would sell to ​SPOUSES YU​. The
Diaz) - Lot 1 - Sitio of Kay Monica, Barrio Pugad Lawin, Las
Mayuga lots would end up with Ayala Land Inc (​ALI​).
Pinas, Rizal
○ (1925) Dominador Mayuga - ​Lot 3 - Sitio May Kokek, Barrio
Issue:
Almanza, Las Pinas, Rizal
As between the two titles, who has the superior right to the parcel of land.
○ (1930) Guico - ​Lot 2 ​- Barrio Tindig na Mangga, Las Pinas, Rizal
The court ruled that SPOUSES YU had a superior right.
○ (1931) Yaptinchay - ​Lot 2 & Lot 3
Note: Despite being the same lot, the survey plans indicated different
Ratio:
addresses.
The court reiterated that between two conflicting titles, the general rule
● The following Original Certificate of Titles (OCT) were issued in favor
that the title registered earlier prevails IS NOT ABSOLUTE. A
of:
certificate of title is merely an evidence of ownership or title over the
○ (1950) Yaptinchay - ​Lot 2 & Lot 3 covered by OCT No. 242 and
particular property described therein. Merely relying on the date of
244
registration of the original titles is insufficient because ​it is the surveys
○ (1958) - Dominador Mayuga - ​Lot 3​ ​covered by OCT No. 1609
therein that are being assailed. The subsequent surveys contained
● Some of these properties were sold to ​CPJ Corporation
numerous and serious irregularities which cast doubt on the validity
of OCT Nos. 242, 244, 1609 ​(See irregularities in ratio)​.
DIAZ LOTS
● Andres Diaz filed a petition for original registration for Lot 1. In 1970,
Doctrine:
OCT No. 8510​ was issued in the name of Spouses Diaz.
General Assumption: ​Between two conflicting titles, the title registered
○ They subdivided the 460,626 sqm property into 10 lots and
first generally prevails.
conveyed to different third parties
However: ​The rule that between two conflicting titles, the title registered
● In 1976, Andres Diaz sold to Librado Cabautan the following parcels of
earlier prevails IS NOT ABSOLUTE. It reinforced the doctrine that
land:
registering a piece of land under the Torrens System does not create or
○ Lot 1-I covered by TCT No. 287416
○ Lot 1-B covered by TCT No. 287411 ● [YU CASE] ​When the Spouses Yu visited their lots, they were
○ Lot 1-A and Lot 1-D covered by TCT No. 287412 surprised to discover that ALI had already fenced the area and posted
● Spouses Yu Hwa Ping and Mary Gaw (Spouses Yu) acquired guards and they were prevented from entering/occupying the same.
ownership over 67,813 sqm representing the undivided half-portion They also discovered that the transfer certificates of titles covering
of Lot 1-A originating from OCT No. 8510 of Spouses Diaz. The parcels of land overlapping their claim were in the name of ALI. They
said property was co​-owned by Spouses Diaz with Spouses Librado filed a complaint against ALI for declaration of nullity of ALI’s TCTs.
and Susana Cabautan resulting from a civil case decided by the RTC of They also sought the recovery of possession of the property covered by
Makati on March 29, 1986. ALI's title which overlapped their land ​alleging that Spouses Diaz,
● (1994) Spouses Yu acquired ownership over Lot 1-B. TCT Nos. 39408 their predecessors, had open, uninterrupted and adverse possession
and 64549 were issued in their names. of the same from 1921 until it was transferred to Cabautan in 1976
and sold to them (Spouses Yu) in 1994.
DOMINADOR MAYUGA > CPJ LOTS ○ [RTC] ruled in favor of Spouses Yu. ALI’s OCTs were marred
● (1980) CPJ Corporation transferred their interest in the subject with numerous and blatant errors and that ALI did not offer any
properties to third persons. (1988) Ayala Corporation obtained the satisfactory explanation regarding the glaring discrepancies (For
subject properties from Goldenrod, Inc. and PESALA. your reference, the survey/title started with Dominador Mayuga >
● (1992) ALI acquired all the subject properties: Goldenrod, Inc. / PESALA > CPJ Corporation > Ayala Land Inc)
○ Lot 3 covered by TCT No. 41325 ○ [CA] ruled in favor of ALI. It held that Spouses Diaz committed
○ Lot 2, Lot 3, and Lot 6 covered by TCT No. 41261 fraud. It opined that Spouses Diaz knew of CPJ Corporation’s
interest over the subject land but failed to form it of their
● [DIAZ CASE] CPJ Corporation filed a Land Registration Case application. CA also ruled that Spouses Yu could no longer assert
against Spouses Diaz and other named respondents. It sought to that the titles of ALI were invalid because the one-year period to
review OCT No. 8510 in the names of Spouses Diaz on the ground contest the title had prescribed. ​CA ruled that OCTs Nos. 242,
that the interested persons were not notified of the application. 244, and 1609 (ALI’s titles) were issued in 1950 and 1958 while
○ RTC rendered a decision against Spouses Diaz, opining OCT No. 8510 (Spouses Yu’s title) was only issued in 1970
that the Spouses Diaz committed fraud when they filed ○ [CA MR] ​CA granted Spouses Yu and Spouses Diaz’ motion for
their application for original registration w/o informing reconsideration. Citing ​Guico v. San Pedro (​ Guico was one of the
the interested parties in violation of Act No. 496. It also individuals who caused the survey of the lot. See first batch of
held that Spouses Diaz knew that CPJ Corporation had an bullet points), the court noted that there were defects in the survey
appropriate interest over the subject property. of the lot. It also ruled that the ​doctrine that registration done
○ On appeal, CA held that Spouses Diaz had no obligation fraudulently is no registration at all prevails over the rules on
to inform CPJ Corporation and its successors about their equity.
registration because the original titles of the latter, from ○ [CA MR 2] CA reversed again and ruled in favor of ALI
which their transferred titles were derived, were based on noting that in ​Guico v. San Pedro​, the court did not
fraudulent surveys.
categorically declare that the land survey of Guico was invalid registered at a prior date, the Court did not allow the
and it even awarded some of the lots to the applicant. earlier registered title of the respondents to prevail
○ Aggrieved, Spouses Yu elevates the matter to the SC. because of the continuing possession of the petitioners
therein and the laches committed by the respondents.
ISSUE: Hence, the holder of an earlier registered title does not, in
1. Is the complaint of Spouses Yu barred by prescription? ​NO. all instances, absolutely triumph over a holder of a latter
2. Between the registered titles of Spouses Yu and ALI, which is registered title.
more superior? ​THE ​COURT RULED THAT SPOUSES YU’S d. In this case, the petitioners assail the numerous and
TITLE IS MORE SUPERIOR. serious defects in the surveys of OCT Nos. 242, 244 and
1609, which cast doubt on the inclusion of the subject
RATIO: lands in ALI’s titles. Accordingly, the Court must delve
1. [PRESCRIPTION ISSUE] While Section 38 of Act No. 496 states into the merits of their contentions to determine whether
that the petition for review to question a decree of registration must the subject properties are truly and genuinely included in
be filed within one (1) year after entry of the decree, such ALI’s title. ​Merely relying on the date of registration of
provision is not the only remedy of an aggrieved party who was the original titles is insufficient because it is the
deprived of land by fraudulent means. surveys therein that are being assailed.
2. The rule that between two conflicting titles, the title registered 3. The subsequent land surveys conducted (after 1921) contained
earlier prevails IS NOT ABSOLUTE. numerous and serious irregularities which cast doubt on the
a. If it can be clearly ascertained by the ordinary rules of validity of OCT Nos. 242, 244, 1609 which were transferred to
construction relating to written documents, that the ALI
inclusion of the land in the certificate of title of prior a. The survey conducted in favor of Spouses Diaz was
date is a mistake, the mistake may be rectified by conducted by a certain A.N. Feliciano. Curiously, the
holding the latter of the two certificates of title to be subsequent surveys for a certain Dominador Mayuga, a
conclusive. certain Guico, and for a certain Yaptinchay were also
b. It reinforced the doctrine that registering a piece of land conducted by A.N. Feliciano. It is dubious how the same
under the Torrens System does not create or vest title surveyor or agrimensor conducted the subsequent
because registration is not a mode of acquiring surveys. He should obviously be aware that a previous
ownership. A certificate of title is merely an evidence survey was already conducted. Furthermore, even if a
of ownership or title over the particular property single entity (that is, Feliciano) conducted the survey, that
described therein. lands were described to be located ​in different places​.
c. In ​Golloy v. Court of Appeals,​ there were two conflicting b. Other discrepancies include: Lack of signature of the
titles with overlapping boundaries. The first title was Director of Lands on some surveys, surveys referred to a
registered on March 1, 1918, while the second title was monument that only existed more than seven years after
registered on August 15, 1919. Despite having been the survey, many erasures as to total area of the property,
there was a difference in the intensity of the lower right
portion of one of the subsequent surveys which showed
that it may simply have been an attachment to the main
document, court ruled that irregularities existed in ​Guico
v. San Pedro
4. It was established that Andres Diaz was the very first claimant
of the subject property and was the proponent of the first
survey. The said survey clearly contained the signature of the
surveyor and the Director of Lands, as can be seen on its face.
It did not contain any erasure or alterations thereon. Likewise,
a duly authenticated copy of the survey is readily available in
the Bureau of Lands.

DISPOSITION:
WHEREFORE, the petitions are ​GRANTED​. The June 19, 2006 Decision
of the Court of Appeals in CA​G.R. CV Nos. 61593 & 70622 is hereby
REVERSED and SET ASIDE. The February 8, 2005 Amended Decision of
the Court of Appeals is hereby REINSTATED.

ASSENTING/DISSENTING OPINIONS:
NA
64.
65. UNIVERSITY OF THE PHILIPPINES v. ROSARIO
Segundina filed an omnibus motion to dismiss UP’s third cause of action
March 28, 2001 | Pardo, ​J.​ | PD 1529; Section 17: Survey of the Land
and cancel the notice of ​litis pendens annotated in the TCT. But RTC
K. LOPEZ DE LEON & CHECKER
denied the motion. MR was filed but it was again denied.

PETITIONER: ​University of the Philippines ISSUE​: W/N the OCT No. 17 was void ab initio on the ground that CFI
RESPONDENT: ​Segundina Rosario has no jurisdiction since the requisite signature approval of the Director
of Lands over the survey plan was nowhere to be found​ (YES)
RECIT-READY: S​ept 7, 1971, Datu Ditingke Ramos filed at CFI an
application for registration of title covering a parcel of land situated in RULING: No plan or survey may be admitted in land registration
Quezon City, with an area of 100,000 sqm and covered by Plan (LRC) proceedings until approved by the Director of Lands. The submission of
SWO-15055. UP filed an intervention claiming that the land covered by the plan is a statutory requirement of mandatory character. Unless a plan
the application is within its property described in TCT No. 9462. U.P. and its technical description are duly approved by the Director of Lands,
filed with the trial court an opposition and motion to dismiss Datu the same are of no value. Thus, the allegation that the signature approval
Ditingke Ramos' application. The CFI denied motion to dismiss since it for the survey plan was nowhere to be found is an important
did not encroach the land and on June 8, 1973 it granted the Application jurisdictional fact that must be ventilated before the trial court. Thus, as
for Registration of Ramos. CFI ordered the finality of decision and OCT No. 17 is void and Segundina traces her rights to OCT No. 17, her
ordered the LRC to comply with Sec. 21 of Act 2347. LRC issued Decree claim would have no basis as a spring cannot rise higher than its source.
No. N- 150604 in favor of Rosario Alcovendas Vda. de Ramos, and Further, the judgment in LRC Q-329 was subject to the qualification that
pursuant to which the RD of Quezon issued an OCT No. 17 in her name. "If the parcel of land is found to be inside decreed properties, this plan is
Due to some technical errors, RD of Quezon cancelled the OCT and automatically cancelled. Whether the land covered by OCT No. 17 is
issued TCT 223619 in the name of Ramos. February 23, 1988, Ramos inside decreed property is an issue of fact that can be best determined by
issued a Deed of Sale infavor of Segundina Rosario covering said parcel the trial court after an examination of the evidence. We find meritorious
in TCT. A fire razed the City hall (housing the RD) and destroyed the the trial court's rationale for denying Segundina's motion to dismiss. We
TCT 223619. It resulted to the reconstitution and issuance of TCT quote: "To establish their respective rights over the disputed property,
RT-78195 (223619). both plaintiff and respondents submitted documentary exhibits, the
genuineness and authenticity of which can only be proved in a full blown
UP filed for the cancellation of said TCT. Segundina caused the trial. "There is no pretense that the foregoing conflicting claims entail
registration of Deed of Absolute sale to the RD and it cancelled the determination of facts. It, thus, become imperative that both parties be
previousTCT and issued a new TCT in favor of Segundina. U.P. filed an given their day in Court to avoid the danger of committing a grave
amended petition alleging that it is "the true, absolute and registered injustice if they were denied an opportunity to introduce evidence in their
owner of a parcel of land covered by Transfer Certificate of Title No. behalf. "It is within this context that the Court considers it appropriate
9462" of the Register of Deeds of Quezon City and that the "unlawful under the present stage of the action to DENY the instant motion.
acts of ownership being exercised by (Segundina) and (Bugnay Pending final ruling on the merits of the case, Segundina's motion to
Construction and Development Corporation) as well as the existence of cancel the notice of lis pendens must be denied. Case is remanded to the
their spurious certificates of title, create a cloud of doubt on the title of trial court for trial on the merits.
(U.P.). UP also prayed that Transfer Certificate of Title No. 121042 or
the reconstituted titles or derivatives thereof be declared null and void ab DOCTRINE: ​The submission of the plan is a statutory requirement of
initio for being spurious and fraudulently issued (third cause of action) mandatory character. Unless a plan and its technical description are duly
● On Nov 21, 1976, the Register of Deeds of QC cancelled OCT No.
approved by the Director of Lands, the sam are of no value.
17 and issued TCT No. 223619 also in the name of Vda. de Ramos
P. D. No. 1529 requires the Director of Lands to sign and approve the
due to errors in the technical description
survey plan for the land applied for, otherwise, the title is void.
● On Feb 23, 1988, Vda. de Ramos executed a "deed of absolute
"Sec. 17. ​What and where to file - The application for land
sale" in favor of Segundina Rosario (Segundina) covering the land
registration shall be filed with the Court of First Instance of the province
embraced in TCT No. 223619.
or city where the land is situated. The applicant ​shall file together with
● On June 11, 1988, fire razed the QC Hall Building which housed
the application all original muniments of titles or copies thereof and ​a
the Office of the Register of Deeds of Quezon City.
survey plan approved by the Bureau of Lands​.
○ Transfer Certificate of Title No. 223619 was one of the
titles destroyed by the fire.
"The clerk of court ​shall not accept any application ​unless it is shown ● Segundina requested the Register to reconstitute TCT No. 223619
that the applicant has furnished the Director of Lands with a copy of the resulting in the issuance of Transfer Certificate of Title No.
application and all the annexes (emphasis ours). RT-78195 (223619).
● On March 11, 1993, U.P. filed with the RTC of QC a petition for
the cancellation of TCT No. (N-126671) 367316 naming
Segundina, Bugnay Construction and Development Corporation
FACTS: and the Register of Deeds of Quezon City, among others, as
● On Sept 7, 1971, Datu Ditingke Ramos filed with the CFI of respondents.
Quezon City an application for registration of title covering a ● Nov 10, 1994, Segundina registered the deed of absolute sale.so
parcel of land situated in QC, with an area of 100,000 square she was issued in her name TCT No. 121042 name, resulting in the
meters and covered by Plan (LRC) SWO-15055 cancellation of TCT No. RT-78195 (223619).
● Petitioner University of the Philippines (UP) filed with the trial ● On Nov 19, 1996, U.P. filed an amended petition alleging that it is
court a "motion for intervention" in the case, claiming that the land "the true, absolute and registered owner of a parcel of land covered
covered by the application (by Datu Ditingke Ramos) is within its by TCT No. 9462" of the Register of Deeds of QC and that the
property. "unlawful acts of ownership being exercised by Segundina and
● U.P. filed with the trial court an opposition and motion to dismiss Bugnay Construction as well as the existence of their spurious
Datu Ditingke Ramos' application for registration. certificates of title, create a cloud of doubt on the title of (U.P.)."
● On June 6, 1973, the trial court issued an order which reads as ● U.P. prayed that TCT No. 121042 or the reconstituted titles be
follows: declared null and void ​ab initio for being spurious and fraudulently
○ SWO-15055 does not encroach on the property of issued.
the UP and that it is not inside any decreed - ● On Dec 30, 1997, Segundina filed with the trial court an MR and
property, the motion to dismiss the application is motion to cancel the notice of ​lis pendens​, which was later on
hereby DENIED for lack of merit. denied.
● On May 8, 1974, the Commissioner of Land Registration issued ● On May 26, 1998, Segundina filed with the CA a petition for
Decree No. N-150604 in favor of Rosario Alcovendas Vda. de certiorari assailing the orders of the trial court denying her motion
Ramos, pursuant to which the Register of Deeds of QC issued to dismiss.
OCT No. 17 in her name. ● On Sept 18, 1998, the CA ruled in favor of Segundina, reasoned
that the third cause of action is barred by ​res judicata and that the
trial court committed grave abuse of discretion in denying trial court's rationale for denying Segundina's motion to dismiss.
Segundina's "motion to dismiss.” ● Thus, it became imperative that both parties be given their day in
● On Oct 26, 1998, U.P. filed with the CA, a motion for Court to avoid the danger of committing a grave injustice if they
reconsideration of the afore-quoted decision, which was later were denied an opportunity to introduce evidence in their behalf.
denied. ● "It is within this context that the Court considers it appropriate
under the present stage of the action to DENY the instant motion.
ISSUE: Pending final ruling on the merits of the case, Segundina's motion
to cancel the notice of lis pendens must be denied. Case is
● W/N the OCT No. 17 was void ab initio on the ground that CFI has
remanded to the trial court for trial on the merits.
no jurisdiction since the requisite signature approval of the
Director of Lands over the survey plan was nowhere to be found
(YES)

RULING:

● The trial court declared U.P. as having no interest in the land


covered by TCT No. 121042. However, UP's contention that OCT
No. 17 is void for lack of the requisite "signature approval of the
Director of Lands over the survey plan"is worth looking into.
● P. D. No. 1529 requires the Director of Lands to sign and
approve the survey plan for the land applied for, otherwise, the
title is ​void.​
● No plan or survey may be admitted in land registration proceedings
until approved by the Director of Lands. The submission of the
plan is a statutory requirement of mandatory character.
● Unless a plan and its technical description are duly approved by
the Director of Lands, the same are of no value.
● Thus, the allegation that the signature approval for the survey plan
was nowhere to be found is an important jurisdictional fact that
must be ventilated before the trial court.
● Thus, as OCT No. 17 is void and Segundina traces her rights to
OCT No. 17, her claim would have no basis as a spring cannot rise
higher than its source.
● Further, the judgment in LRC Q-329 was subject to the
qualification that "If the parcel of land is found to be inside
decreed properties, this plan is automatically cancelled.
● Whether the land covered by OCT No. 17 is inside decreed
property is an issue of fact that can be best determined by the trial
court after an examination of the evidence. We find meritorious the
66. DEL ROSARIO v. REPUBLIC
RULING: ​Del Rosario’s claim has no merit. Petitioner is duty bound to
June 6, 2002 | J. Mendoza | Section 17, PD 1529 - Survey of the Land
retrieve the tracing cloth plan from the LRA and to present it in evidence
LUCES & MIRANDA
in the trial court. Neither does the advance survey plan, which was
attached to petitioner's application and marked in evidence, su ffice to
PETITIONER: ​Angel Del Rosario comply with the requirement of the law. In this case, what was marked
RESPONDENT: ​Republic of the Philippines in evidence, the advance survey plan and the technical description, lacked
the necessary certification from the Bureau of Lands.
RECIT-READY: ​Angel Del Rosario filed an application for registration
of a parcel of land in Brgy. Pinagsanhan, Maragondon, Cavite. In his The submission in evidence of the original tracing cloth plan, duly
application, he stated that he and his predecessors-in-interest had been in approved by the Bureau of Lands, in cases for application of original
the open, continuous, exclusive, and notorious possession and occupation registration of land is a mandatory requirement. The reason for this rule
of the land in question, which was alienable and disposable land, under a is to establish the true identity of the land to ensure that it does not
bona de claim of ownership since the 1920s or even earlier; and that overlap a parcel of land or a portion thereof already covered by a
such land was being occupied and cultivated by him and his family. previous land registration, and to forestall the possibility that it will be
Petitioner Del Rosario annexed to his application SEVERAL documents overlapped by a subsequent registration of any adjoining land. The
(see facts). On the same day that he filed his application, petitioner also failure to comply with this requirement is fatal to petitioner's application
submitted to the Branch Clerk of Court, Atty. Jameswell Resus, the for registration.
original tracing cloth plan for the Lot. The clerk of court transmitted to
the Land Registration Authority (LRA) the duplicate copy of petitioner’s DOCTRINE: ​The submission in evidence of the original tracing cloth
application, the original tracing cloth plan, and all other documents plan, ​duly approved by the Bureau of Lands​, in cases for application of
submitted by petitioner. The trial court rendered its decision granting the original registration of land is a mandatory requirement. The reason for
application of petitioner. The Republic of the Philippines (respondent) this rule is to establish the true identity of the land to ensure that it does
appealed to the CA, putting in issue the failure of petitioner to submit in not overlap a parcel of land or a portion thereof already covered by a
evidence the original tracing cloth plan for Lot No. 1891 and to establish previous land registration, and to forestall the possibility that it will be
that he and his predecessors-in-interest had been in open, continuous, and overlapped by a subsequent registration of any adjoining land. The
notorious possession of the land applied for registration for the period failure to comply with this requirement is fatal to petitioner's application
required by law. CA reversed RTC’s decision. Hence, this petition. for registration.
Petitioner argues that the denial of his application because of his failure
to submit in evidence the original tracing cloth plan of Lot No. 1891 was
FACTS:
unjustified. He claims that he should not be faulted for such failure since
● Angel Del Rosario filed an application for registration of a parcel
he turned over the same to the trial court on the day he led his
of land in Brgy. Pinagsanhan, Maragondon, Cavite.
application, but it was submitted to the LRA by the branch clerk of court
● In his application, petitioner stated that he is a Filipino, married to
and could not be produced during the trial.
Agustina Catalasan, and a resident of Poblacion, Ternate, Cavite;
that he and his predecessors-in-interest had been in the open,
ISSUE:
continuous, exclusive, and notorious possession and occupation of
● W/N the denial of petitioner’s application for original
the land in question, which was alienable and disposable land,
registration was unjustified - ​NO
under a bona de claim of ownership since the 1920s or even
earlier; and that such land was being occupied and cultivated by
him and his family. Petitioner further alleged that there was no ● The trial court rendered its decision granting the application of
mortgage or encumbrance on the land; that the same was not petitioner.
bound by any public or private road or by any river or creek; and ● The Republic of the Philippines (respondent) appealed to the Court
that there was no person having any interest therein, legal or of Appeals, putting in issue the failure of petitioner to submit in
equitable, or having possession thereof other than himself. evidence the original tracing cloth plan for Lot No. 1891 and to
● Petitioner indicated the owners/ claimants/ occupants of the establish that he and his predecessors-in-interest had been in open,
adjoining properties, and annexed to his application the following continuous, and notorious possession of the land applied for
documents: (a) an advance survey plan of the land applied for with registration for the period required by law.
technical descriptions; (b) Technical Description of Lot No. 1891; ● The Court of Appeals rendered its decision reversing the decision
(c) Certi cation in lieu of Geodetic Engineer's certification issued of the trial court on the ground that petitioner indeed failed to
for registration purposes, attesting to the genuineness of the survey submit in evidence the original tracing cloth plan of the land
plan; (d) Certification that the subject land is alienable and applied for registration. Hence, this petition.
disposable; (e) Certification that the property is not covered by any
public land application or patent; (f) Tax Declaration covering the ISSUES:
parcel of land; and (g) Official Receipt showing petitioner's ● W/N the denial of petitioner’s application for original registration
payment of the realty taxes on the said lot up to 1997. was unjustified - ​NO
● On the same day he fi led his application, petitioner also
submitted to the Branch Clerk of Court, Atty. Jameswell M. RELEVANT ARGUMENTS (if any):
Resus, the original tracing cloth plan for Lot No. 1891. On ● Petitioner: Petitioner argues that the denial of his application
October 15, 1997, the clerk of court transmitted to the Land because of his failure to submit in evidence the original tracing
Registration Authority (LRA) the duplicate copy of petitioner's cloth plan of Lot No. 1891 was unjustified. He claims that he
application for registration of title of Lot No. 1891, the original should not be faulted for such failure since he turned over the same
tracing cloth plan, and the other documents submitted by to the trial court on the day he led his application, but it was
petitioner in support of his application. submitted to the LRA by the branch clerk of court and could not be
● In his testimony, petitioner reiterated the allegations in his produced during the trial.
application and identified the annexed documents. He claimed he
and his family planted in the subject lot mango and bamboo trees RATIO:
and raised animals on it. Petitioner testified that he inherited the ● The submission in evidence of the original tracing cloth plan, duly
land from his grandfather, who caused the survey of the said lot to approved by the Bureau of Lands, in cases for application of
be made in his name as the original claimant. He said that he original registration of land is a mandatory requirement. The
possessed the subject property from 1984, the time the cadastral reason for this rule is to establish the true identity of the land to
survey was made thereon, but also claimed that the rst survey on ensure that it does not overlap a parcel of land or a portion thereof
the land was made in 1930. Petitioner also stated that his already covered by a previous land registration, and to forestall the
predecessors-in-interest started cultivating the property in 1940, possibility that it will be overlapped by a subsequent registration of
planting kakawati trees along its boundaries. He claimed that he any adjoining land. The failure to comply with this requirement is
and his family alone were the ones who gathered the fruits and fatal to petitioner's application for registration.
forest products of the land and that no one had ever disturbed his ● Petitioner contends, however, that he had submitted the original
possession over the lot or questioned his ownership of the same. tracing cloth plan to the branch clerk of court, but the latter
submitted the same to the LRA. This claim has no merit. Petitioner
is duty bound to retrieve the tracing cloth plan from the LRA and of ownership. For him, possession is not exclusive and notorious
to present it in evidence in the trial court. so as to give rise to a presumptive grant from the state. The
● Neither does the advance survey plan, which was attached to possession of public land, however long the period thereof may
petitioner's application and marked in evidence, suffice to comply have, extended, never confers title thereto upon the possessor
with the requirement of the law. Although in one case it was ruled because the statute of limitations with regard to public land does
that a mere blueprint copy of the cloth plan, together with the lot's not operate against the state, unless the occupant can prove
technical description, was su cient to identify the land applied for possession and occupation of the same under claim of ownership
registration, both the blueprint copy and the technical description for the required number of years.
were certified as to their correctness by the Director of Lands. In ● Although petitioner claims that he possessed Lot No. 1891 by
this case, what was marked in evidence, the advance survey plan himself and through his predecessors-in-interest since the 1930s,
and the technical description, lacked the necessary certification his tax declaration and tax payment receipt belie the same. The
from the Bureau of Lands. taxes due on the land subject of the application covered only a
● Moreover, petitioner failed to establish that he and his period of 10 years beginning 1988 and was paid only on
predecessors-in-interest had met the legal requirements as to the September 9, 1997, or a little more than a month prior to the filing
nature and length of possession leading to a registrable title over of the application. There is no other tax declaration or receipt for
the land. Petitioner claims that he and his family cultivated the tax payments by petitioner's predecessors-in-interest. Moreover,
subject land, without the help of tenants, in order to plant bamboo tax declarations and receipts are not conclusive evidence of
and mango trees thereon. His witness also testified that the land ownership but are merely indicia of a claim of ownership.
was for a time planted with coconut trees and palay. However, ● Hence, in view of the lack of suffcient evidence of the 30-year
from the testimonies of petitioner and his witness, it appears that open, notorious, and conclusive possession in the concept of an
petitioner is a businessman who, while born in Maragondon, owner, as required by C.A. No. 141, §48 (b), as amended,
Cavite, has actually been a resident of Poblacion, Ternate, Cavite petitioner's application for original registration of Lot No. 1891
from childhood until the present. Moreover, it appears that the land cannot be granted.
was only planted with bamboo trees, which do not require much
tending to. There is also doubt as to how many mango trees, if any, WHEREFORE, the decision of the Court of Appeals denying the
existed on the land or to the volume of fruits harvested from these application of petitioner Angel del Rosario for original registration of
trees, since there was no testimony to that effect and the tax Lot No. 1891, Cadastral 457-D, Maragondon, Cavite, Ap-04-0011601, is
declaration offered in evidence stated that the improvements found AFFIRMED.
on the land were only bamboo trees.
● Raymundo Telia testified he remembered that there existed on the
land some coconut trees, but these were no longer there at the time
of his testimony. He also testified that the land was planted with
palay, but not by petitioner or his predecessors or his family but by
kaingeros, including himself, who only asked permission from
petitioner to use the land. Assuming that petitioner had planted the
bamboo and mango trees thereon, this fact would hardly su ce to
prove possession as it would constitute "a mere casual cultivation"
of that large tract of land. A mere casual cultivation of portions of
the land by the claimant does not constitute possession under claim
67.REPUBLIC OF THE PHILIPPINES v. SARMIENTO
been declared alienable? ​NO
March 13, 2007 | J. Carpio-Morales | Sec. 17. Survey of the Land
J. Miranda &
RULING: ​This proof is not sufficient. Section 2, Article XII of the 1987
Constitution, provides: "All lands of the public domain, waters, minerals,
PETITIONER: ​Republic of the Philippines coal, petroleum, and other mineral oils, all forces of potential energy,
RESPONDENT: ​Restituto Sarmiento, represented by his fisheries, forests or timber, wildlife, flora and fauna, and other natural
Attorney-In-Fact, Magdaleno Sarmiento resources are owned by the State. . . ."

RECIT-READY: ​Respondent Restituto Sarmiento through his In the present case, petitioners cite a surveyor-geodetic engineer's
brother-attorney-in-fact Magdaleno Sarmiento filed on November 29, notation indicating that the survey was inside alienable and disposable
2000 with the MeTC of Taguig, Metro Manila an application for land. Such notation does not constitute a positive government act validly
registration of a parcel of land with a total land area of 2,664 square changing the classification of the land in question. By relying solely on
meters and located at Barangay Wawa, Taguig, Metro Manila. He the said surveyor's assertion, petitioners have not sufficiently proven that
claimed to have acquired the lot through donation under a Kasulatan ng the land in question has been declared alienable..
Pagkakaloob dated July 16, 1988 executed by his father, Placido
Sarmiento, who in turn inherited it from his mother Florentina
Sarmiento. Respondent added that he and his predecessors-in-interest DOCTRINE: ​It is well settled that no public land can be acquired by
have been in open, continuous, uninterrupted, adverse, and public private persons without any grant, express or implied, from the
possession of the lot in the concept of an owner for more than 30 years. government, and it is indispensable that the person claiming title to
Respondent’s proof is a blue print copy (photocopy) of the conversion public land should show that his title was acquired from the State or any
and subdivision plan approved by the DENR Center. The Solicitor other mode of acquisition recognized by law. Verily, a mere surveyor has
General opposed the application stating that the possession claim was not no authority to reclassify lands of the public domain.
true, that the tax declarations and tax payment receipts do not appear to
be genuine, that the claim of ownership in fee simple on the basis of a For the original registration of title, the applicant must overcome the
Spanish title or grant can no longer be availed of by respondent as he presumption that the land sought to be registered forms part of the public
failed to file an appropriate application for registration within six months domain. Unless public land is shown to have been reclassified or
from February 16, 1976, as required under P.D. No. 892, and that the lot alienated to a private person by the State, it remains part of the
is part of the public domain not subject to private appropriation. The inalienable public domain. Indeed, "occupation thereof in the concept of
MeTCt ruled for Sarmiento and ordered the decree of registration to be owner, no matter how long, cannot ripen into ownership and be
issued. The Republic appealed to the CA, but the CA affirmed the lower registered as a title." To overcome such presumption, incontrovertible
court’s decision, hence the appeal to the SC. evidence must be shown by the applicant. Absent such evidence, the land
sought to be registered remains inalienable.
ISSUE:
● Whether or not the blue print copy of the conversion and
FACTS:
subdivision plan approved by the DENR Center which bears the
● Respondent Restituto Sarmiento through his
notation of the surveyor-geodetic engineer that "this survey is
brother-attorney-in-fact Magdaleno Sarmiento filed on November
inside the alienable and disposable area, certified by the Bureau
29, 2000 with the MeTC of Taguig, Metro Manila an application
of Forestry." is a sufficient proof that the land in question has
for registration of a parcel of land, delineated as Lot 535-D under
Approved Survey Plan Swo-13-000465 with a total land area of RATIO:
2,664 square meters and located at Barangay Wawa, Taguig, Metro ● It is well settled that no public land can be acquired by private
Manila (the lot). persons without any grant, express or implied, from the
● Respondent claimed to have acquired the lot through donation government, and it is indispensable that the person claiming title to
under a Kasulatan ng Pagkakaloob dated July 16, 1988 executed public land should show that his title was acquired from the State
by his father, Placido Sarmiento, which lot formed part of Lot 535 or any other mode of acquisition recognized by law.
that was allegedly inherited by Placido from Florentina Sarmiento. ● To prove that the land in question formed part of the alienable and
● Respondent further claimed that he and his predecessors-in-interest disposable lands of the public domain, petitioners relied on the
have been in open, continuous, uninterrupted, adverse, and public printed words which read: "This survey plan is inside Alienable
possession of the lot in the concept of an owner for more than 30 and Disposable Land Area, Project No. 27-B as per L.C. Map No.
years. 2623, certified by the Bureau of Forestry on January 3, 1968,"
● To discharge the onus, respondent relies on the blue print copy of appearing on Exhibit "E" (Survey Plan No. Swo-13-000227).This
the conversion and subdivision plan approved by the DENR Center proof is not sufficient. For the original registration of title, the
which bears the notation of the surveyor-geodetic engineer that applicant (petitioners in this case) must overcome the presumption
"this survey is inside the alienable and disposable area, Project No. that the land sought to be registered forms part of the public
27-B. L.C. Map No. 2623, certified on January 3, 1968 by the domain. Unless public land is shown to have been reclassified or
Bureau of Forestry." alienated to a private person by the State, it remains part of the
● On January 17, 2001, the Solicitor General, through the Prosecutor inalienable public domain. Indeed, "occupation thereof in the
of Taguig who was deputized to assist in the case, filed, as counsel concept of owner, no matter how long, cannot ripen into ownership
for the Republic of the Philippines (petitioner), an Opposition to and be registered as a title." To overcome such presumption,
respondent’s application for registration. However the MeTC incontrovertible evidence must be shown by the applicant. Absent
ordered the issuance of a decree of registration for the said land in such evidence, the land sought to be registered remains inalienable.
favor of the respondent. ● In the present case, petitioners cite a surveyor-geodetic engineer's
● By Decision of May 20, 2005, the appellate court held that as the notation in Exhibit "E" indicating that the survey was inside
lot was sufficiently identified by the blue print copy of the plan and alienable and disposable land. Such notation does not constitute a
the technical description, the presentation of the original tracing positive government act validly changing the classification of the
cloth ceased to become indispensable for the grant of the land in question. Verily, a mere surveyor has no authority to
application. The appellate court thus affirmed the decision of the reclassify lands of the public domain. By relying solely on the said
MeTC. Petitioner’s motion for reconsideration having been denied surveyor's assertion, petitioners have not sufficiently proven that
by Resolution 28 of August 19, 2005, petitioner appealed to the the land in question has been declared alienable.
SC. ● Even assuming that respondent has proven that the lot is alienable,
his application would still be denied for failure to comply with the
ISSUES: period of possession requirement.
● Whether or not the blue print copy of the conversion and ● To this Court, Tax Declaration No. 9631-Exhibit "N-4" does not
subdivision plan approved by the DENR Center which bears the constitute competent proof of Placido’s title over Lot 535. For one,
notation of the surveyor-geodetic engineer that "this survey is respondent failed to prove that Placido is an heir of Florentina. For
inside the alienable and disposable area, certified by the Bureau of another, respondent failed to prove the metes and bounds of the
Forestry." is a sufficient proof that the land in question has been "palayero" allegedly owned by Florentina and that the lot actually
declared alienable? ​NO forms part thereof.
● But even assuming arguendo that, as found by the MeTC, Placido
was an heir and inherited Lot 535 from Florentina, respondent still
failed to provide proof, nay allege, that Florentina possessed Lot
535 since June 12, 1945 or earlier under a bona fide claim of
ownership

WHEREFORE, the petition is GRANTED. The Decision and


resolution of the Court of Appeals dated May 20, 2005 and August 19,
2005, respectively, are REVERSED and SET ASIDE. The application
for registration filed by respondent, Restituto Sarmiento, over Lot
535-D, with a total area of Two Thousand Six Hundred Sixty Four
(2,664) square meters situated at Barangay Wawa, Taguig, Metro
Manila is DENIED
WHEREFORE, start here.

68. BENIN v. TUASON


Date | J. Zaldivar | Topic: Sec. 19 Amendments of Boundaries or Area
CHECKER & MAKER

PETITIONER: ​Victor Benin


RESPONDENT: ​Mariano Severo Tuason

RECIT-READY: ​Start here.

ISSUE:
● W/N? ​YES/NO
● W/N? Y​ ES/NO

RULING: ​Start here.

DOCTRINE: ​Start here.

FACTS:
● Fact #1
● Fact #2

ISSUES:
● Issue #1
● Issue #2

RELEVANT ARGUMENTS (if any):


● Petitioner:
● Respondent:

RATIO:
● Ratio #1
● Ratio #2
69. REPUBLIC v. SANTOS III. FACTS:
November 12, 2012 |Bersamin, J. | Sec. 19 Amendments of Boundaries
or Area ● Arcadio Ivan A. Santos III and Arcadio C. Santos, Jr., are the
NOEL & CHECKER owners of the land subject of this application which was previously
a part of the Parañaque River which became an orchard after it
dried up.
PETITIONER: Republic of The Philippines ● Alleging continuous and adverse possession of more than ten
RESPONDENT: Arcadio Ivan A. Santos III, Arcadio C. Santos Jr.
years, respondent Arcadio Ivan Santos III applied on March 7,
RECIT-READY: ​Arcadio Ivan A. Santos III and Arcadio C. Santos, Jr.,
1997 for the registration of Lot which was bounded in the
are the owners of the land subject of this application which was
previously a part of the Parañaque River which became an orchard after it Northeast by Lot 4079 belonging to respondent Arcadio C. Santos,
dried up. They are alleging ​that the property had been formed through Jr. in the Southeast by the Parañaque River, in the Southwest by an
accretion and had been in their joint open, notorious, public, abandoned road, and in the Northwest by another lot also owned
continuous and adverse possession for more than 30 years. ​The City by Arcadio Ivan.
of Parañaque is opposing it arguing that their property was not formed ● Arcadio Ivan amended his application for land registration to
through accretion, but was the result of a part of the Parañaque river include Arcadio, Jr. as his co-applicant because of the latter’s
drying up. The RTC and CA granted Santos ownership
co-ownership of the property. He alleged that the property had
been formed through accretion and had been in their joint
ISSUE:
open, notorious, public, continuous and adverse possession for
● Whether or not the land in question can be acquired through more than 30 years.
accretion. (NO) ● The City of Parañaque opposed the application for land
registration, stating that it needed the property for its flood control
RULING: ​By law, accretion - the gradual and imperceptible deposit program, and that the property was within the legal easement of 20
made through the effects of the current of the water- belongs to the owner meters from the river bank. Also, assuming that the property was
of the land adjacent to the banks of rivers where it forms. ​The drying up not covered by the legal easement, title to the property could not be
of the river is not accretion. ​Hence, the dried-up river bed belongs to
registered in favor of Santos for the reason that the property was an
the State as property of public dominion, not to the riparian owner,
orchard that had dried up and had not resulted from accretion.
unless a law vests the ownership in some other person. ​Also, since it is
property of the State it cannot be acquired through prescription unless the ● The RTC granted the application for land registration declaring
same had been declared alienable and disposable. Arcadio III and Arcadio Jr., both as the TRUE and ABSOLUTE
OWNERS of the land being applied for which is situated in the
DOCTRINE: ​Start here. Barangay of San Dionisio, City of Parañaque City.
● The CA affirmed. Both lower courts used Art. 457 of the Civil
Code as their basis in awarding the land to Santos. Civil Code
Article 457- To the owners of the lands adjoining the bank of
rivers belong the accretion which they gradually receive from the ● Under the Regalian doctrine, all lands not otherwise appearing to
effects of the current of the waters." be clearly within private ownership are presumed to belong to the
State. No public land can be acquired by private persons without
ISSUE/S: any grant, express or implied, from the Government. It is
indispensable, therefore, that there is a showing of a title from the
State.
● Whether or not the land in question can be acquired through
accretion. (NO)
WHEREFORE, the Court REVERSES and SETS ASIDE the decision
RATIO: of the Court of Appeals promulgated on May 27, 2003; DISMISSES the
● ​By law, accretion - the gradual and imperceptible deposit made application for registration of Arcadio C. Santos, Jr. and Arcadio Ivan
through the effects of the current of the water- belongs to the S. Santos III respecting Lot 4998-B with a total area of 1,045 square
owner of the land adjacent to the banks of rivers where it forms.
meters, more or less, situated in Barangay San Dionisio, Parañaque
The drying up of the river is not accretion. ​Hence, the dried-up
river bed belongs to the State as property of public dominion, City, Metro Manila; and DECLARES Lot 4998-B as exclusively
not to the riparian owner, unless a law vests the ownership in belonging to the State for being part of the dried--up bed of the
some other person. Parañaque River.
● Accretion is the process whereby the soil is deposited along the
banks of rivers. ​The deposit of soil, to be considered accretion,
must be:

(a) gradual and imperceptible;

(b) made through the effects of the current of the water; and

(c) taking place on land adjacent to the banks of rivers.

● Respondents Santos should establish the concurrence of the


elements of accretion to warrant the grant of their application for
land registration, and they failed to do so.

● Even conceding, for the sake of argument, that respondents


possessed Lot 4998-B for more than thirty years in the character
they claimed, ​they did not thereby acquire the land by
prescription or by other means without any competent proof
that the land was already declared as alienable and disposable
by the Government. Absent that declaration, the land still
belonged to the State as part of its public dominion.
70. DREAM VILLAGE NEIGHBORHOOD ASSOCIATION, INC. v.
area declared in Proclamation Nos. 2476 and 172 as alienable and
BASES CONVERSION DEVELOPMENT AUTHORITY
disposable.
July 24, 2013 | J. Reyes | Amendments of Boundaries or Area
OPINION &
It was noted that while some portions of Fort Bonifacio have been
declared “alienable and disposable”, the said lands did not thereby
PETITIONER: ​Dream Village Neighborhood Association, Inc., become patrimonial, since the BCDA law (RA 7227) makes the express
represented by its Incumbent President, Greg Seriego reservation that they are to be sold in order to raise funds for the
RESPONDENT: ​Bases Conversion Development Authority conversion of the former American bases in Clark and Subic.

RECIT-READY: ​The present petition by Dream Village concerns a The property sought to be registered must not only be classified as
dispute over a lot in present-day Fort Bonifacio against BCDA. Dream alienable and disposable, it must also be expressly declared by the State
Village claims to represent more than 2,000 families who have been that it is no longer intended for public service or the development of the
occupying a 78,466-square meter lot in Western Bicutan, Taguig City national wealth, or that the property has been converted into patrimonial.
since 1985 “in the concept of owners continuously, exclusively and Absent such an express declaration by the State, the land remains to
notoriously.” It argues that according to Proclamation No. 172, such area be property of public dominion.
was made alienable and disposable. On the other hand, BCDA asserts
that its title to the subject property is pursuant to R.A. No. 7227. It is the DOCTRINE: ​While property of the State or any of its subdivisions
holder of patrimonial government property. patrimonial in character may be the object of prescription, ​those
“intended for some public service or for the development of the
ISSUE: national wealth” are considered property of public dominion and
● W/N the portion of Lot 1 of subdivision Plan SWO-13-000298, therefore not susceptible to acquisition by prescription.
situated in the barrio of Western Bicutan, Taguig, Metro Manila,
which is presently being occupied by Dream Village is within
FACTS:
the coverage of Proclamation Nos. 2476 and 172 and outside the
● Petitioner Dream Village Neighborhood Association, Inc. (Dream
claim of BCDA? ​NO. The BCDA holds title to Fort Bonifacio
Village) claims to represent more than 2,000 families who have
by virtue of RA 7227.
been occupying a 78,466-square meter lot in Western Bicutan,
● W/N area occupied by Dream Village is susceptible of
Taguig City since 1985 “in the concept of owners continuously,
acquisition by prescription? ​NO. Not while it is intended for
exclusively and notoriously.”
some public service or for the development of the national
● The lot can be traced back to Dolores Casal y Ochoa, owner of
wealth.
Hacienda de Maricaban ​(covers parts of present-day Makati,
Pasig, Taguig, Pasay, Paranaque). Maricaban was purchased by the
RULING: ​The court ruled that the BCDA has title to Fort Bonifacio, a
US government, who transferred 30 hectares to the Manila
matter which has long been decided with finality in ​Samahan ng Masang
Railroad Company. The US formally ceded Fort William Mckinley
Pilipino sa Makati, Inc. v. BCDA​. It is unequivocal that the Philippine
to the Republic of the Philippines.
Government, and now the BCDA, has title and ownership over Fort
● President Carlos P. Garcia issued Proclamation No. 423
Bonifacio.
withdrawing from sale or settlement the tracts of land within Fort
William Mckinley, now renamed ​Fort Bonifacio​, and reserving
Dream Village sits on the abandoned C-5 Road, which lies outside the
them for military purposes.
● President Ferdinand E. Marcos issued Proclamation No. 2476 the boundaries and technical description of these exempt
declaring certain portions of Fort Bonifacio alienable and areas shall be determined by an actual ground survey.
disposable in the manner provided under Republic Act (R.A.) Nos. ● Conflict arose when Dream Village and its members were
274 and 730, in relation to the Public Land Act, thus allowing the subjected to summary demolition by BCDA, resulting in unrest
sale to the settlers of home lots in Upper Bicutan, Lower Bicutan, and tensions among the residents ​over the subject 78,466-sq m
Signal Village, and Western Bicutan. property, which they claimed is within Lot 1 of Swo-13-000298
● President Corazon C. Aquino issued Proclamation No. 172 and thus is covered by Proclamation No. 172.
amending Proclamation No. 2476 by limiting to Lots 1 and 2 of ● A relocation/verification survey was conducted over the subject
the survey Swo-13-000298 the areas in Western Bicutan open lot. The Commission on the Settlement of Land Problems
for disposition. (COSLAP) received the report and resolved that the lot lies outside
● In 1992, R.A. No. 7227 was passed creating the Bases Conversion of BCDA, and thus directed the LMB of the DENR to process the
and Development Authority (BCDA) to oversee and accelerate the applications of Dream Village’s members for sales patent, noting
conversion of Clark and Subic military reservations and their that in view of the length of time that they “have been openly,
extension camps (John Hay Station, Wallace Air Station, continuously and notoriously occupying the subject property in the
O’Donnell Transmitter Station, San Miguel Naval concept of an owner, x x x they are qualified to apply for sales
Communications Station and Capas Relay Station) to productive patent on their respective occupied lots pursuant to R.A. Nos. 274
civilian uses. and 730 in relation to the provisions of the Public Land Act.”
○ Section 8 provides that the capital of the BCDA will be ● CA ruled that COSLAP has no jurisdiction.
provided from sales proceeds or transfers of lots in nine
(9) military camps in Metro Manila, including 723 has. of ISSUES:
Fort Bonifacio. The law, thus, expressly authorized the ● W/N the portion of Lot 1 of subdivision Plan SWO-13-000298,
President of the Philippines “to sell the above lands, in situated in the barrio of Western Bicutan, Taguig, Metro Manila,
whole or in part, which are hereby declared alienable and which is presently being occupied by Dream Village is within the
disposable pursuant to the provisions of existing laws and coverage of Proclamation Nos. 2476 and 172 and outside the claim
regulations governing sales of government properties,” of BCDA? ​NO. The BCDA holds title to Fort Bonifacio.
specifically to raise capital for the BCDA. Titles to the ● W/N area occupied by Dream Village is susceptible of acquisition
camps were transferred to the BCDA for this purpose by prescription? ​NO. Not while it is intended for some public
○ Excepted from disposition by the BCDA are: a) service or for the development of the national wealth.
approximately 148.80 has. reserved for the National
Capital Region (NCR) Security Brigade, Philippine Army RELEVANT ARGUMENTS (if any):
officers’ housing area, and Philippine National Police jails ● Petitioner:
and support services (presently known as Camp Bagong ○ They claim the portion of of Lot 1 of subdivision Plan
Diwa); b) approximately 99.91 has. in Villamor Air Base SWO-13-000298, situated in the barrio of Western
for the Presidential Airlift Wing, one squadron of Bicutan, Taguig, Metro Manila.
helicopters for the NCR and respective security units; c) ○ They claim that they have been occupying the area for
twenty one (21) areas segregated by various presidential thirty (30) years “in the concept of owners continuously,
proclamations; and d) a proposed 30.15 has. as relocation exclusively and notoriously for several years,” and have
site for families to be affected by the construction of built their houses of sturdy materials thereon and
Circumferential Road 5 and Radial Road 4, provided that introduced paved roads, drainage and recreational and
religious facilities. The lot is not among those transferred ○ The mere fact that the original plan for C-5 Road to
to the BCDA under R.A. No. 7227. cross Swo-00-0001302 was abandoned by deviating it
● Respondent: northward to traverse the southern part of Libingan ng
○ Its title to the subject property is pursuant to R.A. No. mga Bayani ​does not signify abandonment by the
7227. ​It is the holder of patrimonial government government of the bypassed lots, nor that these lots
property. would then become alienable and disposable
● In ​Heirs of Mario Malabanan v. Republic,​ it was pointed out that
RATIO: from the moment R.A. No. 7227 was enacted, the subject military
● That the BCDA has title to Fort Bonifacio has long been decided lands in Metro Manila became alienable and disposable. ​However,
with finality. In ​Samahan ng Masang Pilipino sa Makati, Inc. v. it was also clarified that the said lands did not thereby become
BCDA​, it was categorically ruled as follows: patrimonial, since the Bases Conversion and Development
○ First, it is unequivocal that the Philippine Authority (BCDA) law makes the express reservation that they
Government, and now the BCDA, has title and are to be sold in order to raise funds for the conversion of the
ownership over Fort Bonifacio. The case of Acting former American bases in Clark and Subic.
Registrars of Land Titles and Deeds of Pasay City, Pasig ○ The Court noted that the purpose of the law can be tied to
and Makati is final and conclusive on the ownership of either “public service” or “the development of national
the then ​Hacienda de Maricaban e​ state by the Republic wealth” under Article 420(2) of the Civil Code, such that
of the Philippines. Clearly, the issue on the ownership of the lands remain the property of the public dominion,
the subject lands in Fort Bonifacio is laid to rest. albeit their status is now alienable and disposable​. The
○ The facts in ​Samahan ng Masang Pilipino sa Makati are Court then explained that it is only upon their sale to a
essentially not much different from the controversy private person or entity as authorized by the BCDA law
below. There, 20,000 families were long-time residents that they become private property and cease to be
occupying 98 has. of Fort Bonifacio in Makati City, who property of the public dominion: For as long as the
vainly sought to avert their eviction and the demolition of property belongs to the State, although already classified
their houses by the BCDA upon a claim that the land was as alienable or disposable, it remains the property of the
owned by the USA. The SC ruled that their TCT has been public dominion when it is “intended for some public
cancelled in the name of the Republic and transferred to service or for the development of the national wealth.”
BCDA. ​The Court ruled that the BCDA’s aforesaid ● While property of the State or any of its subdivisions patrimonial
titles over Fort Bonifacio are valid, indefeasible and in character may be the object of prescription, ​those “intended for
beyond question pursuant to an explicit authority some public service or for the development of the national
under R.A. No. 7227. wealth” are considered property of public dominion and
● Dream Village sits on the abandoned C-5 Road, which lies outside therefore not susceptible to acquisition by prescription. ​(Article
the area declared in Proclamation Nos. 2476 and 172 as alienable 420, par. 2 of the Civil Code)
and disposable. ○ The property sought to be registered must not only be
○ Dream Village is not situated in Lot 1 of Swo-13-000298 classified as alienable and disposable, it must also be
but actually occupies Lots 10, 11 and part of 13 of expressly declared by the State that it is no longer
Swo-00-0001302, part of the abandoned right-of-way of intended for public service or the development of the
C-5 Road. national wealth, or that the property has been converted
into patrimonial. ​Absent such an express declaration by
the State, the land remains to be property of public
dominion.
● Dream Village has been unable to dispute BCDA’s claim that Lots
10, 11 and part of 13 of Swo-00-0001302 are the abandoned
right-of-way of C-5 Road, which is within the vast titled territory
of Fort Bonifacio. ​We have already established that these lots
have not been declared alienable and disposable under
Proclamation Nos. 2476 or 172. Moreover, it is a settled rule
that lands under a Torrens title cannot be acquired by
prescription or adverse possession.

WHEREFORE, premises considered, the petition is DENIED.


71. GENEROSA MENDOZA v. CA
the issuance of the decree of title.
July 14, 1978 | J. Santos | Sec. 22 Dealings with Land Pending
Registration DOCTRINE: T​he law expressly allows the land subject matter of an
F. QUIJANO & N. SABBAN application for registration to be "dealt with", i.e., to be disposed of or
encumbered during the interval of time between the filing of the
application and the issuance of the decree of title, and to have the
PETITIONER: ​Generosa Mendoza, substituted by his wife and instruments embodying such disposition or encumbrance presented to the
administratrix Diega De Leon Vda. De Mendoza registration court by the “interested party" for the court to either "order
RESPONDENT: ​Hon. Court of Appeals, Daniel Gole Cruz and Dolores such land registered subject to the encumbrance created by said
Mendoza instruments, ​or order the decree of registration issued in the name of the
buyer or of the person to whom the property has been conveyed by said
RECIT-READY: ​Generoso Mendoza, filed for an application for the instruments.​
registration of two parcels of land. It was proven that he and his wife,
Diega de Leon, were the owners of the parcels of land subject of the
application but the same were sold by them, during the pendency of the FACTS:
case, to the spouses Daniel Gole Cruz and Dolores Mendoza, subject to ● Generoso Mendoza, filed with the Court of First Instance of
the vendors' usufructuary rights. After the decision had become final, Bulacan an application for the registration of two parcels of land,
Generoso Mendoza, filed a motion for the issuance of the decree. Decree with a residential house thereon, situated in the Poblacion of Sta.
No. 114454 was issued confirming the title to the land of vendees Daniel
Maria Bulacan.
Gole Cruz and Dolores Mendoza, and ordering the registration of the
same in their names. Generoso Mendoza filed an urgent petition for ● From the evidence presented by Generoso Mendoza, it was proven
reconsideration praying that the decision and the decree issued pursuant that he and his wife, Diega de Leon, were the owners of the parcels
thereto be set aside and that Original Certificate of Title No. 03787 be of land subject of the application but the same were sold by them,
cancelled, on the ground that the vendees, the registered owners, had during the pendency of the case, to the spouses Daniel Gole Cruz
failed to pay the purchase price of the lands. and Dolores Mendoza, subject to the vendors' usufructuary rights.
The instrument embodying such sale was presented as Exhibit 1.
ISSUE: On the basis of such evidence, the registration court rendered a
● W/N the registration court could legally order the registration of
decision on July 21, 1965, ordering the registration of the two
the land in the names of the vendees-respondents, who were
parcels of land in the names of the vendees, Daniel Gole Cruz and
neither the applicants nor the oppositors in the registration case
Dolores Mendoza, subject to the usufructuary rights of the
below - ​YES
vendors, Generoso Mendoza and Diega de Leon.
● After the decision had become final, Generoso Mendoza, filed a
RULING: ​Petitioner overlooks Section 29 of the Land Registration Act
which expressly authorizes the registration of the land subject matter of a motion for the issuance of the decree. On May 16, 1967, Decree
registration proceeding in the name of the buyer or of the person to No. 114454 was issued confirming the title to the land of vendees
whom the land has been conveyed by an instrument executed during the Daniel Gole Cruz and Dolores Mendoza, and ordering the
interval of time between the filing of the application for registration and registration of the same in their names, subject to the usufructuary
rights of the vendors. Consequently, Original Certificate of Title
No. 0-3787 was issued to spouses Daniel Gole Cruz and Dolores property has been conveyed" for the applicant. Neither does it
Mendoza. require that the "buyer" or the "person to whom the property has
● Generoso Mendoza filed an urgent petition for reconsideration been conveyed" be a party to the case. He may thus be a total
praying that the decision dated July 21, 1965 and the decree issued stranger to the land registration proceedings. The only
pursuant thereto dated May 16, 1967 be set aside and that Original requirements of the law are: (1) that the instrument be presented to
Certificate of Title No. 03787 be cancelled, on the ground that the the court by the interested party together with a motion that the
vendees, the registered owners, had failed to pay the purchase price same be considered in relation with the application; and (2) that
of the lands. prior notice be given to the parties to the case.
● T​he instrument embodying the sale of the subject property by the
ISSUES: petitioner to the private respondents was duly presented to the
● W/N the registration court could legally order the registration of registration court for consideration. That the purpose was to have
the land in the names of the vendees-respondents, who were the land registered in the names of private respondents subject to
neither the applicants nor the oppositors in the registration case the usufructuary rights of petitioner and his wife is explicit in the
below - ​YES following facts and circumstances. Firstly, it was the petitioner
himself, the applicant in the registration proceedings, who
RATIO: presented the deed of sale (Exh. I) to the court and testified before
● Petitioner overlooks Section 29 of the Land Registration Act which the same that he did sell the land to the private respondents. This
expressly authorizes the registration of the land subject matter of a was done by him despite the fact that he could easily have the land
registration proceeding in the name of the buyer or of the person to registered in his name — as an order of general default had been
whom the land has been conveyed by an instrument executed issued and the hearing on the application for registration had been
during the interval of time between the filing of the application for conducted EX-PARTE. Secondly, as if to fully convince the court
registration and the issuance of the decree of title of the fact of sale, petitioner presented his wife, Diega de Leon,
● It is clear from the provision that the law expressly allows the land and private respondent, Daniel GolE Cruz, to confirm the said sale
subject matter of an application for registration to be "dealt with", of the land and the stipulated usufructuary rights. Finally, the
i.e., to be disposed of or encumbered during the interval of time petitioner even filed the motion for the issuance of the decree of
between the filing of the application and the issuance of the decree confirmation of title after having received the decision of the court
of title, and to have the instruments embodying such disposition or ordering the registration of the title to the land in the names of
encumbrance presented to the registration court by the ,interested vendees-respondents, subject to the stipulated usufructuary rights
party" for the court to either "order such land registered subject to — thereby signifying his full assent to the same.
the encumbrance created by said instruments, ​or order the decree
of registration issued in the name of the buyer or of the person to WHEREFORE ​the decision of the Court of Appeals dated February 17,
whom the property has been conveyed by said instruments.​ The 1973 is hereby affirmed with costs against petitioner.
law does not require that the application for registration be
amended by substituting the "buyer" or the person to whom the
72. Director of Lands v. CA and Abistado
There was failure to comply with the explicit publication
July 28, 1997 | J. Panganiban | Notice of Initial Hearing requirement of the law. Thus, the application for land registration
N. SABBAN & J. SALAMAT filed by the Abistados must be dismissed without prejudice to
reapplication in the future, after all the legal requisites shall have been
PETITIONER: ​Director of Lands duly complied with.
RESPONDENT: ​Court of Appeals and Teodoro Abistado, substituted
by Margarita, Marissa, Maribel, Arnold, and Mary Ann Abistado DOCTRINE:

RECIT-READY: Upon receipt of the order of the court setting the time for initial hearing,
Teodoro Abistado filed a petition for registration of title over a parcel of the Commissioner of Land Registration shall cause a notice of the initial
land. During the pendency of the petition, Abistado died, hence, he is hearing to be published ​once in the Official Gazette and once in a
now represented by his heirs. The RTC dismissed the petition and ruled newspaper of general circulation in the Philippines​.
that applicants failed to comply with the provisions of Section 23 of PD
1529, ​requiring the Applicants to publish the notice of Initial Hearing in a Notice shall be addressed to all persons appearing to have an interest in
newspaper of general circulation in the Philippines​. In this case, notice the land involved including the adjoining owners so far as known. It shall
was only published in the Official Gazette. also require all persons concerned to appear in court at a certain date and
time.
ISSUE: ​Whether newspaper publication of the notice of initial hearing in
an original land registration case mandatory? ​(YES)
FACTS:
RULING: ● In December 1986, Teodoro Abistado filed a petition for
registration of his title over 648 sqm of land under PD 1529.
Land registration is a proceeding ​in rem,​ which is validated During the pendency of the petition, Abistado died. Hence, his
essentially through publication​. The ​process must strictly be heirs were substituted as applicants.
complied with. Otherwise, persons who may be interested or whose ● The land registration court dismissed the petition for want of
rights may be adversely affected would be barred from contesting an jurisdiction. However, it found that the applicants through their
application which they had no knowledge of. predecessors-in-interest had been in open, continuous, exclusive
and peaceful possession of the subject land since 1938.
Official Gazette is not as widely read and circulated as newspapers and is ● RTC: Dismissed the petition. Ruled that applicants failed to
oftentimes delayed in its circulation, such that the notices published comply with the provisions of Section 23 of PD 1529, ​requiring the
therein may not reach the interested parties on time. In sum, the Applicants to publish the notice of Initial Hearing in a newspaper
all-encompassing ​in rem nature of land registration cases, the of general circulation in the Philippines​. Notice was only published
consequences of default orders issued against the whole world and the in the Official Gazette.
objective of disseminating the notice in as wide a manner as possible ● CA: Set aside the decision of RTC and ordered the registration of
demand a mandatory construction of the requirements for publication, the title in the name of Teodoro Abistado.
mailing and posting.
Petitioner’s Contention​: Under Section 23 of PD 1529, the notice of initial ● In ​Republic vs. Marasigan, ​the Court through Mr. Justice Hilario
hearing shall be published ​both in the Official Gazette ​and in a newspaper G. Davide, Jr. held that Section 23 of PD 1529 requires notice of
of general circulation. the initial hearing by means of (1) publication, (2) mailing and (3)
Respondent’s Contention: Failure to comply with the requirement of posting, all of which must be complied with.
publication in a newspaper of general circulation is a mere procedural ● Land registration is a proceeding ​in rem;​ such proceeding
defect; publication in the Official Gazette is sufficient. requires constructive seizure of the land as against ​all persons,
including the state, who have rights to or interests in the
ISSUE: ​Whether newspaper publication of the notice of initial hearing in property.
an original land registration case mandatory? ​(YES) ● An ​in rem proceeding is validated essentially through
publication​. The ​process must strictly be complied with.
RULING: Otherwise, persons who may be interested or whose rights may be
adversely affected would be barred from contesting an application
● Newspaper Publication Mandatory which they had no knowledge of.
○ The pertinent part of Section 23 of Presidential Decree ● Official Gazette is not as widely read and circulated as newspapers
No. 1529 requiring publication of the notice of initial and is oftentimes delayed in its circulation, such that the notices
hearing reads as follows: published therein may not reach the interested parties on time, if at
■ Sec. 23. ​Notice of initial hearing, publication, all. In sum, the all-encompassing ​in rem nature of land registration
etc. -- The court shall, within five days from cases, the consequences of default orders issued against the whole
filing of the application, issue an order setting world and the objective of disseminating the notice in as wide a
the date and hour of the initial hearing which manner as possible demand a mandatory construction of the
shall not be earlier than forty-five days nor later requirements for publication, mailing and posting.
than ninety days from the date of the order. ● There was failure to comply with the explicit publication
■ The public shall be given notice of initial hearing requirement of the law. Thus, the application for land
of the application for land registration by means registration filed by the Abistados must be dismissed without
of (1) publication; (2) mailing; and (3) posting. prejudice to reapplication in the future, after all the legal requisites
● Upon receipt of the order of the court setting the time for initial shall have been duly complied with.
hearing, the Commissioner of Land Registration shall cause a
notice of the initial hearing to be published ​once in the Official WHEREFORE, the petition is ​GRANTED and the assailed Decision
Gazette and once in a newspaper of general circulation in the and Resolution are ​REVERSED ​and ​SET ASIDE​. The application of
Philippines​. private respondent for land registration is ​DISMISSED w ​ ithout
● Notice shall be addressed to all persons appearing to have an prejudice. No costs.
interest in the land involved including the adjoining owners so far
as known. It shall also require all persons concerned to appear in
court at a certain date and time.
● Publication in the Official Gazette is enough merely to confer
jurisdiction upon the land registration court. However, the question
boils down to ​whether, absent any publication in a newspaper
of general circulation, the land registration court can validly
confirm and register the title of private respondents​.
73. REPUBLIC OF THE PHILIPPINES v MANNA PROPERTIES, FACTS:
INC. (​SUPRA BUT EDITED​) ● Manna Properties filed an Application for the registration of title of
January 31, 2005 | CARPIO, J | PD 1529 Notice of Initial Hearing two (2) parcels of land
H SANCHEZ ● Initial hearing was set on February 16, 1995
● But, the full names and complete postal addresses of all adjoining
lot owners were not stated for notification purposes so they were
PETITIONER: ​Republic of the PH
RESPONDENT: ​Manna Properties Inc, Represented by its President required to be submitted first and the initial hearing was reset to
Jose Tanyao April 13, 1995.
● The Land Registration Authority requested for the resetting of the
RECIT-READY: ​Manna wants to register titles over 2 parcels of land. initial hearing since April 13, 1995 fell on Holy Thursday, a
But, the OSG, representing the Republic, said the applicant is a private non-working day. So, it was reset to April 20, 1995
corporation disqualified under the new Philippine Constitution to hold ● the court a quo received a letter dated March 6, 1995 from the
alienable lands of public domain. They’re also saying Manna failed to LRA with the information that the notice can no longer be
comply with PD 1529’s initial hearing rule for original registrations since
published in the Official Gazette for lack of material time since the
the initial hearing date of the application kept being delayed (From feb
16, 1995 to july 18 1995) National Printing Office required submission of the printing
materials 75 days before the date of the hearing. It was again
requested that the initial hearing be moved to a date. So it was
ISSUE: moved to March 15, 1995
● W/N Manna Properties Failed to Comply with the Jurisdictional ● Per Certificate of Publication issued by the LRA and the National
Requirements for Original Registration due to non-compliance Printing Office, the Notice of Initial Hearing was published in the
with PD 1529’s notice of initial hearing rule? - NO June 12, 1995 issue of the Official Gazette officially released on
June 19, 1995. The same notice was published in the July 12, 1995
RULING: ​The delays in the initial hearing was not the fault of Manna. issue of the The Ilocos Herald.
Sec 23 of PD 1529 is complied with since the records show, the notice of ● But the trial court set the hearing date itself on 18 July 1995.
hearing was published both in the Official Gazette and a newspaper of
general circulation well ahead of the date of hearing. This complies with
the legal requirement of serving the entire world with sufficient notice of
the registration proceedings. ISSUES:
● W/N Manna Properties Failed to Comply with the Jurisdictional
DOCTRINE: ​A party to an action has no control over the Administrator Requirements for Original Registration due to non-compliance
or the Clerk of Court acting as a land court; he has no right to meddle with PD 1529’s notice of initial hearing rule? - NO
unduly with the business of such official in the performance of his duties.
IF the 90-day notice is not followed without fault of the applicant, it can’t RELEVANT ARGUMENTS (if any):
be taken against him.
Petitioner is contending:
○ That PD 1529 sets a 90-day maximum period between the nothing in the records indicates that Manna Properties failed to
court order setting the initial hearing date and the hearing perform the acts required of it by law.
itself. ● We have held that "a party to an action has no control over the
○ Petitioner points out that in this case, the trial court issued Administrator or the Clerk of Court acting as a land court; he has
the order setting the date of the initial hearing on 15 no right to meddle unduly with the business of such official in the
March 1995, but the trial court set the hearing date performance of his duties."
itself on 18 July 1995​. Considering that there are 125 ● It is unfair to punish an applicant for an act or omission over which
days in between the two dates, petitioner argues that the the applicant has neither responsibility nor control, especially if the
trial court exceeded the 90-day period set by PD 1529. applicant has complied with all the requirements of the law.
Thus, petitioner concludes "the applicant [Manna ● Sec 23 of PD 1529 is complied with since the records show, the
Properties] failed to comply with the jurisdictional notice of hearing was published both in the Official Gazette and a
requirements for original registration." newspaper of general circulation well ahead of the date of hearing.
● This complies with the legal requirement of serving the entire
RATIO: world with sufficient notice of the registration proceedings.
● PD 1529 Sec. 23. ​Notice of initial hearing, publication etc. - The
court shall, within five days from filing of the application, issue an WHEREFORE, we GRANT the instant petition. We REVERSE the
order setting the date and hour of initial hearing which shall not Decision of the Court of Appeals dated 20 December 2000 in CA-G.R.
be earlier than forty-five days nor later than ninety days from the CV No. 52562. The Application for Registration filed by Manna
date of the order. Properties, Inc. over Lots No. 9515 and 1006 of Cad. 539-D, with a total
● The duty and the power to set the hearing date lies with the land area of One Thousand Four Hundred Eighty (1,480) square meters
registration court. situated in Barangay Pagdaraoan, San Fernando, La Union, is
● After an applicant has filed his application, the law requires the DENIED.
issuance of a court order setting the initial hearing date. The notice
of initial hearing is a court document.
● The notice of initial hearing is signed by the judge and copy of the
notice is mailed by the clerk of court to the LRA. This involves a
process to which the party applicant absolutely has no
participation.
● Manna Properties was not at fault why the hearing date was set
beyond the 90-day maximum period.
● The Docket Division of the LRA repeatedly requested the trial
court to reset the initial hearing date because of printing problems
with the National Printing Office, which could affect the timely
publication of the notice of hearing in the Official Gazette. Indeed,
74.
75.
76. Director of Lands v. CA and Abistado
There was failure to comply with the explicit publication
July 28, 1997 | J. Panganiban | SEC. 23 - Notice of Initial Hearing requirement of the law. Thus, the application for land registration
N. SABBAN & J. SALAMAT filed by the Abistados must be dismissed without prejudice to
reapplication in the future, after all the legal requisites shall have been
PETITIONER: ​Director of Lands duly complied with.
RESPONDENT: ​Court of Appeals and Teodoro Abistado, substituted
by Margarita, Marissa, Maribel, Arnold, and Mary Ann Abistado DOCTRINE:

RECIT-READY: Upon receipt of the order of the court setting the time for initial hearing,
Teodoro Abistado filed a petition for registration of title over a parcel of the Commissioner of Land Registration shall cause a notice of the initial
land. During the pendency of the petition, Abistado died, hence, he is hearing to be published ​once in the Official Gazette and once in a
now represented by his heirs. The RTC dismissed the petition and ruled newspaper of general circulation in the Philippines​.
that applicants failed to comply with the provisions of Section 23 of PD
1529, ​requiring the Applicants to publish the notice of Initial Hearing in a Notice shall be addressed to all persons appearing to have an interest in
newspaper of general circulation in the Philippines​. In this case, notice the land involved including the adjoining owners so far as known. It shall
was only published in the Official Gazette. also require all persons concerned to appear in court at a certain date and
time.
ISSUE: ​Whether newspaper publication of the notice of initial hearing in
an original land registration case mandatory? ​(YES)
FACTS:
RULING: ● In December 1986, Teodoro Abistado filed a petition for
registration of his title over 648 sqm of land under PD 1529.
Land registration is a proceeding ​in rem,​ which is validated During the pendency of the petition, Abistado died. Hence, his
essentially through publication​. The ​process must strictly be heirs were substituted as applicants.
complied with. Otherwise, persons who may be interested or whose ● The land registration court dismissed the petition for want of
rights may be adversely affected would be barred from contesting an jurisdiction. However, it found that the applicants through their
application which they had no knowledge of. predecessors-in-interest had been in open, continuous, exclusive
and peaceful possession of the subject land since 1938.
Official Gazette is not as widely read and circulated as newspapers and is ● RTC: Dismissed the petition. Ruled that applicants failed to
oftentimes delayed in its circulation, such that the notices published comply with the provisions of Section 23 of PD 1529, ​requiring the
therein may not reach the interested parties on time. In sum, the Applicants to publish the notice of Initial Hearing in a newspaper
all-encompassing ​in rem nature of land registration cases, the of general circulation in the Philippines​. Notice was only published
consequences of default orders issued against the whole world and the in the Official Gazette.
objective of disseminating the notice in as wide a manner as possible ● CA: Set aside the decision of RTC and ordered the registration of
demand a mandatory construction of the requirements for publication, the title in the name of Teodoro Abistado.
mailing and posting.
Petitioner’s Contention​: Under Section 23 of PD 1529, the notice of initial ● In ​Republic vs. Marasigan, ​the Court through Mr. Justice Hilario
hearing shall be published ​both in the Official Gazette ​and in a newspaper G. Davide, Jr. held that Section 23 of PD 1529 requires notice of
of general circulation. the initial hearing by means of (1) publication, (2) mailing and (3)
Respondent’s Contention: Failure to comply with the requirement of posting, all of which must be complied with.
publication in a newspaper of general circulation is a mere procedural ● Land registration is a proceeding ​in rem;​ such proceeding
defect; publication in the Official Gazette is sufficient. requires constructive seizure of the land as against ​all persons,
including the state, who have rights to or interests in the
ISSUE: ​Whether newspaper publication of the notice of initial hearing in property.
an original land registration case mandatory? ​(YES) ● An ​in rem proceeding is validated essentially through
publication​. The ​process must strictly be complied with.
RULING: Otherwise, persons who may be interested or whose rights may be
adversely affected would be barred from contesting an application
● Newspaper Publication Mandatory which they had no knowledge of.
○ The pertinent part of Section 23 of Presidential Decree ● Official Gazette is not as widely read and circulated as newspapers
No. 1529 requiring publication of the notice of initial and is oftentimes delayed in its circulation, such that the notices
hearing reads as follows: published therein may not reach the interested parties on time, if at
■ Sec. 23. ​Notice of initial hearing, publication, all. In sum, the all-encompassing ​in rem nature of land registration
etc. -- The court shall, within five days from cases, the consequences of default orders issued against the whole
filing of the application, issue an order setting world and the objective of disseminating the notice in as wide a
the date and hour of the initial hearing which manner as possible demand a mandatory construction of the
shall not be earlier than forty-five days nor later requirements for publication, mailing and posting.
than ninety days from the date of the order. ● There was failure to comply with the explicit publication
■ The public shall be given notice of initial hearing requirement of the law. Thus, the application for land
of the application for land registration by means registration filed by the Abistados must be dismissed without
of (1) publication; (2) mailing; and (3) posting. prejudice to reapplication in the future, after all the legal requisites
● Upon receipt of the order of the court setting the time for initial shall have been duly complied with.
hearing, the Commissioner of Land Registration shall cause a
notice of the initial hearing to be published ​once in the Official WHEREFORE, the petition is ​GRANTED and the assailed Decision
Gazette and once in a newspaper of general circulation in the and Resolution are ​REVERSED ​and ​SET ASIDE​. The application of
Philippines​. private respondent for land registration is ​DISMISSED w ​ ithout
● Notice shall be addressed to all persons appearing to have an prejudice. No costs.
interest in the land involved including the adjoining owners so far
as known. It shall also require all persons concerned to appear in
court at a certain date and time.
● Publication in the Official Gazette is enough merely to confer
jurisdiction upon the land registration court. However, the question
boils down to ​whether, absent any publication in a newspaper
of general circulation, the land registration court can validly
confirm and register the title of private respondents​.
1. PHILIPPINE MANUFACTURING COMPANY v. IMPERIAL
DOCTRINE: ​An order of court in a cadastral case amending the ofcial
March 31, 1926 | Street, J. | Notice of Hearing plan so as to make it include land not previously included therein is a
SALAMAT, J. nullity unless new publication is made as a preliminary to such step.
Publication is one of the essential bases of the jurisdiction of the court in
land registration and cadastral cases, and additional territory cannot be
PETITIONER: ​Philippine Manufacturing Company
included by amendment of the plan without new publication
RESPONDENT: ​Honoroable Judge Carlos Imperial

RECIT-READY: ​Philippine Manufacturing Company holds Torrens FACTS:


Certificates to lots Nos. 31, 32, & 35. Bec of a mistake in the Cadastral ● The Director of Lands, on behalf of the Government of the
Plan, there were some lands not really adjudicated to PH Manufacturing Philippine Islands, instituted a cadastral proceeding in the Court of
that was included in Lot No. 35. The lands mistakenly included is a strip
First Instance of Manila for the purpose of procuring an
of land - about 3 hectares - that was formed through accretion from the
west side of the sea. Meanwhile, Cabangis heirs filed a petition for adjudication of the titles to certain lands in the Tondo District of
cadastral proceeding where they asserted ownership over the strip of Manila. Among the prop-erties covered by the preliminary survey
land. They asserted that the strip was not included in the cadastral plan in this cadastral "were the lots bearing the numbers 7, 8, 9, 31, 32,
and requested that it be included in the plan that way it would be 35, and 36, all located on a small peninsula in the Tondo District
adjudicated to them. The Bureau of Lands amended the plan and added between Estero Vitas and the Manila Bay.
the questioned land and indicated it as Lot 40. This survey was made ● The Philippine Manufacturing Company, by virtue of a registration
without previous notice to PH Manufacturing and without publishing the effected many years ago, holds Torrens certicates of title to lots
survey in the Official Gazette. Pursuant to the amended Cadastral Plan,
Nos. 31, 32, and 35. By what appears to have been a mistake in the
Respondent judge adjudicated Lot No. 40 to the Cabangis Heirs.
cadastral plan lot No. 35 was made to include some land not really
ISSUE: adjudicated to said company in the former registration proceeding.
● Whether the trial court’s decision was valid - NO The peninsula referred to appears to be gaining land on its west
side from the sea, and this process, even since the date of the
RULING: ​Respondent judge had no jurisdiction whatever over lot No. survey upon which the cadastral plan was based, has resulted in the
40 in the cadastral case now pending before him and the adjudication of making of a new strip on the west side of said peninsula containing
said lot to the Cabangis heirs by the decision of July 16, 1925, is a mere an area of about three hectares. This strip is immediately
nullity. No publication has ever been made except the initial publication, contiguous to lots Nos. 7, 8, 9, 36, and 35, as the latter was
and this did not include lot No. 40. Publication of course is one of the
originally laid down in the cadastral plan. As the Philippine
essential bases of the jurisdiction of the court in land registration and
cadastral cases, and the attempt that was here made to incorporate lot No. Manufacturing Company claims the whole of lot No. 35 it results
40 into the cadastral was futile. Before a cadastral survey can be that the new strip is contiguous to land claimed by said company in
amended so as to include land in which no publication has been made, the cadastral proceeding
new publication is necessary,—a step essential to the protection of ● Publication was duly made as required by law with respect to all
persons interested in the property which is intended to be included land included in the plan, but of course the new strip was not
included therein as it has been made since the survey. The court
entered a partial decision declaring that lots Nos. 31, 32, and 35 ● Whether the trial court’s decision was valid - NO
had already been decreed in case No. 8425 and that these lots
appeared in the cadastral plan in the name of the Philippine RATIO:
Manufacturing Company. ● Respondent judge had no jurisdiction whatever over lot No. 40 in
● The attorneys for the Cabangis respondents, who will hereafter be the cadastral case now pending before him and the adjudication of
referred to as the Cabangis heirs, led a petition in the cadastral said lot to the Cabangis heirs by the decision of July 16, 1925, is a
proceeding in which they asserted ownership to the strip of newly mere nullity. No publication has ever been made except the initial
made land lying on the shore of the Manila Bay contiguous to and publication, and this did not include lot No. 40. Publication of
west of some of the lots already specied. They informed the court course is one of the essential bases of the jurisdiction of the court
that said land had not been included in the cadastral plan and in land registration and cadastral cases, and the attempt that was
requested that the cadastral plan be amended to the end that it here made to incorporate lot No. 40 into the cadastral was futile.
might be adjudicated to them Before a cadastral survey can be amended so as to include land in
● The Bureau of Lands amended its plan and added to it the which no publication has been made, new publication is
questioned lot, indicated as lot No. 40. The actual survey of this lot necessary,—a step essential to the protection of persons interested
had been made at the instance of the Cabangis heirs, and without in the property which is intended to be included. Even if the order
any previous notice of the survey to the Philippine Manufacturing amending the cadastral plan had not been wholly void, the facts
Company. No notice of the aforementioned survey or of the above revealed would justify the granting of a new trial by this
inclusion of lot No. 40 in the cadastral plan was published in the court under section 513 of the Code of Civil Procedure. However,
Ofcial Gazette. Subsequent to the amendment of the cadastral bec of lack of publication, the order of July 16, 1925 is void, and a
plan in the manner above stated the respondent judge, on July 16, new trial is not necessary
1925, adjudicated and decreed lot No. 40 to the Cabangis heirs ● The petition is therefore granted, the order of July 16, 1925, is set
● More than 30 days later, the atty for Philippine Manufacturing aside, and the registration of lot No, 40 in the name of the
Company for the rst time learned of the adjudication of this lot to Cabangis heirs is abrogated. The owner's duplicate certicate,
the Cabangis heirs. As the time for an appeal had passed and the attached as Exhibit 6 (c) to this pro ceeding, shall be surrendered to
court had lost jurisdiction to change the decree, the present petition the register of deeds of Manila for ling and the word "cancelled"
was led for relief in this court stamped upon it as well as upon the original certicate in the ofce
● The claim of the Philippine Manufacturing Company to lot No. 40 of the register. The nal decree by virtue of which the certicate of
appears to be based, rst, upon an assertion of private ownership in title was issued shall be marked "revoked," both in the General
itself by accretion or reclamation; and, secondly, upon rights Land Registration Ofce and in the ofce of the register of deeds.
acquired by it under a lease granted by the Government. In behalf The costs of this proceeding will be paid by the Cabangis heirs
of the Government it is insisted that the land in question is
foreshore land

ISSUES:
78. AMELITA DOLFO v. THE REGISTER OF DEEDS OF THE
the applicants, and not a motion to intervene in the proceedings – NO.
PROVINCE OF CAVITE, ET. AL.
September 25, 2000 | J. Mendoza | Section 26 - Order of
RULING: T​he provisions of §§ 14 and 25 of P.D. No. 1529 (Property
General Default
Registration Decree) show that the applicant and the oppositor are the
B. SALAZAR &
only parties in cases of original applications for land registration, unlike
in ordinary civil actions where parties may include the plaintiff, the
PETITIONER: ​AMELITA DOLFO defendant, third party complainants, cross-claimants, and intervenors. ​A
RESPONDENT: ​THE REGISTER OF DEEDS FOR THE PROVINCE motion to intervene in a land registration case cannot be allowed. A party
OF CAVITE, TRECE MARTIRES CITY, THE REPUBLIC OF THE wishing to be heard should ask for the lifting of the order of general
PHILIPPINES, LAND REGISTRATION AUTHORITY, CESAR E. default, and then if lifted, file an opposition to the application for
CASAL, RUSTICO A. CASAL, ERNESTO A. CASAL, RODOLFO A. registration. ​This is so because proceedings in land registration are ​in
CASAL, ALFREDO A. CASAL, JR., EMMANUEL A. B. CASAL, rema​ nd ​not in personam​, the sole object being the registration applied
RAFAEL S. CASAL, JR., C. JOSEFINA S. CASAL, CELEDONIA S. for, not the determination of any right connected with the registration.
CASAL, WILHELMINA S. CASAL, MELANIO MEDINA,
ADELAIDA MEDINA, AURORA MEDINA, C.P.G. AGRICOM DOCTRINE: A motion to intervene in a land registration case
CORPORATION and HEIRS OF DAMIAN ERMITANIO and cannot be allowed. A party wishing to be heard should ask for the
CELEDONIA MARTINEZ lifting of the order of general default, and then if lifted, file an
opposition to the application for registration. This is so because
RECIT-READY: ​Amelita Dolfo and Yangtze Properties Inc. filed a proceedings in land registration are ​in rema​ nd ​not in personam​, the
motion to intervene in 3 LRC cases pending before the RTC Branch 19 sole object being the registration applied for, not the determination
of Bacoor, Cavite. One case is for Reconstitution of Original Certificate of any right connected with the registration.
of Title and the other two are for Registration of Title. Dolfo alleged that
she is the registered owner of the real property that is the subject of said
FACTS:
LRC cases as shown by a TCT issued in her name by the Register of
● On March 5, 1996, petitioner Dolfo and Yangtze Properties, Inc.
Deeds of Trece Martires City. Yangtze​(kaya siya involved)h​ ad earlier
(Yangtze) filed a motion for leave to file and/or admit
entered into a Contract to Sell with Dolfo over said propery.
complaint-in-intervention in 3 LRC Cases pending before the
RTC, Branch 19, Bacoor, Cavite.
Trial court denied their motion to intervene because there has already
○ The first case is for ​reconstitution of Original
been an order of general default entered by the court against those who
Certificate of Title​No. 362 purportedly covering the
failed to oppose the application. Court also gave greater weight to the
subject real property, while the last two were cases for
report of the LRA that petitioner's certificate of title was issued without
registration of title​.
any legal basis and the report of the NBI that the signature of Antonia
● Petitioner alleged that she is the registered owner of the real
Cabuco, the Register of Deeds of the Province of Cavite signatory on the
property subject of the said LRC Cases as shown by Transfer
certificate, was a forgery.
Certificate of Title No. T-320601 issued in her name by the
Register of Deeds of Trece Martires City.
● Yangtze, petitioner's co-movant, had earlier entered into a Contract
ISSUE: [Syllabus issue] ​W/N the CA erred in holding that the proper
to Sell with Dolfo over the said property.
remedy in a land registration case is an opposition to the application of
● Trial Court denied the motion saying
○ it is a procedural error to file a complaint for intervention land registration are ​in rem​and ​not in personam,​ the sole object
in cases involving original application for land being the registration applied for, not the determination of any
registration, the proceedings therein being ​in rem right connected with the registration.
○ ​There has already been an order of general default
entered by the court against those who failed to oppose W/N the CA erred in not upholding Dolfo’s title – NO.
the application.
● Dolfo and Yangtze filed a motion for reconsideration. The trial ● The rule that a title issued under the Torrens System is presumed
court denied the same. Court gave greater weight to the report of valid and, hence, is the best proof of ownership of a piece of land
the Land Registration Authority (LRA) that petitioner's certificate does not apply where the certificate itself is faulty as to its
of title was issued without any legal basis and the report of the purported origin.
National Bureau of Investigation (NBI) that the signature of ● Petitioner anchors her arguments on the premise that her title to the
Antonia Cabuco, the Register of Deeds of the Province of Cavite subject property is indefeasible because of the presumption that her
signatory on the certificate, was a forgery. certificate of title is authentic. However, this presumption is
● Atty. Artemio Caña, in his capacity as Acting Register of Deeds of overcome by the evidence presented, consisting of the LRA report
the Province of Cavite, filed a complaint for the annulment of that TCT No. T-320601 was issued without legal basis and the NBI
petitioner's certificate of title before the RTC, Branch 89, Bacoor, report that the signature of Antonia Cabuco was a forgery.
Cavite. The matter remains pending in that court. ● Although petitioner submitted documents purporting to show the
● RTC, Branch 19, Bacoor, Cavite rendered a joint decision genuineness of Antonia Cabuco's signature, she has not refuted the
recognizing and confirming the rights of private respondents over findings contained in the LRA report that her certificate of title has
the litigated property and ordered the issuance of a Decree of no legal basis.
Registration in their favor. o ​there is no document on file in the registry vault to
● Dolfo filed before the CA a petition to annul and set aside the support the issuance of TCT No. T-320601 in favor
above orders of the RTC. CA denied the petition. of Amelita Dolfo.
● Thus, petitioner cannot invoke the indefeasibility of her certificate
ISSUES/ RATIO: of title. It bears emphasis that the Torrens system does not create
or vest title but only confirms and records one already existing and
W/N the CA erred in holding that the proper remedy in a land vested.
registration case is an opposition to the application of the applicants,
and not a motion to intervene in the proceedings – NO. W/N petitioner is allowed to intervene – NO.
● The provisions of §§ 14 and 25 of P.D. No. 1529 (Property ● To allow petitioner to intervene in the LRC cases would not avoid
Registration Decree) show that the applicant and the oppositor are multiplicity of suits in view of the case for annulment and
the only parties in cases of original applications for land cancellation of TCT No. T- 320601 now pending before the RTC
registration, unlike in ordinary civil actions where parties may Branch 89, Bacoor, Cavite.
include the plaintiff, the defendant, third party complainants, ● It is premature for petitioner to intervene in the LRC cases because
cross-claimants, and intervenors. her certificate of title, supposedly her best proof of ownership over
● A motion to intervene in a land registration case cannot be the property described therein, is questionable.
allowed. A party wishing to be heard should ask for the lifting of WHEREFORE, the petition is DENIED and the decision and the
the order of general default, and then if lifted, file an opposition to resolution of the Court of Appeals are AFFIRMED. Costs against
the application for registration. This is so because proceedings in petitioner.
79. REPUBLIC v ESTONILO FACTS:
November 25, 2005 | J. Panganiban | Sec. 26 - Order Of General ● This case originated from an application for registration of a parcel
Default of land in Cagayan de Oro, filed by Nazaria Bombeo
H. Sanchez ● Bombeo claimed that said parcel of land was previously owned
and possessed by a certain Rosendo Bacas since 1894 until it was
sold to her by the heirs of Rosendo Bacas, represented by their
PETITIONER: ​Republic of the Philippines
attorney-in-fact and heir himself, Calistro Bacas by virtue of an
RESPONDENT: ​Anatalia Actub Tiu Estonilo & Andrea Actub Tiu Po
Absolute Sale of Realty
(in Substitution Of Nazaria Bombeo)
● After due notice and publication of said application, the Provincial
Fiscal of Misamis Oriental (in behalf of of the Chief of Staff of the
RECIT-READY: ​Bombeo wanted to register her land but the Republic
AFP & Director of Bureau of Lands) opposed to this alleging the
opposed, saying the land is not registrable as it’s been segregated for use
land is not registrable pursuant to PD 265
of the Philippine Army by virtue or PD 265. The lower court & CA ruled
● PD 265 declared the lease-sale of the land to the Phil. Army
that PD 265 wasn’t self-executing since a judicial order is needed for it to
● During the initial hearing set on February 12, 1955, an Order of
take effect.
General Default was issued by the lower court.
● The trial court confirmed title over the land to Nazaria Bombeo
ISSUE:
(substituted by her heirs Anatalia Actub Tiu Estonilo and Andrea
● W/N PD 265 needs judicial order for it to be executed because it
Actub Tiu Po) and ordered its registration under the names of the
is non self executing? NO
latter.
● W/N Estonillo has the required period of possession under the
● Oppositors Bureau of Lands and Chief of Staff of Armed Forces of
Public Land Act? - NO
the Philippines, thru OSG, appealed
● CA ruled that PD 265 failed to segregate effectively Lot 4318 as
RULING: ​The Public Land Act does not require a judicial order to
part of the military reservation. It also said the proclamation was
create a military reservation Under Sec 83, 86, 87, 53 of the law, only a
'not self-executory and self-adjudicating considering that there is a
positive act of the President is needed to segregate a piece of land for a
need to determine private rights of claimants over lands sought to
public purpose. While Section 53 grants authority to the director of lands
be reserved.
to file a petition against claimants of the reserved land, the filing of that
petition is not mandatory. The director of lands is required to file a
ISSUES:
petition only 'whenever in the opinion of the President public interest
● W/N PD 265 needs judicial order for it to be executed because it is
requires it. And since PD 265 is self-executing, the respondents could not
non self executing? NO
have validly possessed it after its issuance. Also, the evidence they
● W/N Estonillo has the required period of possession under the
presented were insufficient to prove possession anyway.
Public Land Act? - NO

RELEVANT ARGUMENTS (if any):


DOCTRINE: ​To segregate portions of the public domain as reservations
● Respondent: Estonillo alleges that their predecessors-in-interest
for the use of the Republic of the Philippines or any of its branches, like
were already in adverse, open, peaceful and continuous possession
the Armed Forces of the Philippines, all that is needed is a presidential
of the property for over 30 years prior to 1938. Thus, they
proclamation to that effect. A court judgment is not necessary to make
conclude that their imperfect title had already attached long before
the proclamation effective or valid.
the issuance of the Proclamation segregating the land as a military declarations were issued under the names of respondents'
reservation. predecessors-in-interest, the earliest one presented was issued only
in 1954.
RATIO: ● Evidence on record merely established the transfer of the property
from Calixto Bacas to Nazaria Bombeo. The evidence did not
W/N PD 265 needs judicial order for it to be executed because it is non show the nature and the period of the alleged possession by Calixto
self executing? NO and Rosendo Bacas.
● The Public Land Act requires applicants for confirmation of ● Applicants for judicial confirmation of imperfect titles must
imperfect titles to prove present specific acts of ownership to substantiate their claims; they
○ (1) that the land is alienable public land; and cannot simply offer general statements that are mere conclusions
○ (2) that their open, continuous, exclusive and notorious of law rather than factual evidence of possession.
possession and occupation of the property has taken place ● Also, a person is not necessarily entitled to have the land registered
either since time immemorial or for the period prescribed under the Torrens system simply because no one appears to oppose
by law. his title and to oppose the registration of his land. He must show,
● If these are complied with, the possessor of the land -- by operation even though there is no opposition, to the satisfaction of the court,
of law -- acquires a right to a government grant, without that he is the absolute owner, in fee simple.
necessitating the issuance of a certificate of title.
● The Public Land Act does not require a judicial order to create a WHEREFORE, the Petition is GRANTED , and the assailed Decision
military reservation of the Court of Appeals is REVERSED and SET ASIDE . The
● Under Sec 83, 86, 87, 53 of the law, only a positive act of the segregation of Lot 4318 as part of a military reservation is declared
President is needed to segregate a piece of land for a public VALID. No pronouncement as to costs.
purpose.
● While Section 53 grants authority to the director of lands --
through the solicitor general -- to file a petition against claimants
of the reserved land, the filing of that petition is not mandatory.
The director of lands is required to file a petition only 'whenever in
the opinion of the President public interest requires it.
● Consequently, respondents could not have validly occupied it in
1954, because it was considered inalienable since its reservation in
1938.

W/N Estonillo has the required period of possession under the Public
Land Act? - NO
● Land that has not been acquired from the government, either by
purchase or by grant, belongs to the State as part of the public
domain.
● In granting respondents judicial confirmation of their imperfect
title, the trial and the appellate courts gave much weight to the tax
declarations presented by the former. However, while the tax
80. HEIRS OF ROXAS v. CA
RULING:
March 21, 1997 | G.R. No. 118436 | Romero | Notice of Initial Hearing
Publication of the Notice of Initial Hearing was made in the O fficial
SEGOVIA | SERRANO
Gazette and in the Record Newsweekly, admittedly not a newspaper of
general circulation. Pursuant to Section 23 of Presidential Decree No.
PETITIONER: ​Heirs of Manuel A. Roxas and Trinidad de Leon Vda. 1529, publication in the O fficial Gazette is su fficient to confer
De Roxas (in substitution of original petitioner) jurisdiction.
RESPONDENT: ​Court of Appeals and Maguesun Management &
Development Corporation While publication of the notice in the O fficial Gazette is su fficient to
confer jurisdiction upon the court, publication in a newspaper of general
RECIT-READY: ​Maguesun Corporation filed an application for circulation remains an indispensable procedural requirement. Couched in
registration of two parcels of unregistered land located in Tagaytay City. mandatory terms, it is a component of procedural due process and aimed
In support of its application for registration, Maguesun Corporation at giving "as wide publicity as possible" so that all persons having an
presented a Deed of Absolute Sale executed by Zenaida Melliza as adverse-interest in the land subject of the registration proceedings may be
vendor. Zenaida Melliza in turn, bought the property from the original noti fied thereof. Although jurisdiction of the court is not affected, the
petitioner herein, Trinidad de Leon vda. de Roxas two and a half months fact that publication was not made in a newspaper of general circulation
earlier, as evidenced by a Deed of Sale. Notices of the initial hearing is material and relevant in assessing the applicant's right or title to the
were sent by the Land Registration Authority to Hilario Luna, Jose Gil land.
and Leon Luna as indicated on Maguesun Corporation's application for
registration. Since Trinidad de Leon vda. de Roxas was not named as an DOCTRINE:
adverse claimant, she was not sent a notice of the proceedings. While publication of the notice in the O fficial Gazette is su fficient to
Publication was made in the O fficial Gazette and the Record confer jurisdiction upon the court, publication in a newspaper of general
Newsweekly. After an Order of general default was issued, the trial court circulation remains an indispensable procedural requirement. Although
proceeded to hear the land registration case. Eventually, the RTC granted jurisdiction of the court is not affected, the fact that publication was not
Maguesun Corporation's application for registration. It was only when made in a newspaper of general circulation is material and relevant in
the caretaker of the property was being asked to vacate the land that assessing the applicant's right or title to the land.
petitioner Trinidad de Leon Vda. de Roxas learned of its sale and the
registration of the lots in Maguesun Corporation's name. Hence,
FACTS:
petitioner filed a petition for review before the RTC to set aside the
● Maguesun Management and Development Corporation (Maguesun
decree of registration on the ground that Maguesun Corporation
Corporation) filed an application for registration of two parcels of
committed actual fraud. The CA ruled in favor of Maguesun Corporation
unregistered land located in Brgy. Sungay, Tagaytay City.
that there was no fruad; and that publication of the initial hearing in the
● In support of its application for registration, Maguesun
O fficial Gazette is su fficient to confer jurisdiction upon the court.
Corporation presented a Deed of Absolute Sale executed by
Zenaida Melliza as vendor and indicating the purchase price to be
ISSUE:
P170,000.00. Zenaida Melliza in turn, bought the property from
● W/N the CA erred in ruling that Maguesun Corporation did not
the original petitioner herein, Trinidad de Leon vda. de Roxas for
commit actual fraud warranting the setting aside of the
P200,000.00 two and a half months earlier, as evidenced by a Deed
registration decree - ​YES
of Sale.
● Notices of the initial hearing were sent by the Land Registration was named, was her niece. Manolita Suntay is the daughter of
Authority to Hilario Luna, Jose Gil and Leon Luna on the basis of Lourdes Guevarra Suntay, a deceased cousin of petitioner Vda. de
Maguesun Corporation's application for registration. Since Roxas who used to help with the latter's business affairs. Manolita
Trinidad de Leon vda. de Roxas was not named as an adjoining Suntay used to take care of the registration and insurance of the
owner, occupant or adverse claimant, she was not sent a notice of latter's cars.
the proceedings. ● The Court of Appeals held that petitioner failed to demonstrate that
● Publication was made in the O fficial Gazette and the Record there was actual or extrinsic fraud, not merely constructive or
Newsweekly. intrinsic fraud, a prerequisite for purposes of annulling a judgment
● After an Order of general default was issued, the trial court or reviewing a decree of registration.
proceeded to hear the land registration case. Eventually, on ● The Court of Appeals also ruled that publication of the initial
February 13, 1991 the Regional Trial Court granted Maguesun hearing in the O fficial Gazette is su fficient to confer
Corporation's application for registration. jurisdiction upon the court.
● It was only when the caretaker of the property was being asked to
vacate the land that petitioner Trinidad de Leon Vda. de Roxas ISSUE:
learned of its sale and the registration of the lots in Maguesun ● W/N the CA erred in ruling that Maguesun Corporation did not
Corporation's name. commit actual fraud warranting the setting aside of the registration
● Hence, petitioner filed a petition for review before the RTC to set decree - ​YES
aside the decree of registration on the ground that Maguesun
Corporation committed actual fraud. She alleged that the lots were RULING:
among the properties she inherited from her husband, former [On Fraud - not really relevant to our topic but to explain lang how the
President Manuel A. Roxas, who died on April 15, 1946 and that Court set aside the CA decision]
her family had been in open, continuous, adverse and uninterrupted ● The Court here finds that respondent Maguesun Corporation
possession of the subject property in the concept of owner for more committed actual fraud in obtaining the decree of registration
than thirty years before they applied for its registration under the sought to be reviewed by petitioner.
Torrens System of land titling. Petitioner further denied that she ● Petitioner Vda. de Roxas contended that Maguesun Corporation
sold the lots to Zenaida Melliza whom she had never met before intentionally omitted their name, or that of the Roxas family, as
and that her signature was forged in both the Deed of Sale and the having a claim to or as an occupant of the subject property.
A davit of Self-Adjudication. In support of her claims, she also ○ In the corporation’s application for registration filed with
listed a number of irregularities in the documents to prove actual the trial court, it seems that the name of Roxas was
fraud. In addition, and perhaps more signi cantly, she claimed that intentionally omitted with correction fluid, and typed
Maguesun Corporation intentionally omitted her name as an over, with a different typewriter, with the words “
adverse claimant, occupant or adjoining owner in the application Provincial Road all at Tagaytay City”.
for registration submitted to the Land Registration Authority such ● The truth is that the Roxas family had been in possession of the
that the latter could not send her a Notice of Initial Hearing. As property uninterruptedly through their caretaker, Jose Ramirez.
result, an order of general default was issued and Maguesun ● Respondent corporation's intentional concealment and
Corporation's application for registration was granted. representation of petitioner's interest in the subject lots as
● She charged Maguesun Corporation with knowledge or authorship possessor, occupant and claimant constitutes actual fraud justifying
of the fraud owing to the fact that Maguesun Corporation's the reopening and review of the decree of registration.
president, Manolita Guevarra Suntay after whom the corporation
[On Notice of Initial Hearing - RELEVANT!] corresponding decree of registration and certificate of title pursuant to
Section 39 of Presidential Decree No. 1529.
● Publication of the Notice of Initial Hearing was made in the
O fficial Gazette and in the Record Newsweekly, admittedly not a
newspaper of general circulation. The Court of Appeals held that
pursuant to Section 23 of Presidential Decree No. 1529,
publication in the O fficial Gazette is su fficient to confer
jurisdiction. Said provision of law expressly states that "the
Commissioner of Land Registration shall cause a notice of initial
hearing to be published once in the O fficial Gazette and once in a
newspaper of general circulation in the Philippines. Provided,
however, that the publication in the O fficial Gazette shall be
sufficient to confer jurisdiction upon the court. . . ."
● While publication of the notice in the O fficial Gazette is
su fficient to confer jurisdiction upon the court, publication in a
newspaper of general circulation remains an indispensable
procedural requirement. Couched in mandatory terms, it is a
component of procedural due process and aimed at giving "as wide
publicity as possible" so that all persons having an adverse-interest
in the land subject of the registration proceedings may be
noti fied thereof. Although jurisdiction of the court is not
affected, the fact that publication was not made in a newspaper of
general circulation is material and relevant in assessing the
applicant's right or title to the land.

WHEREFORE, the instant petition is hereby GRANTED. The Decision


of the Court of Appeals in C.A. G.R. CV No. 38328 ("Trinidad de Leon
Vda. de Roxas v. Maguesun Management & Development Corporation,
et al.") promulgated on December 8, 1994 is hereby REVERSED AND
SET ASIDE. Accordingly, registration of title over the subject parcels
of land, described in Plan AS-04-000108, Lot Nos. 7231 and 7239, with
an area of 3,461 and 10,674 square meters, respectively, as shown and
supported by the corresponding technical descriptions now forming
part of the Records of LRC No. TG-373, is awarded to herein petitioner
Trinidad de Leon vda. de Roxas and her heirs, herein substituted as
petitioners. Upon nality of this Decision, the Land Registration
Authority is hereby directed to ISSUE with reasonable dispatch the
81. REPUBLIC OF THE PHILIPPINES v JEREMIAS AND DAVID
ISSUE:
HERBIETO
W/N the mandatory requirement of publication was complied with
26 May 2005 | G.R. No. 156117 | Chico- Nazario, J | When is a land of
sufficiently, making the MTC judgment valid ​(NO)
public domain alienable and disposable
Made by: SERRANO
RULING:
The MTC Judgment, dated 21 December 1999, ordering the registration
Recit Ready​: and confirmation of the title of respondents Jeremias and David over Lots
No. 8422 and 8423, respectively; as well as the MTC Order, dated 02
On September 23, 1998, Herbieto brothers,, Jeremias and David, filed February 2000, declaring its Judgment of 21 December 1999 Final and
with the MTC a single application for registration of 2 parcels of land executory, and directing the LRA Administrator to issue a decree of
located in Cabangahan Consolacion Cebu (Subject Lots). They claim to registration for the Subject Lots, are both null and void for having been
be owners in fee simple of the Subject Lots which was purchased from issued by the MTC without jurisdiction. The late publication of the
the parents. (Sps. Gregorio Herbieto nd Isabel Owatan) on June 25, 1976. Notice of Initial Hearing in the newspaper of general circulation is
The MTC scheduled a hearing on ​September 3, 1999 MTC ​set an ​initial tantamount to no publication at all, having the same ultimate result.
hearing at 8:30 am. All owners of land adjoining the Subject Lots were Indubitably, such late publication of the Notice, way after the date of the
sent copies of Notice of Initial Hearing. On ​July 27, 1999​, ​copies were initial hearing, would already be worthless and ineffective. Whoever read
placed on a conspicuous plac​e on the Subject Lots as well as the the Notice as it was published in The Freeman Banat News and had a
bulletin board of the municipal building of Consolacion Cebu where the claim to the Subject Lots was deprived of due process for it was already
lots were located. ​August 2, 1999 Notice was published in the Official too late for him to appear before the MTC on the day of the initial
Gazette. ​December 19, 1999 Notice was published in ​The Freeman hearing to oppose respondents' application for registration, and to present
Banat News. S ​ ept. 3, 1999, ​The MTC issued an Order of Special Default his claim and evidence in support of such claim. Worse, as the Notice
with only the petitioner Republic opposing the application for the itself states, should the claimant-oppositor fail to appear before the MTC
registration of the Subject Lots. on the date of initial hearing, he would be in default and would forever be
barred from contesting respondents' application for registration and even
The MTC and CA granted the application. They both held the land the registration decree that may be issued pursuant thereto. In fact, the
sought to be registered has been classified within the alienable and MTC did issue an Order of Special Default on 03 September 1999.​The
disposable zone since June 25, 1963. In accordance to Section 12(4) of all encompassing in rem nature of land registration cases, the
PD 1529, which allows individuals to own land in any manner provided consequences of default orders issued against the whole world and the
by law. objective of disseminating the notice in as wide a manner as possible
demand a mandatory construction of the requirements for publication,
However, the Republic appealed and averred that the CA erred in mailing and posting.
granting the petition.
No. 8422 and Dacid over Lot No. 8423. The order also directed the
Doctrine: The all encompassing in rem nature of land registration cases,
Administrator of the RA to issue a decree of registration for the
the consequences of default orders issued against the whole world and
Subject Lots.
the objective of disseminating the notice in as wide a manner as possible
○ Feb. 2, 2000 ​declared its judgement.
demand a mandatory construction of the requirements for publication,
● CA​: Affirmed the MTC judgment saying that the land sought to be
mailing and posting.
registered as within the alienable and disposable zone. CA also
acknowledge that the parents of the appellees have acquired
FACTS: subject parcels of land since 1950 and cultivated it with jackfruits,
● This petition seeks to reverse the judgment granting the application bamboos, coconuts, and other trees In sort, it is undisputed that the
for land registration of the respondents. appellees are predecessors-in-interest.
● Herbieto brothers (Jeremias and David as respondents) filed a
single application for registration of two parcels of land (Lots No. ISSUE:
8422 and 8423, located in Cabangahan, Consolacion, Cebu (1) W/N the mandatory requirement of publication was complied with
(Subject Lots). sufficiently, making the MTC judgment valid ​(NO)
● The brothers claim to be in fee simple of the Subject Lots which
they purchased from their parents, Sps. Herbieto and Isabel RATIO:
Owatan on June 25, 1976. The MTC Judgment, dated 21 December 1999, ordering the registration and
● On 11 December 1998, petitioner Republic of the Philippines confirmation of the title of respondents Jeremias and David over Lots No.
(Republic) filed an opposition to the application of the Subjects 8422 and 8423, respectively; as well as the MTC Order, dated 02 February
Lots because of the ff argument. 2000, declaring its Judgment of 21 December 1999 Final and executory,
● September 3, 1999 MTC ​set an ​initial hearing at 8:30 am. all and directing the LRA Administrator to issue a decree of registration for the
owners of land adjoining the Subject Lots were sent copies of Subject Lots, are both null and void for having been issued by the MTC
Notice of Initial Hearing without jurisdiction. The late publication of the Notice of Initial Hearing in
● July 27, 1999​. ​Copies were also placed on a conspicuous plac​e the newspaper of general circulation is tantamount to no publication at all,
on the Subject Lots as well as the bulletin board of the municipal having the same ultimate result. Indubitably, such late publication of the
building of Consolacion Cebu where the lots were located. Notice, way after the date of the initial hearing, would already be worthless
● August 2, 1999​ Notice was duly published in the Official Gazette. and ineffective. Whoever read the Notice as it was published in The
● December 19, 1999 Notice was published in ​The Freeman Banat Freeman Banat News and had a claim to the Subject Lots was deprived of
News due process for it was already too late for him to appear before the MTC on
● Sept. 3, 1999, ​The MTC issued an Order of Special Default with the day of the initial hearing to oppose respondents' application for
only the petitioner Republic opposing the application for the registration, and to present his claim and evidence in support of such claim.
registration of the Subject Lots. Worse, as the Notice itself states, should the claimant-oppositor fail to
● MTC on December 21, 1999 ​promulgated its judgement ordering appear before the MTC on the date of initial hearing, he would be in default
the registration and confirmation of the title of Jeremias over Lot and would forever be barred from contesting respondents' application for
registration and even the registration decree that may be issued pursuant its Order, dated 02 February 2000 are declared NULL AND VOID.
thereto. In fact, the MTC did issue an Order of Special Default on 03 Respondents' application for registration is DISMISSED.
September 1999.​The all encompassing in rem nature of land registration SO ORDERED.
cases, theconsequences of default orders issued against the whole world and
the objective of disseminating the notice in as wide a manner as possible
demand a mandatory construction of the requirements for publication,
mailing and posting.

PLEASE NOTE:
Section 23 of the Property Registration Decree requires that the public be
given Notice of the Initial Hearing of the application for land registration by
means of:
(1)publication; (2) mailing; and (3) posting.

Publication of the Notice of Initial Hearing shall be made in the following


manner:
1. By publication. —
Upon receipt of the order of the court setting the time for initial hearing, the
Commissioner of Land Registration shall cause a notice of initial hearing to
be published once in the Official Gazette and once in a newspaper of
general circulation in the Philippines: Provided, however, that the
publication in the Official Gazette shall be sufficient to confer jurisdiction
upon the court. Said notice shall be addressed to all persons appearing to
have an interest in the land involved including the adjoining owners so far
as known, and "to all whom it may concern." Said notice shall also require
all persons concerned to appear in court at a certain date and time to show
cause why the prayer of said application shall not be granted.

DISPOSITION:
WHEREFORE, based on the foregoing, the instant Petition is GRANTED.
The Decision of the Court of Appeals in CA-G.R. CV No. 67625, dated 22
November 2002, is REVERSED. The Judgment of the MTC of
Consolacion, Cebu in LRC Case No. N-75, dated 21 December 1999, and
82. VERGEL V. CA FACTS:
September 28, 2001​ | ​PARDO, ​J​.​ | General default
Serrano | Soriano ● On May 26, 1994, Digna Vergel, Eduardo Salvacruz, Beatriz
Mañacop, Felicisima Flores, Generoso and Blandino Salvacruz,
Milagros Evangelista and the heirs of Corazon Santiago, namely:
PETITIONER: ​Digna Vergel, Eduardo Salvacruz, Beatriz Manacop,
Leocadio, Jr. and Concepcion Santiago filed with the Regional
Felicisima Flores, Generoso Salvacruz, Blandino Salvacruz, Milagros
Trial Court, Calamba, Laguna an application for registration of a
Salvacruz And The Heirs Of Corazon Santiago
parcel of land (for titling purposes).
RESPONDENT: ​Court Of Appeals And Dorotea Tamisin Gonzale​s
● On July 20, 1994, the Republic of the Philippines represented by
the Director of Lands filed an opposition to the application for
RECIT-READY: ​Vergel et al., filed with the RTC of Calamba Laguna
registration.
an application for registration of land. Republic of the Philippines filed
● On December 15, 1994, the trial court issued "an order of general
an opposition to the said application. RTC issued a general default with
default against the whole world with the exception of Republic of
regards to the said land. Gonzales filed a motion to set aside the order of
the Philippines
general default claiming that she is the owner of said land. RTC denied
● On October 3, 1995, respondent Dorotea Tamisin Gonzales filed
motion for lack of merit. Gonzales filed with the CA a petition for
with the trial court an "Urgent Motion to Set Aside the Order of
certiorari and CA subsequently reversed the RTC decision. Hence this
General Default" alleging, in her affidavit that she is claiming the
appeal by the Vergel et al.
land in question subject of this petition as an owner which motion
was opposed by the Vergel et al herein.
ISSUE: ​The issue presented is whether the Court of Appeals erred in
● On October 18, 1995, the trial court issued the first assailed order,
setting aside the trial court’s order of general default in the land
the dispositive portion of which is quoted, as follows:
registration case involved without making a specific finding of fraud,
negligence, accident or excusable mistake ​(YES) ‘WHEREFORE, the motion to set aside the Order of default as
well as the motion to suspend the proceedings filed by the movant
RULING: ​We grant the petition. The Court of Appeals arbitrarily set through counsel is hereby denied for lack of merit.’
aside the trial court’s order of general default without factual basis save
for its own gut feeling, ​ipse dixit​. Gonzales’ failure to file timely ● On November 21, 1995, Gonzales filed with the trial court a
opposition to the application for land registration in itself may not be "motion for reconsideration" of the order denying the motion to set
considered excusable negligence. Hence, we find that the appellate court aside the order of general default, which motion Vergel et al
erred in setting aside the order of general default in the Land Registration opposed.
Case No. 88-94-C, without making a specific finding of fraud, accident ● On November 28, 1995, the trial court issued its second questioned
or excusable neglect that prevented respondent from timely opposing the order, the dispositive portion of which reads as follows:
application.
‘WHEREFORE, in view of the foregoing, the motion for
DOCTRINE: ​It is erroneous to set aside the order of general default reconsideration, dated November 16, 1995, is hereby denied for
without making a specific finding of fraud, accident or excusable neglect. lack of merit.’

● On December 13, 1995, Gonzales filed with the Court of Appeal sa


petition for certiorari alleging that the trial court judge "acted
capriciously and without or in excess of his jurisdiction and ● We are not a trier of facts. Consequently, we have to remand the
gravely abused the exercise of his discretion" in issuing the two case to the Court of Appeals for it to make findings of fact
aforementioned orders." constituting fraud, accident or excusable neglect sufficient for the
● On April 02, 1996, the Court of Appeals promulgated a decision court to lift the order of general default in the land registration case
annulling the trial court’s orders dated October 18, 1995 and involved.
November 28, 1995, and consequently, setting aside the trial
court’s order of general default dated December 15, 1994, in Land WHEREFORE, the Court SETS ASIDE the decision of the Court of
Registration Case No. 88-94-C with respect to Gonzales. Appeals in CA-G. R. SP No. 39239.
● Hence, this appeal
Let the case be remanded to the Court of Appeals for further
proceedings with instructions to determine whether there exists facts
ISSUES: warranting the lifting of the order of general default in LRC Case No.
● The issue presented is whether the Court of Appeals erred in 88-94-C of the trial court.
setting aside the trial court’s order of general default in the land
registration case involved without making a specific finding of
fraud, negligence, accident or excusable mistake (YES)

RATIO:

● We grant the petition. The Court of Appeals arbitrarily set aside


the trial court’s order of general default without factual basis save
for its own gut feeling, ​ipse dixit​. Gonzales’ failure to file timely
opposition to the application for land registration because she
missed reading the publication of the notice in the Official Gazette
or in the newspaper "Malaya" issue of August 8, 1994, in itself
may not be considered excusable negligence.
● In Gonzales’ motion to set aside order of general default, she
alleged that petitioners were aware of her claim of ownership over
the subject property, but did not give her personal notice of the
filing of the application. She learned about the application by
accident. In the petition for certiorari she filed with the Court of
Appeals, Gonzales alleged that Vergel et al filed the application in
bad faith, surreptitiously and without notice to her. The Court of
Appeals did not make a finding on this.
● Hence, we find that the appellate court erred in setting aside the
order of general default in the Land Registration Case No.
88-94-C, without making a specific finding of fraud, accident or
excusable neglect that prevented respondent from timely opposing
the application.
83. SPS MANUEL DEL CAMPO v. CA
appearing, order a default to be recorded and require the applicant
1 Feb. 2001| J. Quisumbing | Order of General Default
to present evidence.
P. SORIANO & A. ALVERO
Where an appearance has been entered and an answer filed, a
PETITIONER: ​Spouses Manuel and Salvacion del Campo default order shall be entered against persons who did not appear
RESPONDENT: ​Court of Appeals and Heirs of Jose Regalado, Sr and answer.

RECIT-READY: ​Salome and 7 others co-owned a lot. The lot was The facts in italics are the only passages directly related to the provision.
divided in aliquot shares. Salome sold her part to Soleda with a Deed of Case did not discuss default. Best guess is Regalado’s widow and one
Absolute Sale passing the property to Soledad. Soledad mortgaged the child were issued order of special default. Said order is when an
property to Regalado Sr as evidenced by a Deed of Mortgage. Soledad appearance has been entered and answer filed, default order shall be
died. Soledad's heirs later paid the mortgage and Regalado Sr. executed a entered upon against persons who didn't appear and answer.]
Discharge of Mortgage. The Soledad heirs then sold their lot to Spouses
del Campo.
FACTS:
Regalado caused the reconstituted of the original certificate of title which
● Salome, Consorcia, Alfredo, Maria, Rosalia, Jose, Quirico and
included the lot previously mortgaged. Sps del Campo now assail the title
Julita, all surnamed Bornales, were the original co-owners of Lot
on the ground that they own part of the lot, after Soledad heirs sold it to
162 Original Certificate of Title No. 18047
them. Summons were served to Regalado’s widow and 2 of his children.
● The lot was divided in aliquot shares among the eight (8)
Only 1 child, Antonio answered.
co-owners as follows:
○ Salome Bornales 4/16
ISSUE: ​W/N the heirs of Regalado are estopped from denying the right
○ Consorcia Bornales 4/16
and title of Sps. del Campo. -YES
○ Alfredo Bornales 2/16
○ Maria Bornales 2/16
RULING: ​The deed of mortgage between Regalado Sr. and Soledad as
○ Jose Bornales 1/16
well as the discharge of mortgage between Regalado Se. and Soledad’s
○ Quirico Bornales 1/16
heirs state that Soledad was the owner of the property. Having conformed
○ Rosalia Bornales 1/16
to the documents, Regalado Sr. cannot deny Soledad’s ownership nor can
○ Julita Bornales 1/16
Regalado’s heirs. They are estopped from denying the right and title of
● Salome sold part of her 4/16 share in Lot 162 Soledad Daynolo.
Sps. del Campo, to whom Soledad’s heirs sold the property to.
The portion of Lot 162 sold to Soledad was described in the Deed
of Absolute Sale as having:
DOCTRINE: ​If an individual has executed a Deed of Mortgage over a
land recognizing another as the absolute owner thereof, the individual
63-1/2 meters from point "9" to "10", 35 meters from
and his heirs are estopped from asserting ownership over the same land.
point "10" to point "11", 30 meters from point "11" to a
certain point parallel to a line drawn from points "9" to
[The case is listed under Section 26 which states:
"10"; and then from this "Certain Point" to point "9" and
If no person appears and answers within the time allowed, the court
as shown in the accompanying sketch, and made an
shall, upon motion of the applicant, no reason to the contrary
integral part of this deed, ​to SOLEDAD DAYNOLO, Antonio to have waived his opportunity to present evidence, the
her heirs and assigns. trial court deemed the case submitted for decision.
● The trial court rendered judgment dismissing the complaint.
● Thereafter, Soledad Daynolo immediately took possession of the ● Spouses appealed.
land built a house thereon. A few years later, Soledad and her
husband, Simplicio Distajo, mortgaged the subject portion of ISSUES: ​W/N the heirs of Regalado are estopped from denying the right
Lot 162 as security for a P400.00 debt to Jose Regalado, Sr. and title of Sps. del Campo. -YES
evidenced by a Deed of Mortgage.
● [Later] three of the eight co-owners of Lot 162, specifically, RELEVANT ARGUMENTS (if any):
Salome, Consorcia and Alfredo, sold 24,993 square meters of said ●
lot to Jose Regalado, Sr.
● Simplicio Distajo, heir of Soledad Daynolo who had since died, RATIO:
paid the mortgage debt and redeemed the mortgaged portion of Lot ● Heirs of Regalado are estopped from asserting that they own the
162 from Jose Regalado, Sr. ​The latter, in turn, executed a Deed subject land in view of the Deed of Mortgage and Discharge of
of Discharge of Mortgage 3 in favor of Soledad's heirs​. The Mortgage executed between Regalado and Spouses'
heirs sold the redeemed portion of Lot 162 for P1,500.00 to the predecessor-in-interest.
Manuel Del Campo and Salvacion Quiachon. ● Heirs are barred from making this assertion under the equitable
● Jose Regalado, Sr. caused the reconstitution of Original Certificate principle of estoppel by deed, whereby a party to a deed and his
of Title No. 18047. The reconstituted OCT No. RO-4541 initially privies are precluded from asserting as against the other and his
reflected the shares of the original co-owners in Lot 162. However, privies any right or title in derogation of the deed, or from denying
title was transferred later to Jose Regalado, Sr. who subdivided the the truth of any material fact asserted in it.
entire property into smaller lots, each covered by a respective title ● A perusal of the documents evidencing the mortgage would readily
in his name. One of these small lots is Lot No. 162-C-6 with an reveal that Soledad, as mortgagor, had declared herself absolute
area of 11,732 square meters. owner of the piece of land now being litigated. This declaration of
● Manuel and Salvacion del Campo brought this complaint for fact was accepted by Regalado as mortgagee and accordingly, his
"repartition, resurvey and reconveyance" against the heirs of the heirs cannot now be permitted to deny it.
now deceased Jose Regalado, Sr. Sps del Campo claimed that they
owned an area of 1,544 square meters located within Lot 162-C-6
which was erroneously included in TCT No. 14566 in the name of WHEREFORE, the petition is GRANTED. The assailed decision of the
Regalado. Court of Appeals in CA-G.R. CV No. 30438 is REVERSED and SET
● Summons were served on Regalado's widow, Josefina Buenvenida, ASIDE. The parties are directed to cause a SURVEY for exact
and two of her children, Rosemarie and Antonio. Josefina and determination of their respective portions in Lot 162-C-6. Transfer
Rosemarie were declared in default because only Antonio filed an Certificate of Title No. 14566 is declared CANCELLED and the
answer to the complaint. Register of Deeds of Capiz is ordered to ISSUE a new title in
● Spouses presented the Deed of Absolute Sale executed between accordance with said survey, upon finality of this decision.
Soledad Daynolo and Salome Bornales as well as the Deed of
Mortgage and Deed of Discharge signed by Jose Regalado, Sr.
● Despite the filing of an answer, Antonio failed to present any
evidence to refute the claim of petitioners. Thus, after considering
84. LOPEZ v. ENRIQUEZ
before entry of final judgment. The land registration court granted the
21 Jan 2005 | J. Carpio | Order of General Default
application for registration of title on 31 May 1966 and issued a
R. REALUBIN & MAKER
certificate of finality on 8 March 1991. Petitioners led their motion on 16
July 1997. Thus, even if petitioners led a motion to lift the order of
PETITIONER: ​Heirs of Eugenio Lopez Sr general default, the order of default could not be set aside because the
RESPONDENT: ​Hon. Alfredo R. Enriquez, in his capacity as motion was filed out of time.
Administrator of the Land Registration Authority and the Register of
Deeds of Marikina City DOCTRINE: ​In cases of registration procured by fraud the owner may
pursue all his legal and equitable remedies against the parties to such
RECIT-READY: ​Alfonso Sandoval ("Sandoval") and Roman Ozaeta, fraud, without prejudice, however, to the rights of any innocent holder for
Jr. ("Ozaeta") led an application for registration of title. Their value of a certificate of title. This is called an Action for Reconveyance.
applications were approved and two OCTs were issued. The heirs of
Lopez Sr filed a motion to declare the deeds and OCTs void contending
FACTS:
the land was sold to their predecessor. They filed an application to
● Alfonso Sandoval ("Sandoval") and Roman Ozaeta, Jr. ("Ozaeta")
annotate a notice of lis pendens at the back of the OCTs which the LRA
led an application for registration of title. On 31 May 1966, the
denied. Relying on Section 24, Rule 14 of the Rules of Court, the LRA
land registration court granted the application. The decision
ruled that only a party to a case has the legal personality to le a notice of
became final and executory, and the land registration court issued a
lis pendens relative to the pending case. Since a land registration case is a
certificate of finality dated 8 March 1991. LRA issued on 20
proceeding in rem, an order of general default binds the whole world as a
October 1977 Decree Nos. N-217643 and N-217644 in the names
party in the case. Petitioners are mere movants whose personality the
of Sandoval and his wife Rosa Ruiz, and Ozaeta and his wife Ma.
court has not admitted. Based on Section 26 of PD 1529, the LRA ruled
Salome Lao.
that petitioners should have filed a motion to lift the order of general
● On 16 July 1997, petitioners Eugenio Lopez, Jr., Manolo Lopez,
default.
Oscar Lopez, and Presentacion L. Psinakis, heirs of Eugenio
Lopez, Sr., filed a motion stating that Sandoval and Ozaeta sold the
ISSUE:
lots subject of the application to the late Eugenio Lopez, Sr. on 23
Whether or not the notice of lis pendens can be annotated to the OCTs
September 1970.
granting that an Order for General Default is obtained? - ​NO (the
● The Register of Deeds of Marikina City issued the corresponding
remedy should be to file an Action for Reconveyance)
OCT Nos. O-1603 and O-1604 in favor of Sandoval and Ozaeta
and their spouses only on 18 August 1998.
RULING:
● Petitioners led another motion on 25 November 1998 to declare
In cases of registration procured by fraud the owner may pursue all his
void the Decrees and OCTs. Petitioners pointed out that the OCTs
legal and equitable remedies against the parties to such fraud, without
show that incumbent Administrator Alfredo R. Enriquez signed the
prejudice, however, to the rights of any innocent holder for value of a
Decrees on 20 October 1997, before he assumed office on 8 July
certificate of title. This is called an Action for Reconveyance.
1998 and even before Hon. Briccio C. Ygaña issued the Order of 3
July 1998.
The Supreme Court disagreed with the LRA and the appellate court's
● Enriquez explained that the inconsistency in the dates: the issuance
observation that petitioners need to file a motion to lift the order of
was on 1993 but the release and signing was on 1998.
general default. A motion to lift the order of general default should be led
● On 25 November 1998, petitioners filed with the Register of Deeds registration procured by fraud the owner may pursue all his legal
of Marikina City an application to annotate the notice of lis and equitable remedies against the parties to such fraud, without
pendens at the back of the OCTs which the LRA denied. prejudice, however, to the rights of any innocent holder for value
● The Court of Appeals affirmed the decision of LRA. of a certificate of title . . . ."
● An action for reconveyance is an action in personam available to a
ISSUES: person whose property has been wrongfully registered under the
● Whether or not the notice of lis pendens can be annotated to the Torrens system in another's name. Reconveyance is always
OCTs granting that an Order for General Default is obtained? - ​NO available as long as the property has not passed to an innocent third
(the remedy should be to file an Action for Reconveyance) person for value. A notice of lis pendens may thus be annotated on
the certificate of title immediately upon the institution of the action
RELEVANT ARGUMENTS (if any): in court. The notice of lis pendens will avoid transfer to an
● LRA: Relying on Section 24, Rule 14 of the Rules of Court, the innocent third person for value and preserve the claim of the real
LRA ruled that only a party to a case has the legal personality to owner.
file a notice of lis pendens relative to the pending case. Since a ● [Issue on the motion for general default] The Supreme Court
land registration case is a proceeding in rem, an order of general disagreed with the LRA and the appellate court's observation that
default binds the whole world as a party in the case. Petitioners are petitioners need to file a motion to lift the order of general default.
mere movants whose personality the court has not admitted. Based A motion to lift the order of general default should be led before
on Section 26 of PD 1529, the LRA ruled that petitioners should entry of final judgment. The land registration court granted the
have led a motion to lift the order of general default. application for registration of title on 31 May 1966 and issued a
certificate of finality on 8 March 1991. Petitioners led their motion
RATIO: on 16 July 1997. Thus, even if petitioners led a motion to lift the
order of general default, the order of default could not be set aside
● As decreed by Section 76 of PD 1529, a notice of lis pendens because the motion was filed out of time. Also, petitioners are not
should contain a statement of the institution of an action or mere interested parties in this case. By ling their motion to have
proceeding, the court where the same is pending, and the date of its the decrees and the corresponding certificates of title declared
institution. A notice of lis pendens should also contain a reference void, they took the role of oppositors to the application for land
to the number of the certificate of title of the land, an adequate registration.
description of the land affected and its registered owner.
● The Register of Deeds denied registration of the notice of lis WHEREFORE, ​we DENY the petition. We AFFIRM the Decision of the
pendens because "the application was bereft of the original petition Court of Appeals in CA-G.R. SP No. 55993 dated 29 November 2000.
or complaint upon which this office will base its action." Both the
LRA and the appellate court denied the application for a notice of
lis pendens because petitioners are mere movants, and not original
parties.
● Petitioners committed a fatal procedural error when they led a
motion in LRC on 16 July 1997. The remedy of petitioners is an
action for reconveyance against Sandoval, Ozaeta and their
spouses. Reconveyance is based on Section 55 of Act No. 496, as
amended by Act No. 3322, which states that ". . . in all cases of
85. SSS v. CHAVEZ FACTS:
13 October 2004 | J. Quisumbing | Topic
R. REALUBIN & MAKER ● Juanito and Agustina Obedencio filed a case praying that the
Social Security System (SSS) be ordered to cancel the mortgage on
the properties of the spouses and to release the documents covering
PETITIONER: ​Social Security System
the said properties.
RESPONDENT: ​Hon. Nazar U. Chaves, RTC, BR. 18, Misamis
● SSS filed an Answer with Counterclaim alleging that the private
Oriental, Cagayan de Oro City and Sps. Juanito & Augustina Obedencio
respondents had an unpaid obligation in the amount of P48,188.72
as of September 1, 1994.
RECIT-READY: ​Juanito and Agustina Obedencio filed a case praying
● After the issues were joined, a pre-trial conference was scheduled
that the Social Security System (SSS) be ordered to cancel the mortgage
on February 16, 1995. Atty. Rodrigo B. Filoteo, acting assistant
on the properties of the spouses and to release the documents covering
branch manager of the SSS in Cagayan de Oro City entered his
the said properties. SSS filed an Answer with Counterclaim alleging that
appearance as counsel for the petitioner. The hearing was cancelled
the private respondents had an unpaid obligation in the amount of
because the respondent judge was indisposed. The hearing of the
P48,188.72. Atty. Rodrigo B. Filoteo is the counsel for SSS. The hearing
case was reset on April 18, 1995. This time, Atty. Filoteo failed to
originally scheduled on Feb 16, 1995 was moved to April 18, 1995. He
attend because of an o cial mission to Zamboanga City from April
failed to attend the hearing. On motion of Atty. Alberto Bacal, counsel of
7 to May 8, 1995 involving SSS cases.
the respondent spouses, respondent judge issued an Order dated April 18,
● On motion of Atty. Alberto Bacal, counsel of the respondent
1995 declaring petitioner in default and allowed private respondents to
spouses, respondent judge issued an Order dated April 18, 1995
present their evidence ex parte. SSS moved for reconsideration.
declaring petitioner in default and allowed private respondents to
present their evidence ex parte.
ISSUE:
● SSS filed for a motion for reconsideration to lift the order of
Whether or not the default order of the lower court should be lifted, so
default. It was denied. On appeal, the CA also denied the petition.
that substantial justice would prevail over technical rules? -​ NO
To be relieved of the effects of the order of default, Sec. 3, Rule 18
of the Rules of Court provides that the defendant must le a motion
RULING: ​The counsel of SSS failed to comply not only with one rule.
under oath to set aside the order of default; that he must show that
Other than failing to appear during pre-trial, petitioner does not deny that
his failure to appear at the pre- trial was due to fraud, accident,
its Motion for Reconsideration to lift the order of default lacked
mistake or excusable neglect and accompany the motion with
verification, notice of hearing and affidavit of merit. If not accompanied
affidavit of merit.
by affidavits of merit, the trial court has no authority to consider the
same. A motion to lift an order of default is fatally flawed and the trial
ISSUES:
court has no authority to consider the same where it was not under oath
● Whether or not the default order of the lower court should be
and unaccompanied by an a davit of merit. In effect, the petitioner failed
lifted, so that substantial justice would prevail over technical rules?
to set aside the order of default and must suffer the consequences thereof.
-​ NO
DOCTRINE: ​The declaration of default for non-appearance at a pre-trial
RELEVANT ARGUMENTS (if any):
conference is sanctioned by Rule 20, Sec. 2 of the Rules of ​Court, thus: A
● SSS: Although the respondent judge has the discretion to declare a
party who fails to appear at a pretrial conference may be non-suited or
defendant in default for failure to appear during pre-trial
considered as in default.
conference, the strict, rigid and arbitrary application thereof denied
the petitioner a reasonable opportunity to present its meritorious
defense, refute the evidence of the private respondents, present his
own, and exercise his right to due process. The rules should be
liberally construed in order to protect the substantive rights of the
parties.
● Sps. Obedencio: They claim that they had fully paid their
obligation with the SSS. They allege that they already paid
P63,000, an amount that exceeded their supposed accountability of
P56,427.

RATIO:

● Sadly, the records reveal that petitioner failed to comply not only
with one rule. Other than failing to appear during pre-trial,
petitioner does not deny that its Motion for Reconsideration to lift
the order of default lacked verification, notice of hearing and
affidavit of merit. If not accompanied by affidavits of merit, the
trial court has no authority to consider the same. A motion to lift an
order of default is fatally flawed and the trial court has no authority
to consider the same where it was not under oath and
unaccompanied by an a davit of merit. In effect, the petitioner
failed to set aside the order of default and must suffer the
consequences thereof.

WHEREFORE, ​the petition is DENIED for lack of merit. The Decision


dated February 29, 2000, and the Resolution dated December 12, 2001 of
the Court of Appeals, are AFFIRMED. The case is REMANDED to the
Regional Trial Court of Misamis Oriental, Cagayan de Oro City, Branch 18,
for further proceedings.
1. REPUBLIC v. TSAI
prove that her possession of the subject property started since 12 June
June 22, 2009 | Carpio J. | Land of Public Domain When Alienable and
1945 or earlier because Tsai's earliest evidence can be traced back to a
Disposable
tax declaration issued in the name of her predecessors-in-interest only in
SALAMAT, J. & SORIANO, P.
the year 1948. Finally, ​we note that Tsai also failed to prove that the
subject property has been declared alienable and disposable by the
PETITIONER: ​Republic of the PH President or the Secretary of the Department of Environment and
RESPONDENT: ​Ruby Lee Tsai Natural Resources

RECIT-READY:
Ruby Tsai filed an application for confirmation and registration of her DOCTRINE:
property under PD No. 1529. She alleged that she bought the property The applicant for land registration must prove that the DENR Secretary
from the Carungcongs in 1993, and she and her predecessors in interest had approved the land classication and released the land of the public
has been in open, continuous, exclusive, and notorious possession and domain as alienable and disposable, and that the land subject of the
occupation of the property for more than 30 years. The Republic opposed application for registration falls within the approved area per verication
this alleging that Tsai and her predecessors failed to present sufficient through survey by the PENRO or CENRO. In addition, the applicant for
evidence to show that have been in open, continuous, exclusive, land registration must present a copy of the original classication
notorious possession of the subject property since June 12, 1945 or approved by the DENR Secretary and certied as a true copy by the legal
earlier as required by CA No. 141. The RTC found that the pieces of custodian of the ofcial records. These facts must be established to prove
evidence presented by Tsai was sufficient (Deed of Sale, Tax that the land is alienable and disposable
Declarations etc) and ordered for the registration of the property under
Tsai’s name. The Republic appealed, but the CA affirmed ruling that Tsai
FACTS:
does not have to prove possession since June 1945 because CA 141 has
● On 3 December 1996, Tsai led an application for conrmation
been amended by RA 1942, which provided for a simple 30 year
and registration of the subject property under PD No. 1529. She
prescriptive period.
alleged that she is the owner of the subject property and the
improvements thereon. Tsai stated that on 31 May 1993, she
ISSUE:
purchased the subject property from the Carungcongs
Whether the lower courts erred in granting Tsai’s application for
(Carungcong), through Wendy Mitsuko Sato, Carungcong’s
registration of the said property - YES
daughter and attorney in fact. She also declared that she and her
predecessors-in-interest have been in open, continuous, exclusive
RULING:
and notorious possession and occupation of the subject property
The Court of Appeals failed to consider the amendment introduced by
for more than 30 years
PD 1073. As the law now stands, a mere showing of possession and
● Except for the Republic, there were no other oppositors to the
occupation for 30 years or more is not sufcient. Therefore, since the
application. The Republic opposed on the following grounds: (1)
effectivity of PD 1073 on 25 January 1977, it must now be shown that
that Tsai and her predecessors-in interest failed to present
possession and occupation of the piece of land by the applicant, by
sufcient evidence to show that they have been in open,
himself or through his predecessors-in-interest, started on 12 June 1945
continuous, exclusive and notorious possession and occupation of
or earlier. This provision is in total conformity with Section 14(1) of PD
the subject property since 12 June 1945 or earlier as required by
1529. We agree with the Republic that Tsai's evidence was not enough to
Section 48(b) of Commonwealth Act No. 141 as amended by
Presidential Decree No. 1073 (2) that the tax declarations and tax registration of her title to the subject property under PD 1529.
receipt payments attached to the application do not constitute However, Tsai did not specify under what paragraph of Section 14
competent and sufcient evidence of a bona de acquisition of the of PD 1529 she was ling the application. But going over Tsai's
land applied for or of Tsai's open, continuous, exclusive and application and the evidence she presented before the trial court, it
notorious possession and occupation of the subject property in the appears that Tsai led her application under Section 14(1) of PD
concept of an owner since 12 June 1945 or earlier; and (3) that the 1529
subject property forms part of the public domain and is not subject ● “SEC. 14. Who may apply..
to private appropriation (1) Those who by themselves or through their
● During the trial, Tsai gave the following pieces of evidence: Deed predecessors-in interest have been in open, continuous,
of Absolute Sale between Tsai and Carungcong (Yr 1993); Tax exclusive and notorious possession and occupation of
Declarations (Years 1948, 1960, 1980, 1985); Official receipts for alienable and disposable lands of the public domain under
real property tax (Yrs 1991-1993). The trial court granted Tsai's a bona de ​claim of ownership since June 12, 1945, or
application for registration. The court ruled that Tsai was able to earlier
prove her actual possession of the land for more than 30 years and ● Thus, there are three requisites for the ling of an application for
that the subject property was residential and not within any forest registration of title under Section 14(1) of PD 1529:
zone or the public domain. The Republic appealed to the Court of (1) that the property in question is alienable and disposable land of
Appeals on the ground that the trial court erred in granting the the public domain;
application for registration despite respondent’s failure to prove (2) that the applicant by himself or through his
open, continuous, exclusive and notorious possession of the subject predecessors-in-interest have been in open, continuous, exclusive
property since 12 June 1945 or earlier. According to the Republic, and notorious possession and occupation; and
it is not sufcient that respondent proved possession of the subject (3) that such possession is under a bona de claim of ownership
property for more than 30 years. CA affirmed RTC since 12 June 1945 or earlier
● A similar right is given under Section 48(b) of CA 141, as
ISSUES: amended by PD 1073. According to the Court of Appeals, Tsai
● Whether the trial court can grant the application for registration need not prove possession of the subject property since 12 June
despite the lack of proof of Tsai's open, continuous, exclusive and 1945 or earlier because Section 48(b) of CA 141 was amended by
notorious possession of the subject property since 12 June 1945 or RA 1942, which provided for a simple 30-year prescriptive period.
earlier - NO The Court of Appeals appears to have an erroneous interpretation
of Section 48(b) of CA 141
RATIO: ● Through the years, Section 48(b) of the CA 141 has been amended
● According to the Republic, Tsai only proved possession since several times. The Court of Appeals failed to consider the
1948, which is in violation of Section 48(b) of CA 141, as amendment introduced by PD 1073. As the law now stands, a mere
amended by PD 1073. On the other hand, Tsai insists that it is showing of possession and occupation for 30 years or more is not
sufcient that she proved that she and her predecessors-in-interest sufcient. Therefore, since the effectivity of PD 1073 on 25
have been in open, continuous, exclusive and notorious possession January 1977, it must now be shown that possession and
and occupation of the subject property under a bona de claim of occupation of the piece of land by the applicant, by himself or
ownership for more than 30 years through his predecessors-in-interest, started on 12 June 1945 or
● The Court notes that in Tsai's original application before the trial earlier. This provision is in total conformity with Section 14(1) of
court, she claimed that she was entitled to the conrmation and PD 1529
● Tsai failed to comply with the period of possession and occupation
of the subject property, as required by both PD 1529 and CA 141.
We agree with the Republic that Tsai's evidence was not enough to
prove that her possession of the subject property started since 12
June 1945 or earlier because Tsai's earliest evidence can be traced
back to a tax declaration issued in the name of her
predecessors-in-interest only in the year 1948. In view of the lack
of sufcient showing that Tsai and her predecessors-in-interest
possessed the subject property under a bona de claim of
ownership since 12 June 1945 or earlier, Tsai's application for
conrmation and registration of the subject property under PD
1529 and CA 141 should be denied
● Finally, ​we note that Tsai also failed to prove that the subject
property has been declared alienable and disposable by the
President or the Secretary of the Department of Environment
and Natural Resources​. In ​Republic v Tan Properties the court
said:
● “[T]he applicant for land registration must prove that the
DENR Secretary had approved the land classication and
released the land of the public domain as alienable and
disposable, and that the land subject of the application for
registration falls within the approved area per verication
through survey by the PENRO or CENRO. In addition,
the applicant for land registration must present a copy of
the original classication approved by the DENR
Secretary and certied as a true copy by the legal
custodian of the ofcial records. These facts must be
established to prove that the land is alienable and
disposable.

WHEREFORE, we GRANT the petition. We SET ASIDE the 30


January 2004 Decision of the Court of Appeals in CA G.R. CV No.
70006 and the 21 September 1998 Decision of the Regional Trial Court
of Tagaytay City, Branch 18, in LRC Case No. TG-788. We DENY
respondent Ruby Lee Tsai’s application for conrmation and
registration of Lot No. 7062 described in plan Ap-04-010084, Cad355,
Tagaytay Cadastre.

You might also like